The Finest Argument Against Atheism

The finest argument I have ever heard against atheism would have to be the ‘argument from reason.’

The typical arguments one hears in atheist vs. theist debates generally do not interest me. However, the argument from reason is one of those arguments that would definitely cause me to struggle with atheistic claims if I were an atheist.

It should be noted that this is not an argument “for” Christianity, but rather an argument against atheism, specifically atheism animated by philosophical materialism and/or naturalism. An argument for Christianity would follow a very different set of rules.

Naturalism essentially states that nature is a closed system in which all events within the system are explainable (or are explainable in principle) in terms of the natural order. Thus, naturalism excludes any idea of “god” since the divine does not lie within the total system. If one accepts these presupposition then all events, including that of thought, must be explainable in purely natural/material terms. In short, all events must be the result of mechanical processes linked in a casual chain of events that could be traced back to the very beginning of the universe, if one were so inclined.

The trouble for the Naturalist comes into play when one considers the event of human thought. Since thoughts are events, all of our thoughts should be fully explainable in mechanistic terms and not according to a person’s free-agency. But any thought which is not guided by what is “true” but guided rather by mechanistic, physical necessity is not rational. Hence, Naturalism, philosophically speaking, slits its own throat.

Again, if our thoughts are the inevitable play of firing neurons in our brain set in motion by causal necessity then what we think would be the result of whatever the total system delivered to us and not because it accorded with “truth” necessarily. If the claims of Naturalism are held with consistency, one would have to concede that belief in Naturalism occurs only because nature has determined it (sort of an atheist’s equivalent of Calvinism). And if one arrives at his philosophy not because he chose it, but rather because it was all the total system would allow, then Naturalism is, philosophically, self-defeating.

I like what the late Professor Haldane of Oxford University said concerning the logical conclusion of a strict naturalism: “If my mental processes are determined wholly by the motions of atoms in my brain, I have no reason to suppose that my beliefs are true … and hence I have no reason to suppose that my brain to be composed of atoms.”

Victor Reppert, piggy-backing off the ideas put forth in the book, “Miracles,” by C.S. Lewis, which gave a well-articulated criticism of naturalism, gave the following syllogism to help summarize the argument:

  1. No belief is rationally inferred if it can be fully explained in terms of non-rational causes.
  2. If materialism is true, then all beliefs can be fully explained in terms of non-rational causes.
  3. Therefore, if materialism is true, then no belief is rationally inferred.
  4. If any thesis entails the conclusion that no belief is rationally inferred, then it should be rejected and its denial accepted.
  5. Therefore materialism should be rejected and its denial accepted.

Reppert continues, “Explaining how a person, as a matter of personal history, came to believe something in a rational way is critical to understanding that person as a rational agent.” Naturalism unwittingly denies such an explanation.

In essence, naturalism is a philosophy of existence which precludes human beings from being truly free, rational agents. The very power of reasoning which they use to levy arguments against God is the very power which gives witness to an “intellectual” source for reason in general. Rational inference is the ultimate elephant in the room during most atheist debates.

According to a consistent Naturalism any notion of god must be excluded, but its more than that: any notion of good and evil, right and wrong, love and hate, etc., must also be treated as mythical. At best they are nothing more than helpful categories of thought, but categories with no existence (or ontology) of their own. Life would then be essentially meaningless since its ultimate goal is mere survival, a goal wholly unattainable in a world where death is guaranteed.

This is an extremely brief treatment of the argument and I’ve tried my best to summarize my understanding of its main elements. For the sake of time I’ll let whatever discussion ensues from this article help to ferret out those finer points which were left untouched. Thanks for reading!

194 thoughts on “The Finest Argument Against Atheism

  1. What it truly breaks down to is that there is no observable evidence for a creator of our known universe. Since we have, through scientific means, so far explained many things about the natural laws of the cosmos, we can safely say (but not with 100% certainty) that what we have not yet found an answer to can be found. Another way to put it is that if we can observe it, then given enough time, we can explain it. That being said, it seems that the vast majority of people living today that do believe in a creator also believe that this creator exists in a realm which we cannot observe, no matter how technologically advanced we become. Personally, I’m fine with that aspect, for the most part. A supernatural being created the universe. If I’m not completely ok with it, I can at the very least understand it.

    What I don’t understand, however, is how people can base their whole life structure on a book that was written thousands of years ago by grown people (I can only assume) that knew less about the universe and how it works than the average 8 year old of today. To look at religion objectively, you have to understand that there have been thousands of religions, and every believer of those were just as sure they were right as anyone else. What they also have in common is lack of solid proof. Testable, observable, falsifiable evidence.

    That’s what is so great about science. We don’t merely take people’s word for it, even when they are as smart as Einstein was. Other people look at their hypotheses, ideas and/or theories and calculate, experiment, observe and do it over and over again. If it works out the same every time with many different independent researchers conducting the experiments, then we accept it. Until then, it is just an idea, and not all ideas are good.

    • Hi Jason, thanks for your reply.

      You bring up some good points with regards to putting theism on the stand, but as I said in the opening of the article, if I were to perform a Christian apologetic the argument would be profoundly different. The aim of this article is to expose what is for me the most compelling argument against atheism. Atheism and science are not coequals; they should not be conflated. Atheism is a philosophy whereas science attempts to remove philosophy as much as possible from its evaluation of the natural world and its phenomenon (though such a goal is often thwarted when science is removed from its proper context and used as a philosophical tool against theism).

      But as far as science rejecting God due to not having observable qualities in the natural order (my words, not yours), I would argue that science has no commentary on non-observable phenomenon, whether divine or cosmological, as there are plenty of natural phenomenon that resist scientific commentary – the epitome of which would have to be the mere existence of the natural world in the first place.

      One point I would most like to comment on is your assumption that ancient man was ignorant of cosmology due to the fact that they, “knew less about the universe and how it works than the average 8 year old of today.” I would argue that ancient man was far superior to we moderns who spend half of our lives in front of computer and television screens whereas the ancients spent 100% of their lives in full communion with nature. These were men and women whose lives depended on their close association with nature, people who were perfectly subjected to their world and “subjects” rather than mere “observers” of their world. The sky they saw at night is a sky which none of us alive today have ever seen in all its brilliance due to our “advanced technology” of artificial light. One has to get 500 miles out to sea before they can see the same sky the ancients saw undefiled by city lights. We have an inferior relationship to our world in that our relation to it is one of “objectivity.” The ancients had a relationship of “subjectivity.” As Kierkegaard rightly said, “Truth is subjectivity.” Thus, we may have more “knowledge” about the cosmos, but they had more “insight.”

      Lastly, your point about basing one’s entire life on a book written by these ancient men as being terribly flawed. I would agree. As an Orthodox Christian my life is based not merely on a text but a Holy Tradition which includes the text and the experience of 40 centuries of spiritual refinement and wisdom that no science classroom in the world would even jokingly pretend to replicate. But, there are many Christians who do just as you claimed and I agree with your conclusion.

      Cheers. 🙂

    • I suppose I did get off the specific topic. I agree that there were some advantages of living back then. Seeing the beauty of the night sky unimpaired like you said, but probably the most important being that many people hunted and grew their own food so they likely had a deeper appreciation for what ended up on their plates. Our modern technology has afforded us many luxuries like modern medicine, but we have done a fine job of separating ourselves from the natural world that we claim to know so much about.

      As for Christianity, I think it would garner more respect among nonbelievers or those of other faiths if there weren’t so many denominations, different versions of the bible, interpretations of the bible, etc. I’ve said before, maybe too hastily, that the world would be better off without religion, but I think I’ve come around to thinking that if the religions weren’t so incredibly divided, that would be a more suitable way to address the debate.

      One of the problems I’ve had with Christians blogs is that many times, even when I am respectful, my comments aren’t approved for the simple fact that I disagree. If they are that afraid of differing opinions then that doesn’t say much about their faith, does it? Anyway, it’s nice to see people like you who are open to discussion about it even though we don’t agree. We haven’t advanced far enough in technology to live on different planets, so we might as well get along even if our viewpoints aren’t the same. One thing I can be fairly sure of, though, is that most people are good regardless of their beliefs. If you look around you’ll see the the majority of society treats others like they would like to be treated. At least most of the time. That isn’t called the golden rule for nothing, my friend. Peace 🙂

    • Jason, I share your frustration concerning the attitude of many Christians when confronted with challenges to their faith. The reactions you describe are almost always set in a deep discomfort and lack of integrity of their faith. No person of real Christian insight would fear any challenge to their faith, that’s how I see it. You aptly express why I became an Orthodox Christian; our understanding of Scripture and tradition hasn’t required a Reformation or division of thought in 20 centuries.

      Indeed, denominationalism is the single greatest witness against Christianity and the unity of the Church its greatest witness for Christianity, and their’s only one pre-denominational Church that can claim the latter.

    • I just read your post that’s like this, put titled a fine argument against Christianity, and I saw something you said here that I think can relate back to the other post.
      I found what you said here interesting, specifically, “We have an inferior relationship to our world in that our relation to it is one of “objectivity.” The ancients had a relationship of “subjectivity.” As Kierkegaard rightly said, “Truth is subjectivity.” Thus, we may have more “knowledge” about the cosmos, but they had more “insight.” ”
      You’re right, the ancients saw things subjectively – meaning that had no real reason or logical thought process, only personal experience. So they sparked the beliefs that (at least, this is how I view it) eventually led to your idea of God. They didn’t understand objectivity because they didn’t have any decent ways to test their theories, just believe. So in a way, all ideas of God are the result of disunity from the very first God or Gods, because the idea of a God started with these ancients, likely as a means to explain the world around them. Humanity’s ideas about God went on from there, and more religions were created – everything from Hinduism, to Paganism, to the Gods of the Romans and Greeks which we regard now as stories, to the Judeo-Christian beliefs of today – in fact, many of the teachings and mythology surrounding Christianity appeared in many religions that came before it.

    • Thanks for the reply, Katie. Let me make one main response to your post and see what you think. What Kierkegaard, and I by extension, mean by “subjectviity” is that one is personally involved with, or subjected to a relationship with, something going beyond mere objectification. Similar to the scientist who studies marriage relationships, but is not married vs. a married person with a “subjective” relation with the notion of marriage. Which would you say “knows” about marriage? Well, they both do, in entirely different ways. Kierkegaard would say that the one who is actually engaged in a marriage relationship has far more “knowledge” about marriage than the mere observer. The observer could give you stats and fact about somethings, but would have no idea what it means to “be married” like the one who is married. Relate all this to the idea of God and nature and you’ll see where I was coming from in the article.

    • Hm, I’m not sure I’m getting the analogy, because to me it doesn’t seem that it’s very equal. Maybe I’m being thick, but I’m not sure how this compares in a good way. If you wouldn’t mind explaining it from a different angle? I don’t want to respond yet because I don’t think I fully understand what you mean by this.

    • Well, in other words, the ancients may have experienced God as you said, due to a lack of scientific sophistication, or… they may have been more in tune to the reality of God because they had an immediate relationship with nature and the world. I don’t know about you, but I can spend all day sometimes in front of a computer or in a book and barely realize there is a world outside my window. This scenario has a blinding effect on our relationship to reality, even though we may be studying science in a textbook. The ancients had no such setbacks. There “science” was much more sophisticated then you may be giving it credit for (read some of the accounts of the level of scientific development in the ancient Roman and Byzantine world from which Christianity sprung and you might think the modern West is actually behind in many ways), but besides science, per se, they had a living, working relationship with the world in ways that we simply don’t have in modern society. If anyone was going to discover a real and living God it would be those most involved with reality. That gives the ancients the advantage. But, that’s just my opinion.

    • I apologize for jumping into this conversation late but just discovered this wonderful blog. ThatCatKatie, you make some valid points that can be addressed and I will try to do so.

      1. You accuse the ancients of having no reason or logical thought processes but only personal experience. This is not true and what you have done many people, including Christians, is make the mistake of confusing the lack of knowledge with lack of logic and reasoning. As if logic and reasoning magically appeared around the 16th Century.

      Man has always been a logical being capable of reasoning. The lack of knowledge may cause one to arrive at the incorrect conclusion, but Scientists are still doing that in this century even with a vast amount of knowledge available to them. The behavior of light, the theory of relativity and quantum physics upended all of their conclusions in the mid 20th century and they are still struggling to assimilate things they have learned in the last twenty years.

      I can give you an analogy. The ancients did not have the knowledge that red blood cells carried oxygen to your body in your blood but they were fully aware that blood was vital to living and if you lost enough of it you would perish. The ancients did not know that a man’s sperm would fertilize a woman’s embryo and thus produce a baby, but they were fully aware that a man’s semen was needed to produce a child as this was one of the first primitive forms of birth control (along with the calendaring method as they understood there were only certain times that a woman was likely to get pregnant during her cycle).

      All of these correct conclusions made by the ‘ancients’ came from observation, hypothesis and testing. Signs of logic and reasoning. Yes they came to the wrong conclusions on many things, but so have scientists since the ‘Enlightenment’. Scientists my know that a sperm cell will fertilize an embryo but how the cells understand when its the right time to develop the brain, grow tissue, bone and muscle and when to stop is still a mystery. Scientists still do not understand the mechanism of how a baby is developed in a female womb.

      Scientists understand that there are forces in physical world, but that doesn’t mean they actually understand them. Take gravity for instance. They know its there. They can see and measure the effects of gravity. But just as the ancients knew if a man bled out he would die and not know why scientists today know gravity exists but have no idea how or why.

      2. You stated that the ancients used the ideas of God(s) to explain the world around them. This is not true. They believed in the idea of God(s) to explain philosophical questions. Philosophical questions are questions that Science can’t and never will be able to explain. The reason most people, including Christians, do not understand this is because they really don’t understand Science. Most people believe that Science is a way for human beings to get Truth with a capital ‘T’ using reasoning and logic. But that is not what Science is by a long shot.

      Science is the methodology used to explain how the mechanisms of nature around us actually work. The methodology used is, Observation, Hypothesis, Testing, Conclusion. Rinse, repeat. I am simplifying a bit but in reality Scientists would probably do better if they stuck closer to this methodology.

      Observation is the problem child for Scientists. It is particularly problematic for what I call the ‘Historical Sciences’. Scientists can postulate how old the world is but in the end nobody has a clue because they cannot historically go back in time to see the Earth form and know what the environment was like during that time. Everyone is guessing. Scientists can say there is no God, but they can’t get outside the system to prove it. You can say there is nothing outside the system but you can’t prove that either because your the fish in the aquarium. The fish in the aquarium can deny there is anything outside the aquarium but until he is prepared to die to find out he won’t know.

      So the ‘ancients’ were not being irrational by stating that they believed in a Creator, Supreme Being(s), a Force that created what we see. They were being philosophical. Philosophy uses logic and reasoning to answers questions that can never be answered by Science. Philosophy says, ‘I have never ever, and I mean not once ever, nor has any other reliable human being on the face of this Earth ever seen something arrive from nothing. So if I am experiencing something then it must have come from something else. Logically then, something created everything we see around us, something outside of the present system.”

      The ancients were being incredibly logical to arrive at a reasonable philosophical answer that there is a force outside this system that created this one. Now they may have made their force [God(s)] silly and human like and may have been wrong about the characteristics of their God(s) but that is a Philosophical argument, and not a Scientific one.

    • Very good reply. One of the best explanations, albiet short and summarized, about the Scientific Method I’ve read in a while. As a Believer and a sort of Science Geek (lol), I do wish scientists today, would stick to the strict method that they’re supposed to go by. I honestly think that there would be less Scientists who are Atheists if they really did keep to that methodology.

    • I find there to be 4 errors in your line of reasoning.
      First, you say there is no observable evidence for the existence of God. This is a subjective statement, not an objective statement, as a Jew or Christian would counter that the existence of all of Creation is observable evidence of the existence of God. [See fourth point on evidence vs proof]
      Second, science deals solely with nature, and is ill-equipped to deal with anything that is outside of nature: value, ethics, morality, history, existential truths and abstract concepts to name a few. Science cannot prove the very laws of nature that it relies on through empirical testing; they’re just “taken on faith” to exist.
      Thirdly, empirical evidence is not the only type of evidence available. There’s also testimonial (written or verbal), inductive and deductive evidences, all of which provide evidence.
      Fourth, I think you may be confusing evidence with proof. Evidence is not the same as proof. You begin by stating there is ‘no evidence’ and end by stating there is ‘no proof’. Evidence is the interpretation of information gathered from multiple sources. Proof is the conclusion drawn based on the body of evidence.

    • Atheists belief we are physical beings and nothing else. But how can ANY physical body have a free will that is independent of external conditioning? Is it possible to NOT eat when your hungry? Can you do, think, feel, dream or initiate any action that isn’t simply a complicated stimulus/response system? If so you possess this ethereal, non-physical thing called “will” which proves that at least some part of your being is not dependent on the physical universe otherwise ultimately everything you do could be predicted.

    • “When one honestly assesses the Judeo-Christian doctrine of God he will find multiple thousands of years of human testimony and religious development;”

      So? People in Rome used to believe in Zeus. People in Greece believe in Jupiter. People in Egypt believed in Amun-Ra. People in Norway used to believe in Odin. Were these gods real just because somebody believed they were real? What kind of “logic” are you smoking?

    • “Why can’t there be a physical explanation of existence? Because anything physical is, by definition, something that exists. So there cannot be a physical cause of existence.”

      Why do we need an explanation for existence? Would we cease to exist if there was no reason for us to exist? Isn’t it just hubris (vanity) that causes us to believe that there needs to be a reason for our existence? It is like a puddle of water admiring the hole it finds itself in and figuring that the hole must have been “designed” just for it because it fits the hole so perfectly. Isn’t it more likely that we just simply exist and that if we didn’t exist we wouldn’t be here wondering why we existed? Supposedly if we didn’t exist then somebody else would exist and they would wonder why they existed.

      Do you realize that because the speed of light is constant that we can only see objects that were 13.7 billion light years away 13.7 billion years ago? That means that there could very well be objects that were 13.8 billion light years away 13.7 billion years ago and we won’t see those objects for another 100 million years. The universe could be infinitely large: the assumption that the universe started at a single point may be erroneous. The big bang did not start at a single point: the big bang started everywhere. The big bang was simply the point in the past when the universe was the most dense but that doesn’t mean it started from a single point: the universe may have come into being because it has always existed.

      Do you know who was the first to argue this? Saint Augustine. From “Confessions” Book Eleven Chapter 13:

      “But if the roving thought of someone should wander over the images of past time, and wonder that thou, the Almighty God, the All-creating and All-sustaining, the Architect of heaven and earth, didst for ages unnumbered abstain from so great a work before thou didst actually do it, let him awake and consider that he wonders at illusions. For in what temporal medium could the unnumbered ages that thou didst not make pass by, since thou art the Author and Creator of all the ages? Or what periods of time would those be that were not made by thee? Or how could they have already passed away if they had not already been? Since, therefore, thou art the Creator of all times, if there was any time before thou madest heaven and earth, why is it said that thou wast abstaining from working? For thou madest that very time itself, and periods could not pass by before thou madest the whole temporal procession. But if there was no time before heaven and earth, how, then, can it be asked, “What wast thou doing then?” For there was no “then” when there was no time.”

      St. Augustine was answering the question “What did God do before he created Heaven and Earth?” and he said there was no time before the creation of Heaven and Earth. But that means that Heaven and Earth have “always” existed as there was “no time” when they didn’t exist! So why does the world need a creator?

      Look at it this way, the universe has always existed because as you go farther and farther back in time all the matter of the observable universe is concentrated into a smaller and smaller volume. But as this happens space-time gets warped more and more by the gravitational forces due to the newly formed matter in the universe. As a result, time at the beginning was so slow that as you approach the “beginning” time stops. Thus there was no beginning because as you go back farther and farther in time there is no actual point in time that is the beginning: science has thus shown that not only is there no time before the beginning of time but also no point in time when time actually began. Again, then, what use would be a creator?

    • The argument given in this article is essentially proof by assertion. The reality is that the material world is everything we can see, hear, touch, smell, taste or otherwise measure with scientific instruments. Anything else is imaginary. If your god is not part of the material world then it is imaginary. QED.

  2. You cannot imagine that there would be a process where by consciousness would be better if it tracked reality than if it fabricated it, and the better processes succeed and propagate and the worse ones die off?
    Oh well.

    • Allallt, see Repperts points 2 and 3 and let me know if you find a flaw in his argument.

      As to your overall point, I can see how a physical object like the eye could evolve from a simple receptor able to sense light and dark into a full eye able to see minute distinctions in its environment. But I do not see how the evolution of sight could ever develop into “insight.”

    • There is no reason that non-rational causes necessarily lead only to non-rational results.
      Every “emergent” property is testament to the development of a new character, a character not present in all the lead-up causes.
      Every component of a plane is not a plane.
      This fallacy has a name: the fallacy of composition.

    • Allallt, you’re making some good points, but skipping over the fundamental issue. Under a strict natrualistic rubric, all events within the closed physical system must be explicable in mechanistic terms. Our thoughts are events. The minute one poses that our thoughts are explicable in mechanistic terms we can no longer claim to have rational inference, and rational inference is essential for anything human to work, not the least of which are the natural sciences.

      If this is still not understood, perhaps we could go a different route: In a purely material universe where would our ideas of good and evil, right and wrong, should and should not, etc., fit in? As mere organisms, evolutionary processes would endow us with superior survival traits. If one attributes the belief in good and evil to mere survival traits then good and evil, right and wrong, are wholly imaginary (and worse, completely counterproductive since these ideas have produced every war in history). The problem is, there’s never been a human being, outside of those with an extreme psychopathology, that has been able to live in harmony with this conclusion.

      But it could just be that you personally are not a strict naturalist. Fine by me. But it undercuts a thorough going atheism in my estimation.

    • You’re still defining naturalism under the fallacy of composition.
      But I do have an interesting point to make regarding good/evil and should/should not. There is no ultimate ‘should/should not’, there is merely an evolved priority.
      And it seems we evolve a priority in keeping with morality, which ultimately is out there (approximately as Sam Harris describes it).

    • Allallt, did you make this observation on your own agency or was it the result of natural determinism? Kidding. Thanks for your input. I’ve enjoyed our exchange. 🙂

    • Hmmm, willing to entertain it, but failing to see the connection. Just because “rationality” and “non-rational” are used in the same sentence does not make it begging. If you follow the content and logic there is no such fallacy.

    • It’s been a long time since logic class, but I think it fits the bill. The set of rationally inferred beliefs wouldn’t seem to necessarily exclude the set of beliefs that can be fully explained in terms of non-rational causes (some beliefs might be overdetermined for example), unless one implicitly assumes that one can hold a rational belief if and only if it is rationally inferred. The question this would avoid is “what is rational?”, a much more difficult and interesting question, I think. Is there truly a difference between our notion of rationality and our basic notion of causality as both include similar ideas about priority, dependency, etc. In that case, the “non-rational cause” may be a much more slippery creature than this argument implies.

    • Keithnoback, I’m not sure if I’m understanding you fully, but let me take a shot at a reply. The statement is not setting up multiple sets of beliefs: e.g. set 1, rationally inferred beliefs, set 2, beliefs with non-rational causes. Instead it is simply stating what naturalism explicitly states: that all events can be explained in terms of mechanical and/or non-rational (non-thinking) automatic processes. Since thought (and specifically for his syllogism – rational inference) is an event, according to naturalism, all thought must be explainable in mechanical terms. Once this bridge is crossed, a thorough going naturalism must deny rational inference since it denies humans from having free agency.

    • The statement does want to discuss all beliefs, but it seeks to characterize beliefs as either rationally inferred or fully explainable by non-rational causes. It seeks to do so by definition (the premise proves the conclusion), which begs for an explanation of the definition (of rationality). Some would say that rationality is what it is and therefore cannot be reduced or explained. In that case, they have made their assertion and the rest of the world can take it or leave it as they feel fit.
      I think you rightly conclude that the whole argument is really about determinism (lack of “free agency”, another troublesome phrase), and a particular take on determinism at that. Not everybody who is a materialist or physicalist or naturalist, or whatever you want to call that general territory of thought, subscribes to that take on determinism in the first place. Non-reductive physicalists, for instance, might be offended on finding themselves lumped in with the elimitivists.

    • Well done, Keith. I mean that. It’s good to see the level of thought you have contributed to this post. I fear, though, that if every argument had to first define every term and phrase we would have no arguments, dissertations yes, but regular old fashion arguments no. The syllogism, any syllogism, should not have to shoulder the burden of defining its every element. Think of the famous, “All men are mortal, Socrates is a man, Socrates is mortal.” This argument does not attempt to define “mortal”. Under your rules this argument begs the question. Indeed probably every syllogism ever conceived would then beg the question.

      (Not sure why WordPress is not recognizing me even though I’m logged in, but this response is from Eric Hyde.)

  3. Hi, Eric, nice blog you have here, it looks like I have some perusing to do. I was actually doing some research on The Argument from Reason, and I plan on buying Reppert’s book. This stems from a recent debate which is going on at Reppert’s blog concerning the argument.

    • Zia, its a great book. I know there are others out there, but Reppert’s is pretty thorough. GK Chesterton had a lot to say on the matter as well and injects his famous wit into the debate.

  4. Pingback: On New Blogs and Not So Fine Arguments « The Caveat Lector

  5. Your argument is that thought and “clashing of neurons etc.” are different. Please insert the ” naturists” description into your argument all the way through (replace “thought” with “clashing of neurons etc.”) and magically the apparent logic to your argument falls apart…just try it. It is the same old “I think therefore i am”. But this is not a necessary proposition.

    • Actually no. I’m stating that the Naturalists doctrine claims thoughts ARE clashing neurons, nothing more. Is this not correct from your vantage point? If not, I would be interested to hear your understanding of what thoughts are within the Naturalist rubric.

      Cheers.

  6. Pierre Teilhard de Chardin, in his book “The Phenomenon of Man”, wrote:

    “We are not human beings having a spiritual experience; we are spiritual beings having a human experience.”

    Imagine the multitude of experience there is to be had, even that of the human atheist.

    Objectivity and subjectivity pared are blessed with understanding.

    • Ben, can a thorough and honest atheist have a cosmology and and its concomitant anthropology which differs in any serious way from a strict naturalism? If so, what?

      (I’m choosing to stick with my chosen terms, until the definition of physicalism you ascribe to is given).

  7. Humans are not rational by definition, but they can think and behave rationally or not, depending on whether they apply, explicitly or implicitly, the strategy of theoretical and practical rationality to the thoughts they accept and to the actions they perform. Theoretical rationality has a formal component that reduces to logical consistency and a material component that reduces to empirical support, relying on our inborn mechanisms of signal detection and interpretation. Mosterín distinguishes between involuntary and implicit belief, on the one hand, and voluntary and explicit acceptance, on the other.

  8. Pingback: back to basics | jurnalrohaniku

  9. Why trust your mind if there is a God who is perfectly capable of deceiving you without your knowing it? God could have easily created the universe last Tuesday, and created false memories in us all.

    And you can’t say, “Oh, He’d never do that” because your beliefs about God’s nature could be part of the created deception. Besides, if there’s a good reason for God to let babies die of genetic diseases, there could very well be a good reason for him to deceive us completely. (His ways are higher than ours, after all!)

    Here’s an analogy: if you’re lost in the woods, you may not have any way to know if your compass is working correctly. That’s naturalism. But theism is more like being lost in the woods following a stranger who swears he’s honest, but won’t show you the compass. Now you not only have to worry about whether the compass is working correctly, you also have to worry about whether the stranger is lying to you. Theism has two problems; naturalism only has one.

    Actually, I don’t believe there’s a philosophical problem at all, only a scientific one. Our reasoning processes are faulty whether or not theism is true. That’s an observable fact that everybody, not just the naturalist, has to account for. Theists also have to explain why we can trust our reason even though it’s faulty. But if they managed to do that in a way that ruled out naturalism, they would still have to explain why we can trust that God is not deceiving us. Even if random collections of atoms can be wrong, at least they can’t actively deceive you. God could.

    So how can the theist trust his own mind any more than the naturalist?

    • Leon, thank you for your post.

      Assuming philosophic naturalism is correct, your entire response was simply what you “must” have said given the particular cause-and-effect chain of mechanical processes leading up to your response. You did not rationally infer your argument, it was given to you by clashing atoms working within your brain without influence from you as a free-agent. I am responding in a like manner. In short, we are only having a conversation “determined” by natural mechanical processes. Neither of us are “right” or “wrong,” for right and wrong in this paradigm do not exist. Hence, it would be ridiculous to speak of a deceiving God. The fact that you are able to conceive of such an alternative is demonstrative that your premise of naturalism is false.

      As to whether or not one can trust God, since He is able to deceive without detection, is an interesting thought. My response is simple: If God is a deceiver then He cannot be God – He cannot be the “way the truth and the life”. If one believes in truth and deception to begin with, he must have some source of truth to which he establishes its existence. If “truth” exists, God exists. And if God is truth He cannot also be “untruth.” I’ll just throw it out there and see if you want to take it further. As I said in the original post, this argument is made against atheism, not for Christianity. To begin an argument for Christianity I would use a different dialectic. This argument is only meant to pull the feet out from under a strict philosophical naturalism. Once that is accomplished, an atheist is left with little else to cling to for a thorough-going atheistic philosophy of life.

      Cheers.

    • Thank you, Eric, for your speedy reply to a year-old post.

      I don’t think you’re aware of the diversity within atheism, or naturalism, for that matter. The Argument from Reason is, properly speaking, an argument against hard determinism, not against naturalists, and while there is significant overlap, the two are not the same. It’s somewhat like how some atheists attempt to refute biblical inerrancy and think that they’ve therefore shown Christianity as a whole to be false.

      I, for one, though a naturalist, am not a hard determinist. In part it is because of a similar argument, though presented in a slightly different form by the atheist novelist/philosopher Ayn Rand. She argues for the existence of free will, and is commonly labeled a compatibilist. So the Argument from Reason does nothing to undercut naturalists/atheists who accept the concept of free will. You have to do more than assert that such a naturalism is incosistent, or quote naturalists who say so. You have to positively argue that free will is impossible unless there is a God. And considering that Christians have been struggling for centuries to explain how free will is possible with an omniscient God, that’s a big hurdle to leap.

      In any case, I was not arguing for naturalism. I was pointing out that the same argument applies to theists, so they are no better off than naturalists, and possibly worse off.

      I do not find your reply persuasive. It seems that you are defining God in such a way that is convenient for your theology. I’m sure you would object if I said, “If God is a killer then he cannot be God.” But we ordinarily think that murder is worse than deception. I do agree that there must be truth of some kind, but I was arguing that God is capable of systematically misrepresenting it without us even suspecting it.

      In any case, even if naturalism cannot account for knowledge claims, that doesn’t make it untrue. Maybe the philosophical skeptics are right – truth claims cannot be verified. I sure hope they’re not right, but that doesn’t prove anything one way or the other. The same goes for moral claims – even if right and wrong make no sense under naturalism, that doesn’t make it untrue. At that point, we would have to take nihilism seriously. That’s not something most people want to do, but we can’t just say, “If there were a God, He’d fix this philosophical problem. Therefore there must be a God.” The Argument from Reason, even at its best, does not show naturalism to be false, but only extremely undesirable.

      Thanks again for your reply!

    • “Thank you, Eric, for your speedy reply to a year-old post.”

      The pleasure is mine. 🙂

      “I don’t think you’re aware of the diversity within atheism, or naturalism, for that matter. The Argument from Reason is, properly speaking, an argument against hard determinism, not against naturalists, and while there is significant overlap, the two are not the same.”

      Good point, and yes, I am aware of said differences; however, when I imagine myself holding an atheistic paradigm I think in terms of strict philosophic naturalism (because for me it is the most consistent and honest view for a full-orbed atheism). Again, remember, my article is not an argument that I believe is most convincing for everyone; rather it’s the one that would shake me up the most if I were atheist.

      “I, for one, though a naturalist, am not a hard determinist. In part it is because of a similar argument, though presented in a slightly different form by the atheist novelist/philosopher Ayn Rand. She argues for the existence of free will, and is commonly labeled a compatibilist. So the Argument from Reason does nothing to undercut naturalists/atheists who accept the concept of free will. You have to do more than assert that such a naturalism is incosistent, or quote naturalists who say so.”

      I will level with you, I’m not a huge Ayn Rand fan. She’s fantastic on many counts, but I agree with one of her critics who calls her philosophy “stillborn.” I think she makes a calculated exception for reason to escape the grasp of physical determinism because it is so deadly for someone like her who needs to believe her views are a result of her own doing – that she is a free agent. Hell, what would she do with her political philosophy if she didn’t believe in freewill? It would be comical. And, for me, she is.

      “You have to positively argue that free will is impossible unless there is a God. And considering that Christians have been struggling for centuries to explain how free will is possible with an omniscient God, that’s a big hurdle to leap.”

      Actually, this has been an issue for Latin based Christianity. It has not been a hurdle for the modern Eastern Church, nor for the ancient Church, which history bears out.

      “I do not find your reply persuasive. It seems that you are defining God in such a way that is convenient for your theology. I’m sure you would object if I said, “If God is a killer then he cannot be God.” But we ordinarily think that murder is worse than deception. I do agree that there must be truth of some kind, but I was arguing that God is capable of systematically misrepresenting it without us even suspecting it.”

      You may need to review the rest of that paragraph where you quoted me. My explanation is partly there. In short, if one believes we must strive for truth, because truth is real and not some imaginary, deterministic, evolution f—k up on a grand scale which only the human race is cursed with believing in, then one must hold a teleological cosmology. More than that, they must hold that there is an ultimate standard from which truth is based. Christians believe this truth is God Himself. Ultimate truth cannot simultaneously be non-truth. The atheist who holds a teleological cosmology must answer for what exactly they mean by truth, good, evil, false, etc without crossing over into the divine. If I were atheist, I would not be able to pull this off. Maybe you or someone else can. If so, I’d love to hear it.
      Anyway, great discussion. Thanks again for your posting.

    • “Good point, and yes, I am aware of said differences; however, when I imagine myself holding an atheistic paradigm I think in terms of strict philosophic naturalism (because for me it is the most consistent and honest view for a full-orbed atheism). Again, remember, my article is not an argument that I believe is most convincing for everyone; rather it’s the one that would shake me up the most if I were atheist.”

      Why do you think that the most consistent, honest naturalism is the one which has such a fatal flaw? Other brands of naturalism, such as soft deterministic ones, or compatibilist ones, are not affected by the Argument from Reason. What redeeming qualities does hard determinism have that makes you prefer it over other varieties of naturalism?

      In my worldview, the Argument from Reason only shows that hard determinism is incoherent. I take free will to be a natural, emergent property of consciousness. Animals have consciousness of their surroundings, but only humans have consciousness OF their consciousness. Only we are aware THAT we are aware. It is this unique property that enables us to make free choices, and to direct our consciousness in ways that make true knowledge possible. I’m still unclear on what exactly that means or how it comes about, but I don’t find this any more troubling than being unclear on what a memory is or how exactly we are able to form them. And as with memories, babies don’t appear to possess that self-reflective awareness, which leads me to believe that it is a natural phenomenon that requires a certain critical mass of neuronal activity, not a mysterious property conferred by a god at conception or birth. If we truly do have free will in a naturalistic universe, then the Argument from Reason fails. Do you think there is some even more serious fatal flaw in my account of free will than the flaws you see in hard determinism? Are there any ways in which it seems inconsistent with a thoroughgoing atheism?

      “I will level with you, I’m not a huge Ayn Rand fan…”

      I agree that Ayn Rand is not the best, but in this case I found her persuasive. I’ve read a lot of her work, and I couldn’t resist the comparison because the arguments are so parallel, even though one comes from an atheist and another comes from a Christian. I think she would be well worth reading to clarify the Argument from Reason, and I’ve yet to hear acknowledgment of Rand’s argument, much less a response, from any hard determinist. Here’s a quote to illustrate:

      “Can one prove that man’s consciousness does not function automatically, as determinists would say? If man’s consciousness WERE automatic, if it did react deterministically to outer or inner forces acting upon it, then, by definition, a man would have no choice in regard to his mental content; he would accept whatever he had to accept, whatever ideas the determining forces engendered in him… Volition is inescapable. Even its enemies have to accept and use it in the process of any attempt to deny it.” (Objectivism: The Philosophy of Ayn Rand, pp. 69-70).

      “Actually, this has been an issue for Latin based Christianity. It has not been a hurdle for the modern Eastern Church, nor for the ancient Church, which history bears out.”

      I find Eastern Orthodoxy fascinating, but I do not understand what you mean here. Could you please clarify why the Eastern church has not had a problem reconciling free will with divine omniscience?

      “The atheist who holds a teleological cosmology must answer for what exactly they mean by truth, good, evil, false, etc without crossing over into the divine. If I were atheist, I would not be able to pull this off. Maybe you or someone else can. If so, I’d love to hear it.”

      I will leave off good and evil for some other time, but I will try to explain what I mean by truth. I often say that I am a truth-seeker first, a rationalist second, a naturalist third, and an atheist fourth. My definition of the truth does not exclude the possibility of the divine, but neither does it assume that the divine must exist in order to “account” for it or to be its “source.” I reread your original statement, but I still do not understand why you think those things are necessary. The way I see it, truth is simply that which really exists, whatever its nature may be. It is the source of all else, and cannot be accounted for, because it must be used in order to account for other things. Under this definition, even a Christian should place truth-seeking above serving God, because if God were not real, He would not be worth serving. The truth could very well be that we are a deterministic, evolutionary f-k up, and if so, we should accept that, whatever “accept” means in such a case. We should accept the truth even if it’s disappointing, depressing, or downright scary, because if our actions are based on untruth, if they are not aligned with reality, they cannot bear good fruit.

      I have not yet read Reppert’s book, and I’m a little bit suspicious that it would not be worth my time, as it does not appear to challenge my own worldview. Is there any reason you would recommend it over a more thoroughgoing Christian like Alvin Plantinga, whose Evolutionary Argument Against Naturalism seems very similar to the Argument from Reason?

      Thanks again for your time.. cheers!

    • Leon, great response! I’m responding here to let you know I will respond at a later time below (at the very bottom of the thread). My time is crunched at the moment and I do not want to post a hasty response. You’ve been a delight to banter with and want to give it my full attention when I don’t have a thousand things going on. Till later, cheers!

    • Naturalism has two problems, not one. The second problem is the assumption that there isn’t anything outside the system. This is the mother of all assumptions. There is no way to get outside the present system so there is no way to prove that assumption. You can believe it, but it has to be by faith.

      If I cannot trust my mind then then all arguments are moot; including naturalism. There is one truth we all live by: I exist. If that truth is a lie then everything is damn nonsense. And we know everything is not damn nonsense because of two truths; if everything was nonsense we wouldn’t know it and we don’t live that way. This is like the Buddhist who says that pain does not really exist if we really understood pain. My response is to punch him.

      I will have to disagree with Eric on the deceiving God question. This is what C.S. Lewis called the “invisible cat argument.” That argument goes like this; “If there was an invisible cat on the chair I would not be able to see him. Since I cannot see him then there must be an invisible cat on the chair.”

      “If God was a deceiving God then I would not know it therefore he can/must be a deceiving God (or I can’t trust my mind).”

      You can believe this Leon, and I can’t argue against it but it is, in Vulcan, “completely illogical”.

      If your beef with the existence of God is that he allows pain, that is a valid point and debatable. But that we can’t believe in him because he could possibly be deceptive is not.

      To answer your final challenge we need to understand the differences between Science, Philosophy and Reasoning.

      Human reasoning is unique and separates us from the animals and computers. Reasoning is the ability to come to the correct answer without all of the data. Example; man figured out long before Science that it was his sperm that impregnated a woman. This was before we knew of the biological process. Man knew that blood carried life in the body long before they knew about red blood cells. In fact, man reasoned that if there was a God then there would be order in nature and therefore its mechanisms could be discovered. Hence Reason gave forth to Science.

      Science is simply the mechanical understanding of the forces around us. Scientists do not, and have not ever discovered a new thing. (Engineers take the mechanical understanding of how things work and do create new things however). Using the Scientific methodology (Observe, Hypothesis, Test, Rinse & Repeat) they are able to discover the mechanisms of how things work. Therefore Scientists can never tell us how old the Earth is; how the Universe was formed, how the forces of Physics came to be and why the are they way they are. Historical Science does not exist as it violates the first tenant of Science which is Observation. Historical events cannot be observed in their environment so everything is a theory at that point. Some theories may be better than others, but they are still just theories and will always be.

      Philosophy uses human Reasoning where Science cannot go. Naturalism is a Philosophical problem, not a Scientific one. Scientists are fish in the aquarium, they can never get outside the aquarium to tell you how the aquarium got there (they can, but they have to die so they can’t come back and tell the other fish). Assuming you cannot get outside the Aquarium is just that; an assumption and not Science.

    • Hi savebyj,
      Thank you for your thoughtful response. I actually agree with a lot of what you said, but I was simply responding to the Argument from Reason in a tit-for-tat fashion. I will try to make it clear in this post what it is I do and do not accept as true, rather than argue purely hypothetically.

      You pointed out a problem with naturalism, as you see it, that “There is no way to get outside the present system so there is no way to prove that [there isn’t anything outside the system].” I agree, but I don’t see this as much of a problem, and for the same reasons as your “invisible cat argument.” We can hypothetically imagine that there’s something outside or beyond the physical universe, something that is completely undetectable, just as we can imagine an invisible cat. But that doesn’t mean there has to be such a thing.

      Further, the assumption that there is nothing outside the physical world is bolstered by three observations most people agree on: 1) The existence of the physical world has been proven, but the existence of something outside it has not yet been. 2) Increasingly intimate study of our universe has not yielded anything which is unexplainable within a naturalistic framework. 3) Many phenomena that were once thought to be obvious proof of supernaturalism (such as weather and mental illnesses) are now known to be natural. None of this amounts to a proof that naturalism is true, but it does justify the assumption that we should proceed as though the natural world is all there is until proven otherwise. I would not call that “faith.” I usually define faith as belief in something that is disproportionate to the evidence for it, and I do not think that qualifies, but if you’re using the word in some other sense, it might be an appropriate label.

      You say, “If I cannot trust my mind then then all arguments are moot; including naturalism.” And I completely agree. This is why I do not regard the Argument from Reason as persuasive. It undercuts itself for the same reason it purports to undercut naturalism. You could make the same argument against any belief system, including theism, as I point out, so it just amounts to selective philosophical skepticism.

      “You can believe this Leon, and I can’t argue against it but it is, in Vulcan, “completely illogical”.” I agree that it is completely illogical. I only meant it as a hypothetical parallel to the “Finest Argument Against Atheism.” If this argument rules out naturalism, it does so only by ruling out knowledge claims under any philosophical system. But I don’t think it even rules out naturalism.

      “If your beef with the existence of God is that he allows pain, that is a valid point and debatable. But that we can’t believe in him because he could possibly be deceptive is not.” I’m sorry if I didn’t make myself clear, but this was not the point I was making. I was not directly talking about the problem of pain here. I was merely using it to illustrate that there is plenty of conceptual space within Christian theology to permit a God who would deceive us in a wholesale fashion.

      For example, perhaps we are all in heaven right now, and God has supernaturally created a perfect simulation of earthly life, with our memories temporarily erased, so as to build our moral character and empathy. This is very similar to the “soul-making” theodicy of John Hicks. And likewise, at some point, nearly every Christian will fall back on the belief that God must have some unknown reason for allowing the amount and kinds of evil we see in the world, and that we simply can’t understand it because His ways are higher than ours. This same reasoning could be used to explain why God is deceiving us. But I definitely was not arguing that we should not believe in God because God might possibly be deceptive. I agree with you that that is nonsense.

      I would, however, challenge strongly on this point you make about science: “Therefore Scientists can never tell us how old the Earth is (etc.)…Historical Science does not exist as it violates the first tenant of Science which is Observation.” I don’t think you realize how much sound knowledge this principle undermines. For example, I doubt you would say that forensic science is completely illegitimate as a discipline simply because it deals with things that happened in the (recent) past. If so, then there are a lot of criminals who have been put behind bars on “flimsy” historical science. But why should the scientific method be any more illegitimate if it deals with things in the distant past? Taken to its logical conclusion, your reasoning would show that we can only have knowledge of the present moment, because by the time the results have been published in a scientific journal, it’s history, not observation.

      (Aside: When it comes to the age of the earth, it IS based on things we currently observe. We can measure the decay rate of different radioactive molecules based on ticks on a Geiger counter, an instrument which has proven itself reliable in many other contexts, and thus calculate for a wide variety of molecules its “half-life,” the time it takes for half of the original parent material to decay from one kind of molecule to another. These range from a few seconds to many months to several million years. We can (and have) subjected those materials to a variety of temperatures, pressures, radiation levels, and so forth, and observed that those half-lives remain constant (which is unfortunate because it would solve our nuclear waste problem if we could speed it up). We can observe that certain kinds of molecules do not appear under any conditions on Earth, to the best of our knowledge, except following the decay of that particular radioactive molecule. And then we can observe that there is a certain amount of that molecule type in a rock or organism, and a certain amount of its radioactive parent. If 50% of the parent molecule remains, it’s undergone one half life; if 25%, then two half-lives, and so on.

      And even though the events happened in the past, determining the age of the rocks produces real predictions about the future, as well: If you find a rock formation that dates to younger than 65 million years old, you can confidently predict that you will find no dinosaur bones in it. So far that prediction, and millions like it, have proven true every single time. Knowing the ages of rocks has also been crucial in finding new oil deposits. So these people are on to something. And different dating methods have been used to cross-check the accuracy of others, and they converge on the same approximate ages. When it comes down to it, determining the age of the earth is, in principle, no different than determining the approximate time of death of an individual based on the level of decomposition, which is often crucial in solving murder cases. End of aside.)

      Science is not simply observation. It’s making inferences from that observation. And you’re completely right that those inferences can be faulty if the scientist’s background assumptions are wrong. I, for one, think that more scientists need to study philosophy and logic for that very reason. But that doesn’t mean that all science is just a ink blot test, completely open to interpretation based on your biases. Observation constrains what inferences we can make.

      Maybe there really is something “outside the aquarium” as you put it. I won’t rule that out a priori. Science, at its best, is not about dogmatizing, but about being willing to accept the truth no matter where it may lie. But until some kind of evidence points convincingly to a realm beyond the natural world, it might as well be the invisible cat on my chair.

      I hope this clarifies what I was trying to achieve with my response to Eric. Thanks again for your insights!

  10. To Leon above:

    “Why do you think that the most consistent, honest naturalism is the one which has such a fatal flaw? Other brands of naturalism, such as soft deterministic ones, or compatibilist ones, are not affected by the Argument from Reason. What redeeming qualities does hard determinism have that makes you prefer it over other varieties of naturalism?”

    I don’t see it like this at all. Even the so-called “soft deterministic” ones, once all the linguistic evasions are dealt with, reveal themselves to be every bit deterministic as straight forward determinism is. Take for example the idea that a person’s motives are at the mercy of the human agent, thus freewill is accepted to a degree, i.e., people are free to direct their internal workings but not their external workings (they can want a beer and decide to go get one, whereas they may not want cancer but genetics may have it in for them, etc). But allowing a redefining of freewill so that internal motives are acknowledged on some level as “free” only places the problem one step back in the discussion. The obvious question then is: is a person’s internal workings something outside of the material universe? Does whatever “emergent property” from a critical mass of neurological structuring in one’s brain (as you noted) really lie outside the natural system? If all events are determined by a chain of cause and effect beginning from the beginning of time then one’s “emergent consciousness” is not somehow made exempt simply because it received a fancy new name. And this is precisely what I see happening – sometimes people feel that they’ve conquered a conundrum because they’ve given it a new title, a new classification category, but one is wise to hear Wittgenstein on this: “Philosophy is a battle against the bewitchment of our intelligence by means of language.” If the emergent property of consciousness is something exempt from the total system then this exemption possess the true fatal flaw of soft determinism, for it cannot answer to what “other” system this property belongs. In short, natural determinism involves human beings in ALL their workings otherwise human beings fall outside material nature is a significant way and become something other than mere nature (this is Christianity’s claim!).

    I’d like to address your quote from Ayn Rand, as it seems to be to make my case:

    “Can one prove that man’s consciousness does not function automatically, as determinists would say? If man’s consciousness WERE automatic, if it did react deterministically to outer or inner forces acting upon it, then, by definition, a man would have no choice in regard to his mental content; he would accept whatever he had to accept, whatever ideas the determining forces engendered in him… Volition is inescapable. Even its enemies have to accept and use it in the process of any attempt to deny it.” (Objectivism: The Philosophy of Ayn Rand, pp. 69-70).

    I couldn’t agree more – “Volition is inescapable.” But the soft determinist must, if pressed, admit that volition does not escape the total system. What caused the volition? The answer must reflect natural mechanistic workings – albeit, within the thinking subject – otherwise the cause or source of the volition is exempt from being considered a “natural event.”

    ‘Actually, this has been an issue for Latin based Christianity. It has not been a hurdle for the modern Eastern Church, nor for the ancient Church, which history bears out.’ “I find Eastern Orthodoxy fascinating, but I do not understand what you mean here. Could you please clarify why the Eastern church has not had a problem reconciling free will with divine omniscience?”

    Without getting into a theological dissertation of sorts, if I can simplify it (perhaps to a fault) the East has always been content to see human free-agency as a gift from God. God willed that we would also be capable of making decisions based on our love or neglect (or hatred) of Him. Many parables in Scripture reflect this idea. Take for example the parables of the vineyard where the master puts his vinedressers in charge of His estate to tend it and keep it. Or the parable of the Talents where the master gave his servants a number of talents (money) to do business until he returned. In these examples the servants are in full control of their decision making process of whether to serve their master or despise him. Read the parables in full and you will see this is definitely the attitude the master takes toward his servants. Thus, God is all-knowing and all-powerful but has purposed to allow mankind to be rulers of the earth. The primordial “fall” changed mankinds relationship to God. They fell from a position of “priests of creation,” so to speak, to something slightly above the mere animal kingdom.

    I know there was more to your post but this is all I have time for at the moment. Please let me know if and where I’ve misunderstood you. Thanks again for the wonderful discussion.

    Cheers!

  11. You guys are withholding a very difficult conversation to keep up with, since I’m only in the 8th grade, but I understand most of it and I agree with pretty much all of the points made. But there is one that I have not heard and I would like to make it.
    One of the main reasons I disagree with theism is because there are so many religions and religous practices. Most of them contradict each other, believe in seperate gods, different after life’s, different books and so on. My question is if one religion is supposed to the one, right, and only way, then how come there are people within every religion that believe their religion is the right way. How does one within a religion nor question the high possibility that they are amongst the wrong faith. Snd why would a god allow so many false faiths to drive people in the wrong direction.

    • Thanks for the post, Elaine. Congrats on diving into this stuff at such a young age! In answer to your first question I would say that the existence of many different religions is no reason to disbelieve “theism” (not that all religions are theistic, but you get the point). It seems that it would actually be evidence in the opposite direction, evidence that something is going on here and humanity as a whole senses something much bigger and more profound about existence than any form of atheism is willing to give it.

      Second it is not surprising that there would be many divergent paths. Think of a beautiful painting, or song, or coin, or whatever. Anytime something original and beautiful hits the scene, a thousand copycats pop up everywhere. If there is one true path then it seems impossible that there would not be copycat paths, mimicking the real one, or at least trying to obtain what the real one has to offer without going through the trouble of attaining it with effort.

      And “why would God allow so many false faiths”? For the same reason that death and sin exist in a world created by a good God. He gave the progression of the human race in the hands of humans to either follow him or follow their own ways. Most follow their own ways and the result is total confusion.

    • Either you are not really in the 8th Grade or you are so far ahead of where I was in the at that age based on the fact that this is an excellent question

      One of the problems that modern Evangelical Christians have is the way they portray God. If one listens to Christians explaining how God feels about man it can be summed up as such,

      “Man kind of slipped and made this mistake and it really wasn’t that bad and God, being the old grandfatherly man that he is said, ‘Whoopsee, did we make a boo boo?’ and sent his son, Jesus, down to help us clean up the mess.’ He then said that along as we said, ‘Sorry,” then we would go to Heaven.

      Which of course is not what the Bible says.

      God created the world and then we decided that we would live our lives on our own way, FU God we don’t need you or want you in our lives. God did not see this as a ‘mistake’. God sees this as EVIL. And the punishment for this evil wasn’t a slap on the wrist. It was death. Physical and spiritual. He sent his son Jesus to pay for the eternal punishment we should have received. And if you repent (turn the other direction) from your sins and believe in the work of Jesus on the cross then you will have eternal life.

      So the reason there are so many different religions is we are back to the original sin. All of those religions are man attempting to set the conditions of the relationship with God. We are back to man telling God how things are going to be. Christianity is God telling man how things are going to be.

      Hope that helps.

    • “So the reason there are so many different religions is we are back to the original sin. All of those religions are man attempting to set the conditions of the relationship with God. We are back to man telling God how things are going to be. Christianity is God telling man how things are going to be.”

      Excellent quote, here. I’ve heard said in similar way before as such; “‘Religion’ is man’s way of trying to reach God. The Christian Gospel of Jesus Christ is us recognizing that we can NEVER get to Him in our sinful state, thus, God Himself (in the person of Jesus Christ) came down to us and made of way for us to be reconciled with the Father.”

      In essence, God has made the way, as you alluded to. All we have to do is follow the path that is laid out for us. The Bible illustrates the path and where and how it leads back to God. But it also tells us that such a path was not made possible by us, but by God Himself. That’s the difference between ‘religion’ and Christianity.

  12. Reblogged this on Marius Cruceru and commented:
    Adrian Iosif: “Una din premizele argumentului este revoluta: ” Since thoughts are events, all of our thoughts should be explainable in mechanistic terms, i.e. as the results of previous events in a cause and effect relation ultimately…” Este un fel de “demon al lui Laplace” aplicat in fiziologia creierului. Stiinta moderna, incepand cu teoria cuantica si sfarsind cu teoria complexitatii, pune cu totul altfel problema. Nu mai putem vorbi de determinist mecanicist si de cauzalitate in sens clasic. Ceea ce se intampla nu este numai rezultatul intercatiunii atomilor. Chiar si conceptul de materia este astazi definit altfel. Multe fenomene “mistice” pentru nivelul de intelegere al secolului XIX au astazi explicatii banale. Cartile lui Penrose despre creierul uman dezvolta subiectul mult mai bine decat o pot face eu in acest moment (utilizand un creier suprasolicitat de mai multe zile in care a trebuit sa functioneze in regim optim 16-18 ore/zi ). Arguemntul expus este un bun exercitiu, dar demonstreaza limitele apologeticii stiintifice: stiinta “sapa” continuu si, unoeri, gaseste explicatii rationale care par a strivi “corola de minuni a lumii”. Desigur, nu e asa, frumusetea creatiei nu scade, ci chiar sporeste pentru ca rationalul este parte a acestei frumuseti. Nu ucizi taina dandu-i o explicatie naturalist rationala, ci doar descoperi ca exista o ordine si un sens care fac parte din poemul celor vazute.”

    • In case anyone else is interested in it, I will leave here the translation from Romanian:

      “One of the premises of the argument is revolution: ” Since thoughts are events, all of our thoughts should be explainable in mechanistic terms, i.e. as the results of previous events in a cause and effect relation ultimately…” It is a sort of “Laplace’s demon as applied to the physiology of the brain. Modern science, from quantum theory to complexity theory, takes on the issue quite differently. We can no longer speak of mechanistic determinism and classical causality. That which happens is no longer the result of the interaction between atoms. The very concept of matter is defined differently today. Many phenomena deemed “mystical” in the 19th century as per the level of understanding of that day and age have commonplace explanations today. Penrose’s books on the human brain expands on the topic much better than I can at the moment (using an overexerted brain than needs to function optimally for 16-18 hours/day). The argument being presented is a good exercise, but it demonstrates the limits of scientific apologetics: science keeps “digging” and, sometimes, comes by rational explanations that seem to crush “the wreath of wonders of the world” [1]. Of course, it is not so; the beauty of creation does not diminish, but is actually enhanced because the rational aspect is part and parcel of this beauty. You do not extinguish a mystery[1] by giving it a rational, naturalistic explanation, but find that there is an order and meaning within the poem of that which we see.”

      [1] these two are metaphors taken from a poem by a highly regarded Romanian poet and philosopher, Lucian Blaga. The poem is called “I do not break the wreath of wonders of the world” and this is the best translation I can do as Romanian poetry is particularly difficult to render in English while preserving the original meaning, style and impact. In a nutshell, the poem is a lyrical confession that deals mainly with poetic art and discovering the awe-inspiring mysteries of the Universe through poetic contemplation as opposed to the tools of reason which “breaks this wreath of wonders”.

  13. Eric, I appreciate the articulated argument; however, it is unsound. Let me begin by using the exact same logic to formulate a different argument:
    P1: Kinetic energy is produced from motion.
    P2: Inert objects have no motion.
    P3: Therefore, inert objects have no kinetic energy.
    P4: If an object is inert it cannot cause motion.
    P5: Cars are constructed from inert objects.
    C: Therefore, cars cannot cause motion.

    I’m sure the problem with this argument is plainly obvious to you. This is exactly the same problem inherent in the claim that natural determinism disqualifies our thoughts from being our own and not being a result of rational inference. The natural forces create the conditions for, and the causally linked materials which form, the mechanism which is capable of producing thoughts from natural inference. Much like metal (and the rest of the materials used in constructing a car) is inherently inert, but once constructed into a specific mechanism (the parts of a car) can produce motion.

    The quote from Professor Haldane derives from a long ago dismissed Neo-Platonic Hermeticism which is, comically, self-refuting. “I have no reason to suppose that my beliefs are true … and hence I have no reason to suppose that my brain to be composed of atoms.” And by extension he would have no reason to have the belief that he has no reason to suppose his brain is composed of atoms, thus he would have to re-establish that belief and reasoning right before he rejects it again, and re-establishes it once more, and so on.

    Lastly, Victor Reppert’s argument has several problems, but most noteworthy is the fallacy that follows that same error in reasoning that is pervasive in this entire line of thought. Namely, that one cannot trust an internal source for reasoning because it originates from an non-cognitive external source. This could best be described by the “wetness of water” analogy. How can water be wet if Hydrogen is not wet and Oxygen is not wet? The lack of “wetness” of the molecules individually does not prevent the effect (and natural state) of the combination.

    In simple terms, thought (and reason) is an emergent property of the complex biological system we call a brain (which is in turn an emergent property of other natural systems).

    • Pavlos, the example you gave above is not convincing:
      It is indeed true that if P5 is correct C follows (Newtons first law of motion). However P5 is incorrect. Fuel is not an “inert object” but an essential element of a functional car. You cannot get a car in motion without a source of energy that is in its parts already, in this case the chemical potential of the fuel.
      If one does not understand how a car functions it does not help to call it’s motion an “emergent property”. However, if one observes a moving car one could postulate that this motion cannot be caused by a system of inert objects alone and would be motivated to look for another component that could cause this and hopefully find the fuel tank.

  14. Pavlos, thanks for you post.

    Your comparison is not a similar argument. I think sometimes people mistakenly suppose that because two arguments are made using the same method (i.e. a syllogism) that they are eo ipso the same argument. Your example demonstrates this faulty reasoning.

    If your argument was to truly mirror Repperts it would read something like this:

    P1: The production of Kinetic energy can be fully explained in terms of inert objects in motion.
    P2: If anti-kineticism is true, then the production of kinetic energy can be fully explained in terms of inert objects at rest.
    P3: Therefore, if anti-kineticism is true, inert objects at rest can produce kinetic energy.
    P4: If any thesis entails the conclusion that kinetic energy is produced by inert objects at rest, then it sould be rejected and its denial accepted.
    P5: Therefore anti-kineticism should be rejected and its denal accepted.

    To your second point, of course Haldane’s argument is ridiculous and self-defeating. I believe that was his point – to make fun of the ridiculous using the ridiculous.

    To your third, you use another faulty analogy which does not address the overall point. By observing the subjective accidental quality of wetness is not at all the same as observing rational inference. If your analogy were to hold it would imply that rational inference is an emergent quality (or “accident”) not inherent in the substance of a human’s core. In other words, humans could take it or leave it. But its of course much more than that. It is the very quality which is definitionally “human” in every way outside mere biological observations. But this is only one of the flaws in your reasoning on this point. It does not address the main thesis: If materialism is true then all events can be accounted for – fully explained – in terms of the non-rational, mechanistic, network of cause-and-effect. Thoughts are events. If thoughts are merely the end result of natural/mindless clashing of atoms, or whathaveyou, then humans are anything but free agents equipped with rational inference. Rational inference is by definition free from exterior influences save logical deduction alone. If it is in anyway the result of nature acting upon one’s thinking (as in the case of a brain injury, alcohol, emotions, etc.) then it is not “rationally” inferred. But according to a strict materialism this is how all thoughts occur regardless if one puts it back a few steps by introduction the explanation of “emergent” qualities. Unless, of course, one postulates that emergent qualities are somehow outside the causal, divinity-less, universe.

    • You are correct that two arguments of the same form are not analogous because of that form. However, my comparison was not to match Victor Reppert’s syllogism, but your argument. Indeed, if I were to offer similar syllogisms I would have done so accurately. And it’s an irrelevant fact that I shall not pursue, but I will mention that your “corrected” syllogism is also inaccurate.

      The “ridiculousness” of Haldane’s argument, unfortunately, is misdirected as it is not in the least absurd, or self-refuting, when properly understood that biology is, in fact, the mechanism which produces such events. It is only when one discounts such “truths” dismissively that the absurdity arises; especially when the hidden alternative is assumed in its place.

      Regarding your comment on the “wetness of water” analogy as an emergent property (as it relates to thoughts), without any disrespect intended, you leave me wondering one of two things (or both): 1) You don’t understand what an emergent property is; and/or 2) You have no kept up with recent neurological studies and their conclusions.

      An emergent property is not an “accident” which can be willingly accepted or dismissed. We can’t take it or leave it anymore than we can choose to feel liquid as wet or dry. Similarly with fire and heat. You can’t say that heat (the transfer of molecules with a high kinetic energy to your receptors) is an “accident” of fire. It is a necessary condition of the complex system (an emergent phenomenon).

      You are also skipping ahead to rational inference to illustrate your point, but this is precisely where the fallacy in your argument arises. Biology “produces” consciousness (or more accurately, consciousness is the expression of the physical state of the brain), within which thoughts exist from which rational inference can be made. By skipping ahead to rational inference you are constructing an argument akin to “Nobody has been to the moon, because it’s impossible for horses to fly.” (forgive the exaggerated absurdity of the analogy). My point is that much has happened between the “horses” and the “landing on the moon,” and we can’t disregard it and hope to construct a sound argument from this position.

      Furthermore, your definition of “rational inference” is either accidentally mistaken, or purposeful for an etymological fallacy. Here is the correct definition:
      in·fer·ence (ĭn′fər-əns)
      n.
      1.
      a. The act or process of deriving logical conclusions from premises known or assumed to be true.
      b. The act of reasoning from factual knowledge or evidence.

      Your definition was “Rational inference is by definition free from exterior influences save logical deduction alone.”

      Premises are known or assumed to be true based on the necessary and sufficient conditions for knowledge, namely a “justifiable true belief.” All such properties derive from external sources. If not, then they cannot be claimed to be justifiable, or true.

      Of course “reasoning from factual knowledge or evidence” is self-evident to be from an external source.

      Our interaction with such external sources is what causes us to connect aspects of our consciousness so as to create a rational inference. Our consciousness, again, is an expression (representation) of the physical state of the brain (that is to say the brain precedes the mind), therefore rational inference can be, and is, produced from non-rational sources which are completely natural and explained by strict materialism.

      Towards the end of your last paragraph you give the impression that you are going down the road of either a slippery slope fallacy or a personal incredulity fallacy. You didn’t actually conclude your thought, or if it were a conclusion it did not follow from any given premises. I read your misunderstood criticism of Sam Harris so I already know where you stand on this position (and I’m not going defend Sam here), but if I were to try and distinguish your conclusions here it would be that you can’t accept strict materialism because you don’t want to accept the inevitable consequence that we are, indeed, not free agents operating independently from our system.

      Essentially it seems to me that you are viewing the very nature of consciousness from an archaic position that places it in an ethereal realm, almost like it is a invisible bubble floating around, or inside our heads. If you understand and accept the materialistic causal chain leading up to a specific event, then you are left with no footing to simply disregard a consequence of that chain without first showing a problem with a previous link. For example (an analogy), a seed grows into a sprout, the sprout grows into a tree, the tree produces fruit, a man picks the fruit and takes a bite. The bite travels down his esophagus, enters the stomach, a miracle happens, the man now has more energy. Notice, of course, that no miracle actually takes place. But that is essentially what your argument seems to be progressing towards. You accept all the materialistic events leading up to a specific event and then you choose to sever the connection in favor of something “other.” Meanwhile, the only thing preventing this argument of yours from becoming an argument from ignorance is that you have not yet produced your alternative. You’ve merely tried to refute the claim which is based on observable evidence, and have not offered anything in replacement. Perhaps this is because you understand that you cannot do so without being guilty of an argument from ignorance.

    • Hello again, Pavlos.

      In response to the first part of your reply I can only say that it did not seem to me that your mirrored argument was comparable in the least, which is why I offered an alternative. It’s not irrelevant, but if you’d prefer to let it go that’s fine.

      Next you said that “The “ridiculousness” of Haldane’s argument, unfortunately, is misdirected as it is not in the least absurd, or self-refuting, when properly understood that biology is, in fact, the mechanism which produces such events.” I was merely agreeing with your earlier contradictory comment that “The quote from Professor Haldane derives from a long ago dismissed Neo-Platonic Hermeticism which is, comically, self-refuting.” It is indeed self-refuting, but purposely so in order to show the comical self-refuting posit that knowledge is based on mere neurological functions, such as clashing of atoms. His point is that if it is mere biology that is making him think what he thinks, and he has no free-agency, then it is not “he” that is thinking but rather reflecting whatever it is that the mechanical structures in the total network of the universe would determine him to think. Thus, why trust it? It’s not based on any “truth” necessarily but rather on clashing atoms. On these same lines I’m curious if you are married or dating and if you love your mate. If so, can you accept that “your” feelings are mechanically stimulated results based not on free agency but based on a non-rational causal chain of reactions traceable back to the big bang. Your love is no more meaningful or interesting then fruit falling from a tree (if your premise is correct). More than that, you can do nothing but “love” her. How is that love?

      Regarding your point about wetness not being an accidental quality, I concede. My overall point was that “wetness” is subjective according to our own senses and is not necessarily the substance of water. But the point can be argued either way depending on your philosophical bend.

      Moving on, you said that I was skipping ahead to rational inference and thus leaving much intellectual territory ignored. In essence your argument is that I accept materialism until it reaches the production of thought, or more specifically rational inference. You countered my definition with something else, which if you read it all again will find no serious deviation from the definition you provided, minus that you misunderstood what I meant by “exterior influences” (i.e. I did not mean exterior evidences from which reason has to work with, but rather encroachments on the reasoning process by nature, such as a brain injury, a drug, a toothache, etc.). To counter your misunderstanding I would simply ask if you believe that someone was engaged in rational inference if their thinking was colored with alcohol intoxication? If your answer is anything but “no” then we have entirely different understandings of what constitutes rational inference. I would say it was the Rum talking, not the person.

      In a like manner, if a person is not a free agent and able to examine nature from a distance then he/she is not making “their” own observations but rather are making observations according to what the total system would have them make. Maybe this will help: by “distance” I mean a person must have the ability to transcend nature in order to study it. In the same way that I must transcend a fish tank in order to study the nature within a fish tank otherwise, if my observations come from within the fish tank and the fish tank wholly determines what I have to say about the fish tank, then my observations are…what should we say?… ridiculous? At a minimum you must concede that they’re not serious. But, I digress.

      It is not that I trust materialism right up to the point of thought, but rather I do not mistaken the physical process for what is produced. In other words, I do not mistaken, for instance, the feeling of love for mere neuro-chemical reactions. Love is a much more existential and profound phenomenon than anything an MRI can scan. But that does not dismiss the fact that “love” has neuro pathways. Hope that makes sense. I trust that my biological processes are manifesting thought, but I understand thought to be much more than a mere emergent property. It is at the core of what it is to be human. It is the ding an sich of humanity. It is that part of us which the Bible describes as being made in the likeness and image of God. Human reason has invaded nature. It is not wholly outside of nature, but it is not merely a product of nature.

      There’s much more I’d like to respond to but simply don’t have the time. Let me make a final observation. When it comes to emergent properties I get the feeling that you and likeminded materialists tend to treat emergent properties as if they somehow escape from the grasp of materialism – as if they are not subject to the same causal chain which determines everything else (in your philosophy that is). Simply stating that rational inference and reason is an emergent quality does nothing to solve the problem of materialism. It seems to be more of a smoke and mirrors game. Either thought is dependent on the non-rational/non-thinking/non-conscious mechanical network of the universe, and thus not an experience of free-agents but rather the determined output of humanoid robots, or materialism has found a convenient halfway house to build its sky castle.

      And, I do not believe I misunderstand Sam Harris one bit. He’s riding the philosophic short-bus, as I said in the article. He’s one of the most unpolished and short-sighted philosophical thinkers I’ve ever seen behind a podium. But that’s another discussion.

      This has been fun.

      Cheers.

  15. I wrote a rather long reply that attempted to cover most of the points, but then I realized much of it has become about peripheral issues and we are in danger of not seeing the forest for the trees. I can still offer that reply later on, if you wish (I have it saved just in case) and want to resolve those issues, but right now I think it would better if we focused our attention on the core of the topic because it seems to me you just conceded the point.

    You said “It is not that I trust materialism right up to the point of thought, but rather I do not mistaken the physical process for what is produced,” and “I trust that my biological processes are manifesting thought, but . . .” Consider carefully what you wrote there. Materialism, the theory you have been arguing against even though you’ve given the article a title that suggests this is an argument against atheism, holds the position that everything originates from physical process and their interactions. You just agreed to that point. The fact that you go on to add “but” and explain that there is something more to it is irrelevant because materialism doesn’t say your feelings of love or your thoughts can’t be special or meaningful to you. In other words, to argue against materialism you need to show exactly where the materialistic causal chain is broken and something supernatural is introduced. But if you agree that from the observed beginning of the observable universe everything has been a result of a materialistic causal chain then you are in agreement with materialism.

    • Haha, well played, Pavlos.

      It’s not a concession to materialism, the point is that materialism fails to account for the truly existential mysteries. The causal chain of events in nature are readily observable and undisputed. What is disputed is the human’s place in the mix.

      My Orthodox Christian-informed anthropology states that man has a free-will (observable in everything he does), created in the image and likeness of God. As such his reasoning abilities transcend mere physical cause and effect in the sense that he is not wholly subordinate to the clashing of chemicals in his brain, but rather “directs” the clashing chemicals in his brain—the brain follows his spiritual reasoning power, not the other way around. That is not to say that he is always in control of every thought, i.e. in cases where nature has encroached on the normal functioning of his brain (again, as in the case of a physical deformation, intoxication, or if he is given to conflicting impulses like anxiety, which he produces but does so unwittingly. These things are of course “natural” but cause the brain to function in ways it is not designed to function). In short, man is in possession of his reasoning; nature does not possess his reasoning. Nature is used by man; man uses the regular natural processes to accomplish his free-will.

      So, what do I do with the fact that everything follows a causal chain, the brain not excluded? Exactly what I said above. The brain reacts to stimuli in its environment and is also conditioned by certain hereditary traits. However, the human being chooses what to do with both. The stimulus offered to him passes through his reasoning center and he decides what he will “conclude” about the matter, and what he will “do” in response. The brains movements are a meeting place of soul and body. There is no contradiction in saying, “I trust that my biological processes are manifesting thought.” Manifesting thought is the observable result, not the “process” of arriving at the thought. If you pick apart certain bits of my writing and form a complete rebuttal you will miss my point (I am not writing as if to present a water tight expose for an academic publication).

      In practical terms, if materialism were true in relation to man’s reasoning, we should be able to predict exactly what he will do when mapping his neuro-functions. But can we? Because a particular pathway interacts with another pathway, is there a guaranteed outcome reflected in the person’s thought and actions? And if so, are those the only pathways possible? Could the brain have taken, say, 1 of 3 different pathways (or 3,000) based on a single stimulus, with no straight forward scientific explanation as to why? I do not study neuro-biology, but I do study psychology and in psychology there are no guarantees (those who believe so are strict-behaviorists, but they are so on account of philosophy, not pure science). A single stimulus can result in innumerable reactions and emotions. On an off note, I fear that many scientists recoil so violently at the prospect of “mystery” that they can’t sleep at night until they have developed a satisfactory theory which they believe answers everything. “Biologism” is the most whimsical of all philosophies: the belief that biology answers all, and is the reigning ideology of many in the field.

      It should be said that as an Orthodox Christian I do not have a strict separation between the physical and the divine. Christ’s incarnation is the revelation that God divinizes the physical. The spirit and flesh are jointed in a paradoxical way in which they are fully united, yet not confused.

      I admire that you hold the most consistent version of an atheistic philosophy available. You are willing to accept that literally everything you’ve ever thought, felt, or imagined are the products physical determinism. You may attempt to make meaning out of your emotions, but at bottom they are chemical reactions with no relation to “truth.” Truth itself, in your philosophical view, is—and MUST be—a mythical concept. Truth is simply whatever “happens to be” at the moment on account of the mechanical workings of the total system. The same applies to every other deeply existential human concern including good, evil, love, hate, right, wrong, etc. If one is consistent with materialism all of these are simply reduced to “what is.” Anything more is pure fantasy. Whatever value you attach to them is arbitrary and ultimately meaningless(not to mention that the “you” who supposedly assigns meaning is actually chemical reactions “assigning” meaning to other chemical reactions; sort of comedy of forced delusion).

      Not so in reality since we are the product of a loving and personal God who exists ontologically as Trinity.

      Keep in mind that this article is the issue that is “for me” the finest argument against atheism. However, I’m perfectly aware that one can be at peace with this philosophy, as you seem to be.

      Cheers.

  16. As a person who finds immense joy in philosophical concepts, theories, and argumentation I read your latest reply with the same eagerness a starving man enjoys walking into a buffet. However, I must restrain myself once again because I don’t want to lose focus of the specific topic at hand.

    Now, from the very beginning you said “It should be noted that this is not an argument “for” Christianity, but rather an argument against atheism, specifically that of materialism and/or naturalism. An argument for Christianity would follow a very different set of rules.”

    That is completely true and accurate. There are two ways for an argument to fail: 1) It fails on its own (usually because it’s self-contradictory, or illogical), or 2) Because the opposite is true.

    For example: If I declare that I am a rock, you don’t have to prove the opposite is true (I’m not a rock) to show the argument fails. It fails by virtue of the fact that it’s inherently illogical because a rock simply cannot declare itself as a rock (there is no self-awareness, no ability to communicate, etc.). If, however, I declare that I am a woman there is nothing inherently illogical or contradictory about that. To defeat the argument you would have to prove the opposite is true (that I am a man). I can’t be a woman if I am a man, so the argument fails because the opposite is true.

    Going back to the quote from the beginning of the article, you made the case (and were very clear about it) that this is not an argument in favor of theism, and that such an argument would look very different. So you declared clearly that you don’t need to prove the opposite to defeat the atheistic/materialistic view, instead that view fails on its own. However, we have come to an agreement that it does not. The fact that you are offering theistic arguments and beliefs is an attempt to prove the opposite is true (and it might be) because the initial argument does not fail on its own. So, it’s irrelevant what theism has to say unless you want to defeat atheism/materialism by proving its opposite (theism). In the mean time, however, the offered argument is agreeably defeated.

    • Haha, for two people who are on completely opposite sides of the fence, we are very kindred in our love for discussion. Thank you for your input to this post, I’ve enjoyed it immensely.

      As you can probably tell, I have avoided giving my theistic view on the matter up till our last back-and-forth. I only revealed my own because you said I had up to this point offered no alternative, thus out came my Orthodox Christian view. I fully acknowledge that insofar as the materialist argues his case with the facts he chooses to argue with, his case is solid, if only for his own satisfaction, as is the case for the theist making his counterclaim. If you want my most concise version of why I believe the materialists philosophy fails, it has to do with the fact that he/she is unable to live in congruence with that philosophy. Hence, by their own life-witness they deny their espoused belief.

      It could be that everything in existence is marching to the drum of natural determinism. God might be the ultimate fantasy (a fantasy that the total system came up with for some reason). Those who believe so do so with the same activity properly called “faith” which animates the theist own claims. 🙂 I believe in the old adage that: the facts don’t settle the argument, the argument settles the facts. We all work with the same collection of “facts.” What those facts “mean” is the process of applying one’s faith in one’s own integrity of properly interpreting the facts.

      So, in short, my argument against atheism is that it is unlivable for actual human beings. It’s perfect for, what Kierkegaard called, the phantom of pure reason, but not for the living individual. The argument from reason is one of the shining examples (for me anyway). If I’m limited to one of your two options for how to defeat an argument, I suppose this would fall into the category of it failing its own existential test. My argument for Christianity is based on multiple levels of considerations: theological, philosophical, and yes even scientific. My argument for Christianity is not, properly speaking, my anthropological argument.

  17. I’m not sure I understand some points in that reply. I thought we just concluded that the argument does not fail on its own, but instead requires its opposite to be true in order for it to fail. But then you said it fails its own “existential test.” I suppose the problem could be that there are a few ways I could interpret what that means.

    One of those ways leads me to my second point of confusion. You said “If you want my most concise version of why I believe the materialists philosophy fails, it has to do with the fact that he/she is unable to live in congruence with that philosophy. Hence, by their own life-witness they deny their espoused belief.” I’m wondering if you mean this in the same way you asked me about love. In other words, that as a materialist I should see love for my wife and children as something purely physical and having no profound meaning beyond that, but that in truth I don’t see it that way.

    That’s one of the questions I didn’t answer because I wanted to focus on the topic, but now I probably could answer it. I view it in the same way I view the connection of the brain and the mind. In other words, the unromantic, clinical, sterile “truth” is that love is nothing beyond the physical secretion of Oxytocin. If you disrupt that secretion the feeling of love is gone. If you cause that secretion in connection with a random stranger you can cause the feeling of love to appear for that stranger. This, however, does not prevent people to “wax poetic” about love because our brains are capable of creating other cognitive events that can combine to produce beautiful and deeply moving sonnets, paintings, novels, songs, etc.

    So when I say that I would die for my children, I say it honestly while at the same time I acknowledge that if someone disrupted the secretion of Oxytocin in my brain they would not elicit such feelings within me.

    So I’m not sure why atheism is unlivable. It’s like painting by numbers; beneath the awe inspiring beauty of a work of art that causes emotional turmoil within the audience upon seeing it, can be nothing more than a series of lines and numbers with instructions of what paint to apply where, and how.

    Another analogy could be a movie. I took one class in Movie Production back in college and for that semester I felt like all movies had been forever ruined for me. The reason being that I suddenly became aware of the lighting effects, the camera techniques, the directorial instructions, and all such highly technical aspects of movie production. I learned about certain rules of cinematography and names of special techniques . . . my point is that during that semester I’d watch a movie and instead of being immersed in the story I would be analyzing it and distinguishing a rack focus, a bridging shot, cross-cutting, dollying, etc. But that quickly faded and the magic of a well-made movie returned. I am now aware of the technical aspects of movie production, but it does not interfere with my appreciation of the final product.

    I suppose you could say that I can live like an atheist without discomfort because I enjoy the process of pulling back the curtain to reveal the man behind the pretense of the wizard. And if that is an ugly truth then it’s no different than finding out Santa isn’t real. Yes, it makes you sad at first because you feel like you have lost something, but then you realize that it’s not a magic man in a suit with a list, but it’s the people who love you who have taken the time to find the perfect gifts for you because they want you to be happy. And isn’t that a more beautiful reality than the lie we are told?

    Going back to love, you said that if it’s simply a biochemical reaction then what choice do you have? The same could be asked of other views of love. If it’s a spiritual connection what choice do you have? You are connected to them spiritually so you were destined to only ever love this one specific person and it’s out of your control. Then again, if it’s a matter of choice then what kind of love is it really? If you can pick whom to love in the same way you pick what clothes to wear, what book to read, or which route to take to work, then it’s not really special at all, perhaps not even truly love.

    My point is that you can make either option seem as ugly or as beautiful as you desire. I once read a comment from a man who was scheduled to have his leg amputated due to diabetic complications. In the comment he was making the case that it’s a good thing because the result of the surgery would be that he would instantly double the amount of socks he owns, he would have access to the best parking spots, he would enjoy priority boarding at the airport, and he wouldn’t have to wait in line for rides at the amusement park.

    You wrote that you agree with the old adage that “the facts don’t settle the argument, the argument settles the facts.” I agree to an extent. Much of reality is a matter of perception. If you show me a pencil I will tell you it’s a writing instrument. If you show it to someone from a distant tribe with little to no contact with outside civilization they might tell you it’s a weapon or an eating instrument. Their perception changes the use of the object, but not the object itself. No matter what the argument for the use is, it remains true that it’s made of wood and led. No amount of argumentation will change such facts. Similarly with cows and Hindus. To me a cow is an animal that can be used for my own consumption (to be blunt about it), but to a Hindu it’s a sacred animal that should be allowed to roam freely even in their houses. Our perception changes the “use” of the cow, but not the fact that it’s an animal, that it has flesh and blood, etc.

    I think I may have gone on this tangent for too long. I will say, however, that it’s always good to find another person who values discussion. It’s unfortunate how many people view any disagreement as an attack on something sacred that must never be criticized. I also think it’s a pervasive problem in politics, but I won’t go there because I don’t share that same interest in discussing politics as I do philosophy (especially theistic philosophy).

    • Okay, I initially responded to this and deleted it because after rereading it I realized what a goldmine has been opened up for both of us to understand THE major theme of this entire discussion. I will start by quoting.

      You said:

      “In other words, that as a materialist I should see love for my wife and children as something purely physical and having no profound meaning beyond that, but that in truth I don’t see it that way.”

      Your explanation:

      “I view it in the same way I view the connection of the brain and the mind. In other words, the unromantic, clinical, sterile “truth” is that love is nothing beyond the physical secretion of Oxytocin. If you disrupt that secretion the feeling of love is gone. If you cause that secretion in connection with a random stranger you can cause the feeling of love to appear for that stranger. This, however, does not prevent people to “wax poetic” about love because our brains are capable of creating other cognitive events that can combine to produce beautiful and deeply moving sonnets, paintings, novels, songs, etc.”

      Okay, so as far as the brain goes, stop the Oxytocin and you stop the “love” emotion. Does this mean that you love your daughter insofar as oxytocin is flowing? Once it quits it is revealed that your “love” was mere chemical flow, i.e. you no longer love her?

      “Not so fast,” you’ll probably say, “read the rest of the quote.” But as I read I only find more of the same. I find no argument that your love has anything beyond mere physical processes, the same mechanical processes and/or emergent states, if you prefer, which are no more “meaningful” than wetness is to H2O. So what if one is able to “wax poetic” and produce “cognitive events” that are beautiful (e.g. poets, paintings, etc.). You’ve not teased out your theory to the end product: your end product is meaninglessness. Love, waxing poetic, and painting not excluded.

      You then said: “So I’m not sure why atheism is unlivable.”

      It’s unlivable for the simple reason that ultimately human beings cannot live without meaning – actual meaning, not manufactured meaning to fill in the gap of existential despair (read Viktor Frankl’s “Mans Search for Meaning,” on his experience with this during his 3 year stay in 4 different Nazi prison camps (he’s a non-religious writer, btw).

      Then this: “It’s like painting by numbers; beneath the awe inspiring beauty of a work of art that causes emotional turmoil within the audience upon seeing it, can be nothing more than a series of lines and numbers with instructions of what paint to apply where, and how.”

      Exactly! Beneath the love for your daughter is mere physical causality. You are merely parroting what the total system would have you feel, say, and think. Man cannot live this philosophy. Once full honesty kicks in you must become the mechanical being you believe you are – emotions must be seen for what they are – delusions. At that point you either become cold and calculating (like any other mechanical device) or you kill yourself from lack of meaning (happens all the time, please don’t think I’m swinging for the fences here).

      For me to truly understand where you’re coming from I’d need you to answer my above critique and demonstrate what “meaning” is in essence – what its tethered to – within the context of a purely physical, causal reality.

  18. I’ll say not so fast because I oversimplified the matter in my response. It’s not *just* oxytocin that is responsible, but it *is* entirely biological. Much like vision is not the product of just one area of the brain interacting with the eyes, “love” also draws from several areas and hormones. Studies have shown that serotonin flows “freely” (I’m not a biologist so my descriptions will not be entirely precise and accurate) at the beginning stages of a relationship causing that feeling of infatuation. Once oxytocin is introduced to the mix we start feeling that “bond” that is more than sexual attraction. In the process of the relationship, however, there are many parts of the brain that are stimulated and each one plays a different role in that relationship. My point is that I oversimplified the role of oxytocin in love, but that the fact remains that if you interact with the brain in specific ways you can eliminate or cause such feelings.

    Another way to understand this is by the simple question: if you remove the brain of a person, can she still experience love?Assuming they can be kept alive, of course, with machines. Or how about psychopaths? Psychopathy is characterized by the inability to experience any emotion. If love is a spiritual connection, why is it that psychopaths don’t experience love? Do they not have a soul?

    Of course the problem with claiming that love is somehow spiritual is that it makes love something based upon speculation. Since we have no indication whatsoever of the existence of a soul, the statement “I love you” then becomes a conditional statement which requires an assumption that something exists which has never been shown to exist. Whereas with the understanding of love as a feature of biology the statement becomes one grounded in reality.

    And why stop at love? What about other emotions? Is hate a spiritual effect like love? Are anger, sadness, empathy, generosity, and other emotions also spiritual? if so, why is it that we can alter these emotions with medications that interact with the brain?

    You said humans cannot live without meaning and then you separated “meaning” between real and manufactured. I contend the opposite is true. I feel no objective meaning in my life whatsoever, yet if you find me dead with a suicide note next to me you can be sure it was murder because suicide to me is so far from a possibility that I have trouble even sympathizing with those who take their own life (unless their life was so horrifically unlivable, or they had a physiological disorder that altered their abilities to reason). I “love” life, but I don’t feel any meaning other than what I have “manufactured” for myself.

    For clarity I will break up your paragraph I want to address in individual points. “Beneath the love for your daughter is mere physical causality. You are merely parroting what the total system would have you feel, say, and think.” Indeed, this true. It might not seem as poetic, or romantic, but it’s no less true. And it’s hardly different than a belief in “soul mates.” If the problem here is a lack of freedom to choose, then do we not have that same problem with soul mates?

    “Once full honesty kicks in you must become the mechanical being you believe you are . . .” Not so. The illusion is that we are not what we are, but we can know what we are without affecting the illusion, and thus we can continue on without it affecting us. Much like I can know how movies are made and not have that ruin the movie for me.

    ” . . . emotions must be seen for what they are – delusions. At that point you either become cold and calculating (like any other mechanical device) or you kill yourself from lack of meaning . . .” Quite the opposite. An emotion can’t be a delusion if it has a provable biological origin, and grounding, behind it. It’s more of a delusion, in my opinion, if it’s based on assumptions about unprovable entities (such as the soul). Furthermore, I have no interest in killing people or myself (as I mentioned earlier). I’m aware that my emotions are purely biological, yet I still feel the desire to help those in need, I cry when I’m emotionally hurt, I laugh when I’m happy, I feel satisfaction when I give to, or help, others. In fact, as I mentioned earlier, it is those people who are biologically incapable of emotion that are cold and calculating. We call them psychopaths precisely because they cannot experience emotions (biologically), not because they lack a belief in the spirituality of emotions.

    So what is meaning (?), you asked me. From a purely physical standpoint it would appear that our goal in life is to prolong life. In terms of evolution this manifests in increasing the survivability and potential of our genetic continuation. All else seems to stem from this. But that is an explanation that is below the surface, perhaps even the foundation of all things. The rest is circumstantial and dependent on the life we were born into. But I fear we might be conflating “meaning” with “purpose.” So the question “what is the meaning of life?” is different from “what is the purpose of life?” If you mean “meaning,” not “purpose,” I have to ask what do you mean by “meaning”? As far as I can tell it’s an inherently illogical question because it requires the presupposition of an external consciousness who created us with a meaning. So the meaning would be not ours, but our creator’s. From a materialistic standpoint, however, there is no such meaning because there is no intent. We are simply an inevitable outcome (though not necessarily a final one) of a series of events. We are but a link in a causal chain.

  19. I would also like to point out that I’m not sure what the argument is now. This seems to be leading in the direction of an informal fallacy known as an appeal to consequences. This goes back to what you wrote earlier about the argument changing the facts. It can change the perceptual facts (like I illustrated with my analogy of the pencil), but not the objective facts. The consequences of a fact, whether positive or negative, do not negate the objective reality of that fact.

    I could also use a similar argument from consequences and say that if love is a choice then at what point does a parent choose to love their kids? I don’t remember any time where I decided to sit down and weigh the pros and cons of loving my children. And I’m sure you didn’t either. Did you? Did you sit down to try and figure out which choice to make (love your kids or not), or did you just feel the love?

  20. Hello again, Pavlos. Sorry for the delay. I’ve been terribly busy and did not want to cheapen my response with anything but a full treatment of yours so I waited till I had a good amount of time to go through it.

    But, time is still nagging even now so let me use the quote-and-respond method:

    You said: “I’ll say not so fast because I oversimplified the matter in my response. It’s not *just* oxytocin that is responsible, but it *is* entirely biological.”

    Great. Let’s use this thesis for the rest of my responses and see what we come up with.

    Then: “Another way to understand this is by the simple question: if you remove the brain of a person, can she still experience love? Assuming they can be kept alive, of course, with machines. Or how about psychopaths? Psychopathy is characterized by the inability to experience any emotion. If love is a spiritual connection, why is it that psychopaths don’t experience love? Do they not have a soul?”

    I would say psychopaths have a mental illness, and the only way we would know this is by measuring them against the “normal” functioning human, which is someone with an innate knowledge of good and evil, love and hate, as something real and not something subservient to mere survival instinct. According to materialism the whole assumption that love is something real in itself, due to in possessing supra-natural elements (i.e. soul, spirit, etc) and not “entirely biological” is a deception. Btw, in the psychology field it is a matter of speculation that certain psychopathologies are beyond healing, not a matter of scientific fact.

    Then: “Of course the problem with claiming that love is somehow spiritual is that it makes love something based upon speculation.”

    Everything is based on speculation. Materialism speculates that it is all based on firing neurons; indeed, that it can be absolutely reduced to firing neurons (emergent properties notwithstanding). Hence, again going back to our initial argument, “we” are not actually “speculating” about anything. Speculation requires a free-agent to collect data and come to a conclusion based on rational inference – reasoning, logic, etc – and not according to biological fiat. It is not proper to label anything our brain produces as “speculating,” rather it is simply what the total system delivered to us. It is a matter of determined outcome, not reasoning between “true and false” or whatever.

    Then: “Since we have no indication whatsoever of the existence of a soul, the statement “I love you” then becomes a conditional statement which requires an assumption that something exists which has never been shown to exist. Whereas with the understanding of love as a feature of biology the statement becomes one grounded in reality.”

    Whoa, slow down. So many heavy-weighted words here. Let me say briefly (because a full treatment would take volumes) that your own subjective understanding of what is meant by things like “indication,” “existence,” “soul,” “never been shown,” “grounded in reality,” are themselves subject to argument and have not been properly teased out. According to my presuppositions all these things have been demonstrated. Your presupposition of materialism prevents you from accepting these demonstrations (or, properly speaking, the impersonal, non-rational firing neurons in your brain have not allowed you).

    Then: “And why stop at love? What about other emotions? Is hate a spiritual effect like love? Are anger, sadness, empathy, generosity, and other emotions also spiritual? if so, why is it that we can alter these emotions with medications that interact with the brain?”

    Acutally, why stop at emotions, let us push the ball downfield and talk about “good and evil.” Where are these qualities/states discovered in biology? If the psychopath is incapable of empathy and tortures children for fun, is what he does grounded in any reality of “evil”? Of course not. If, proceeding from your thesis that everything is emanating from human consciousness is “entirely biological,” then we should not use such notions as good and evil. Nietzsche should have made this lesson clear for all atheists long ago. What the torturer does is merely biology in action. Nothing more, nothing less. He is an organism bent on survival. Survival is neutral.

    Then: You said humans cannot live without meaning and then you separated “meaning” between real and manufactured. I contend the opposite is true. I feel no objective meaning in my life whatsoever, yet if you find me dead with a suicide note next to me you can be sure it was murder because suicide to me is so far from a possibility that I have trouble even sympathizing with those who take their own life (unless their life was so horrifically unlivable, or they had a physiological disorder that altered their abilities to reason). I “love” life, but I don’t feel any meaning other than what I have “manufactured” for myself.

    Perhaps you simply have not been properly tested. I highly doubt that without a sense of meaning you would survive one week in a Nazi prison camp. And, again, let’s be consistent with the language: you don’t love life; that is a misnomer in your theory. You have a biochemical drive to survive and procreate. Thus, your “love” MUST be merely utilitarian for this non-rational, biological “goal.” You wouldn’t describe a flower’s goal of procreation and survival as a love affair with bees.

    Then: “For clarity I will break up your paragraph I want to address in individual points. “Beneath the love for your daughter is mere physical causality. You are merely parroting what the total system would have you feel, say, and think.” Indeed, this true. It might not seem as poetic, or romantic, but it’s no less true. And it’s hardly different than a belief in “soul mates.” If the problem here is a lack of freedom to choose, then do we not have that same problem with soul mates?

    I don’t believe in “soul mates.”

    (*skipping ahead, forgive me if I leave out anything important*)

    Then: “So what is meaning (?), you asked me. From a purely physical standpoint it would appear that our goal in life is to prolong life. In terms of evolution this manifests in increasing the survivability and potential of our genetic continuation.”

    This theory is ousted by human history. Our survival has not been enhanced as a species due to ideas about love, hate, good and evil. They are the sources of the worst in human nature in terms of war and death. Ultimately, given enough time, these concepts will likely obliterate humanity. No other species is so defunct as the human, in materialistic and evolutionary terms.

    Then: “All else seems to stem from this. But that is an explanation that is below the surface, perhaps even the foundation of all things. The rest is circumstantial and dependent on the life we were born into.”

    Okay. If all stems from mere survival instinct and procreation drives then, again, “meaning” or even “purpose” if you like, fall into an inescapable ditch of nothingness. You, me, and everyone else, our children included, will all die. Thus, there is no ultimate fulfillment of THE foundation of all things. If the purpose is to survive, but survival is the one thing guaranteed to not happen, then its all totally meaningless.

    Then: “But I fear we might be conflating “meaning” with “purpose.” So the question “what is the meaning of life?” is different from “what is the purpose of life?” If you mean “meaning,” not “purpose,” I have to ask what do you mean by “meaning”? As far as I can tell it’s an inherently illogical question because it requires the presupposition of an external consciousness who created us with a meaning. So the meaning would be not ours, but our creator’s. From a materialistic standpoint, however, there is no such meaning because there is no intent. We are simply an inevitable outcome (though not necessarily a final one) of a series of events. We are but a link in a causal chain.”

    If materialism is correct then both meaning and purpose are illogical. In fact, “illogical” is illogical. There is only what is. I cannot say that biological processes delivered illogical thought anymore than its opposite. Logic is a not intrinsic to nature but is relative to an observer of nature. You’ve proposed that mental functions are entirely biological. Thus, our observations are entirely biological and not that of a neutral observer with free-agency outside the constraints of natural determinism. As you say, we are “but a link in the causal chain.” Consistency here requires that you understand yourself as a “thinker” that is merely pushed by a prior causal event, becoming as a result a causal event determined wholly by non-rational biological movement to a determined end (like a computation device). That determined end has nothing to do with “truth.” Thus, materialism as a philosophy slits its own throat.

    Let me conclude and then I’ll give you the last thought in this discussion, since it seems that we’ve exhausted the subject, at least to some degree. What I have been arguing is essentially an ontological argument. Things like love, hate, good, evil, pain and pleasure can be described in biological terms, but only to a very limited extent. One can attempt to reduce, say, love to neurons firing, and even if one does so with perfect scientific precision he does not account for love’s ontology. In short, firing neurons may in a purely physical sense “cause” the feeling of love, but they are not the same thing as the feeling of love. You can get a causal reduction of love to neurons firing but not an ontological reduction. They are two distinct features. And if we could then we have no reason to label emotions anything but the very chemicals that cause them. Next time, be honest with your wife and say: “Honey, I have an accelerated dose of Oxytocin in my brain for you right now, let’s achieve our biological goal of genetic continuance and procreate.” Leave the BS of love completely out of it. Next time someone on the road cuts you off blame it on the total system. The other driver was only doing what he could do in the moment, anger is futile and contributes nothing to your goal of survival, actually it endangers you. In short, I’m only asking that you take up the logical straight shot of your philosophy and become congruent with it. Nietzsche and Freud said it all (though even they themselves couldn’t live their philosophy): it all comes down to will to power and will to pleasure. You must get beyond good and evil, right and wrong, love and hate, if you’re going to be the organic mechanism – the link in the causal chain – you claim to be. Don’t fail the existential test.

    • After my initial draft I realized I had spent too much time on rather irrelevant points (like how much time I spent on what actually constitutes psychopathy). So instead I’ll attempt to “trim the fat” with this rewritten version.

      Having said that I’ll start with theses quotes which offer some insight into my position:

      “Extraordinary claims require extraordinary proof.” (Marcello Truzzi)
      “A wise man . . . proportions his belief to the evidence” (David Hume)

      The claim of the existence of a soul is an extraordinary one that is entirely lacking in evidence. It’s merely based upon other claims which also lack evidence. The same is true for competing theories of materialism (such as dualism). Materialism, on the other hand, is based on observation and validated by its predictive capabilities. No philosophical argumentation is required when there is scientific verification of the materialistic predictions. For example, a materialistic prediction would be that if all things originate from a materialistic cause then absent the material you should expect the emergent phenomena to also be absent. Remove the brain and there is no consciousness. Even without such extremes there are also studies that shown that decisions are formulated in the biological brain prior to them appearing in the awareness of the individual. If dualism were true you would expect/predict the opposite to be true (that consciousness would dictate decisions to biology). Since, however, we can actually see the thought emerge in the brain by as much as 7 seconds before the individual is consciously aware of them, we can conclusively state that consciousness is materialistic in origin. Absent consciousness you also have an absence of all “byproducts” such as emotions (in other words, an unconscious being does not experience love, hate, empathy, sympathy, jealousy, etc.).

      So materialism is based on knowledge. Knowledge is a justified true belief. Any belief that is not justified or true is nothing more than a belief. This is where speculation comes into play. It’s defined as “the forming of a theory or conjecture without firm evidence.” Inference, however, is defined as “a conclusion reached on the basis of evidence and reasoning.” These are diametrically opposite and cannot be used interchangeably as you did.

      The existence of a soul is by definition speculation. There is no firm evidence for its existence, unlike the materialistic claim which is based on the firm evidence of the existence of the brain.

      Now, I realize we differ considerably on some concepts (“indication,” “existence,” “soul,” “never been shown,” “grounded in reality,””) If we ask each other for evidence of such claims I can provide scientific studies of observations. What would you provide me with? If it’s theological evidence then I’m assuming that would be a collection of writings, all of which are in dispute in terms of authenticity, content, and basis. If it’s philosophical evidence then it would be based on further disputed claims that are also based on other philosophical claims that originate from a time when people still thought earthquakes were the result of an angry god or giant stomping around earth. Around the same time that the ancient Greek philosophers first started philosophizing about consciousness they were surrounded by people who thought lightning was thrown from the heavens by Zeus. My point is that these were seemingly valid arguments to make in the absence of even a basic understanding of the brain, but they have no room in the modern era. Similarly, when I say a soul has never been shown to exist, you might want to counter that with tales of personal anecdotes (modern or ancient), yet my counter is based on tangible evidence.

      “This theory is ousted by human history. Our survival has not been enhanced as a species due to ideas about love, hate, good and evil. They are the sources of the worst in human nature in terms of war and death. Ultimately, given enough time, these concepts will likely obliterate humanity. No other species is so defunct as the human, in materialistic and evolutionary terms.”

      You are both correct and incorrect simultaneously here. Human history has, in fact, shown the exact opposite. We (Homo Sapiens) exist specifically because of those traits. Their absence, especially to the degree we possess them, is the reason why we have caused the extinction of all other variants of the species Homo, allowing for the rise and domination of Sapiens. They are the source of the worst in us, and they likely will be our undoing. Indeed, no other species is as uniquely destructive and intolerant as us (minor theological point I can’t help but make: if we are made in the image of our creator, what does this say about him? Nothing pleasant at all.)

      ” If the purpose is to survive, but survival is the one thing guaranteed to not happen, then its all totally meaningless.” Of course it is, for the same reason that I mentioned in my previous comment. Without a creating consciousness there can be no “meaning” to be fulfilled. In terms of existential worth we are no more special that your average meteorite. Evolution is not a purpose driven mechanism. Natural selection simply means that “he” who has the genes which increase survivability gets to pass on those genes in greater numbers and with greater success than “he” who does not. The giraffe with the tallest neck will pass on the genes for that neck by virtue of the fact that it will continue being able to feed even when the leaves are gone from the lowest hanging branches. He will, then, out-compete giraffes with shorter necks. But evolution, being not a consciously created mechanism, cannot do anything to ensure our eternal survival. When our sun goes supernova we will all be obliterated no matter what genes we carry.

      “Consistency here requires that you understand yourself as a “thinker” that is merely pushed by a prior causal event, becoming as a result a causal event determined wholly by non-rational biological movement to a determined end (like a computation device). That determined end has nothing to do with “truth.” Thus, materialism as a philosophy slits its own throat.”

      This is a non sequitur. How does the determined end have nothing to do with truth? It is the only road leading to it. Indeterminism is chaotic at best, and leads to subjective truths because nothing is connected to anything prior and everyone is simply floating through a conscious existence operating independently and with no tangible grounding. Indeterminism is like floating through space; which way is up and which down? The “truth” is there is no up and down except for how you perceive it. You need a grounding to determine truth, and so determinism is like walking on earth with a firm grounding clearly indicating and differentiating up from down.

      The materialistic ontology of feelings such as love are evident in our evolutionary history.

      “Honey, I have an accelerated dose of Oxytocin in my brain for you right now, let’s achieve our biological goal of genetic continuance and procreate.” Leave the BS of love completely out of it.”

      This is, again, an appeal to emotions. The argument is meant to appear flawed because it sounds so terrible. But, again, the ugliness of the truth does not prevent that from being true. And if we can make something ugly appear more pleasant, why should we not do it? I don’t accept this argument at all. A house is nothing more than a compilation of raw materials put together in a specific form, so why bother making it a home? Next time just buy/rent the most basic structure and say “family, let’s live in between these walls that keep the elements out.” It just doesn’t make sense as an argument. We decorate, paint, furnish according to personal preference, hang pictures of loved ones on our walls, and other such rationalizations precisely so that we can enhance our experience with it. But your home doesn’t cease to be a home once it’s no longer a mere bare structure.

      “Next time someone on the road cuts you off blame it on the total system. The other driver was only doing what he could do in the moment, anger is futile and contributes nothing to your goal of survival, actually it endangers you.”

      That is exactly what we should all do, so I don’t understand why I get the impression you offer this as a criticism of something. Are you saying I should focus my anger on that person, drive him off the road and then attack him with a weapon until he’s hospitalized and I’m arrested?

      “Perhaps you simply have not been properly tested. I highly doubt that without a sense of meaning you would survive one week in a Nazi prison camp.”

      If I didn’t know any better I would probably take offense to this. I’m very much aware of the fact that, though I have had my share of negative experiences, I have not been existentially tormented to any degree resembling an experience in a Nazi camp. Having said that, however, I’m driven by a personal desire to prolong my existence, so it’s irrational to suggest that I would not last a week because I lack some imagined meaning handed down to me by an imagined being.

      “And, again, let’s be consistent with the language: you don’t love life; that is a misnomer in your theory. You have a biochemical drive to survive and procreate. Thus, your “love” MUST be merely utilitarian for this non-rational, biological “goal.” You wouldn’t describe a flower’s goal of procreation and survival as a love affair with bees.”

      You seem to be almost offended by my use of the word “love” in this context. A flower does not have the same range of emotions or conscious processes that people have so the term of course does not apply factually, however this has never prevented a poet from doing so.

      Lastly, I have long maintained that persuasion without logic is dangerous, yet logic without persuasion is pointless. So perhaps this might as well be the final word since I’ve been disappointed to see us return to defeated arguments as though they still hold true.

      This has been interesting, and clearly this will not be the final word between us in other topics.

      And since I started with a quote, I’ll end with one as well:

      “The truth may be puzzling. It may take some work to grapple with. It may be counterintuitive. It may contradict deeply held prejudices. It may not be consonant with what we desperately want to be true. But our preferences do not determine what’s true.”
      ― Carl Sagan

    • Well done again, Pavlos.

      I promised you the last word, but if I may make one observation from your reply: you treated my arguments as if I were trying to “prove” materialism false through appeal to emotion. Actually, with our last back-and-forth, I was trying to prove that you, yourself, are not capable of living the espoused philosophy of materialism. In fact, this entire discussion proves it. If you truly, to your core, believed everything was meaningless you would have no need to even carry on this conversation. No one tries this hard at something meaningless. 🙂

      It’s been a lot of fun. Looking back over my past discussions with others its clear that this discussion was the best, and indeed has stretched me the most.

      Cheers, hope to see you around again!

    • With all do respect, Pavlos, and if you wouldn;t mind, I would have a few bones to pick with your comment.

      ““Extraordinary claims require extraordinary proof.”” Okay, so first off, what IS an extraordinary claim? Is it scientifically and objectively determinable what is extraordinary or not? No. It’s unscientific and subjective. What is extraordinary to you may be quite commonplace for me and viceversa. This adage may seem compelling because it subtly plays on preconceived, commonsensical notions of “normal” and “beyond normal”, of “familiar” and “alien”, and of “ordinary” and “extraordinary”.

      “The claim of the existence of a soul is an extraordinary one that is entirely lacking in evidence. It’s merely based upon other claims which also lack evidence. ” There are three issues here:

      1. It is extraordinary only if you presuppose philosophical materialism as being true, but ontological materialism is a metaphysical claim, not a scientific truth. To someone else, the idea that subjective experience, self-awareness, introspection, qualia, reason, complex thought and identity can arise out of inert matter and may have been at least partially an accident of evolution, or the view that the order, harmony and logic we observe in nature is the byproduct of the clash of the unconscious forces of nature and came from a primordial “mindless” cosmic freak accident, so to say, is extraordinary.

      2. Lack of evidence is in no way whatsoever to be conflated with evidence of absence. There was no evidence of dark matter when it was first proposed back in the ’30s, but in the late ’60s evidence that was convincing enough could finally be brought forth. Yet dark matter was always there, both in the ’30s when it was first presented and rejected, and in the ’60s when it gained acceptance in the scientific community.

      You later mention that observation confirms materialism, but observation is done via our senses, and can you prove that your senses are accurate? That they do provide you with an accurate picture of what’s going on? I mean, that would be an extraordinary claim in my eyes. Can you step outside your body to prove that our senses work good? The idea that extraordinary claims must produce extraordinary evidence can at the very best be used to argue for agnosticism in a scientific context. Anything beyond that is personal, subjective preference.

      “Materialism, on the other hand, is based on observation and validated by its predictive capabilities. No philosophical argumentation is required when there is scientific verification of the materialistic predictions.”

      Okay, so the geocentric model too was based on both observation (as far as it was possible back in the day) and had good predictive value – draw your own conclusion from that. Besides that, if observation is our main method of gaining knowledge, then the only way to prove a metaphysical claim is to have perfect observational capabilities which, to the best of our knowledge, is impossible. Materialism is as a result a useful philosophy for epistemic purposes, but its ontological value is overestimated.

      What’s more, let’s imagine for a moment that one day we come close to observing something that would best approximate our notion of a soul; even if we weren’t able to interact with it or influence it and even if measuring it would yield very peculiar results (yet we still can observe enough to deduce that it’s the “soul”), it would not be classified as something different from matter – like “unmatter” or “immaterial substance”; it would most likely be classified as an unusual, different, new type of matter. That means that materialism – the idea that only matter exists – would change depending on what we define as matter. If we define as matter only that which we can observe or interact with, then we find ourselves adopting an ontological position based on an epistemic limitation (our senses). That means, and it bears repeating, that what we define as matter would change based on what we discover about matter, so materialism as a philosophy would end up being just a very eloquent way of “stating the obvious”.

      “Remove the brain and there is no consciousness. ” Remove your computer and you have no internet. The computer, however, is a medium for the internet, NOT its producer.

      If you visit one of those primitive tribes that live in perfect ignorance of the world at large, give them a PC with a wireless internet connection, a very simple interface and minimal instructions on how to browse it, then take it away from them and ask them were did the videos, pictures and writings found when browsing the internet came from, they would say that it was the computer that generated them, because they couldn’t even define the internet, see it other than its manifestations in the form of pages visited, or detect its source. They would thus confuse the medium with the source.

      ” Even without such extremes there are also studies that shown that decisions are formulated in the biological brain prior to them appearing in the awareness of the individual.”

      Okay, those experiments have glaring flaws. At most, those experiments merely demonstrate that there is mental activity in the brain when making a decision. The most famous of these studies is the Libet experiment. To begin with, the idea of “the awareness of the individual” to make the decision is ambiguous at best, so that itself casts shadows on the findings themselves. There was a study suggesting that the “readiness potential” usually cited was not the decision itself, but merely a sign that the brain was paying attention. Also, another study (Banks & Isham, 2009) aiming at the awareness of the intention to move showed that it was not predetermined by the readiness potential. Not only that, but Libet himself said that our consciousness can still interfere at any time with a “veto” which is consistent with Schopenhauer’s “man can do as he wills, but can’t will what he wills”.

      Secondly, the test subjects are people who already had long agreed to follow the scientists’ instructions. That in itself had likely settled them into a compliant disposition. Thirdly, and most importantly, what kind of decisions are we talking about? Motor decisions? And the studies demonstrated what – that motor decisions are taken subconsciously? We know mighty well that is a trivial truth. You don’t direct any conscious thought towards walking other than the decision to walk; the movement of the feet is subconscious. Moreover, what about learning to play an instrument? During the learning phase, the subconscious does not play the same role as it cannot know ahead of time what it will do; consciousness and attention must be involved in the process. Plus, what about when you play high-speed “whac-a-mole”? The subconscious mind cannot no ahead of time where the head of the next mole will emerge.

      ” Since, however, we can actually see the thought emerge in the brain by as much as 7 seconds before the individual is consciously aware of them, we can conclusively state that consciousness is materialistic in origin.”

      Why didn’t you say it all the way: the experiments that “demonstrated” that we can predict decisions for up to 7 seconds had a staggering accuracy of 60% – I can do just slightly worse by flipping a coin. What about the other 40%? Can you leave them unaccounted for and just focus on the 60% because they prove a point you side with? It smacks of Texas sharpshooting. Those experiments too apparently disregarded one aspect: anyone who’s taken a multiple choice test will likely remember an answer coming up in the mind as soon as they read the exam question, but their intention to tick it was withheld for a good while during which we analyzed the question before deciding if the first answer stayed or should be ignored. This example of mine also introduces one aspect that ALL these studies seem to ignore: rational deliberation. When choosing an answer in a test that you want to pass, you invest a great deal more conscious thought and energy than with moving a finger or pushing a button as most these studies have their subjects do. I probably shouldn’t even mention the major decisions in life that far eclipse a simple test.

      Far, very far from proving consciousness as “material in nature” these particular studies at most demonstrate that consciousness has neural correlates, in much the same way objects cast shadows on walls when lit; modifying the light source will influence the shadow on the wall and what it looks like, even if the object casting it did not move. It is silly though to equate an object with its shadow. No study yet has conclusively found whence qualia come from, how they are formed, how subjective experience arises or the specific mechanism by which all neural processes coordinate with one another in a way that makes them “you”. Plus, it has been mentioned that thoughts are merely electrochemical signals in the brain, but it says nothing about the propositional content of the thought and its influence upon this electrochemical mechanism.

      Sound consists as we know, of vibrations in the air. If an employer tells you “we’ll hire you” or “we’ll fire you” in the exact tone and speed of voice, the vibrations produced will be extremely similar to one another (save for the “h”/”f” distinction). Yet, their impact on you will be worlds apart. Is there anything specific about those vibrations themselves that spark these wildly different vibrations of air molecules? The point is that they pertain to a desire in our minds – “career”. Yet, what exactly is the physical form of this thought? Where exactly is “career” located in your brain? In what part, in what neuron, and how? How to differentiate between “career” and “family” in terms of the physiology of thoughts? If two electrical signals travel from one place to another, how would you distinguish which one is pondering on the wage and which one on job security when considering a job to apply for?

      “So materialism is based on knowledge. Knowledge is a justified true belief. Any belief that is not justified or true is nothing more than a belief.”

      Okay, that’s a false implication. One (perhaps surprising) problem which Eric has evoked in this post (to me at least it seems clear) and which may seem to be elusive is that by materialism or philosophical naturalism the acceptance of something as being true is not dependent upon its truth value, because thoughts and ideas are merely electrochemical signals determined by the antecedent causes in which we have no say as to where they would go. Therefore, you can only have the impression of something being true, which is not the same thing as it necessarily being true. You can claim that you establish something as being a justified true belief upon evidence which may seem true, but then the aforementioned problems kicks in: you yourself don’t choose how to interpret evidence; your brain does.

      The evidence doesn’t determine your beliefs; if we assume as accurate the studies about the brain making a decision well before we’re aware of it, then your brain “decides” whether it accepts it as true or not based on the mix of electrochemical mix that goes into it; before you even get a chance to formulate your thoughts, your brain has called on how it interpreted the evidence. You may think you “reached” a conclusion by deliberately weeding out inconsistencies, but you were merely “compelled” to it. Therefore, you may have the impression that you are correct; that a thought or an idea is logical and well-reasoned, but it doesn’t mean it is so. You may be suspicious of an idea even if it seems well thought-out for fear of having left something out, but then even this suspicion comes from a series of uncontrolled – uncontrollable, in fact – interactions in response to a stimulus that may or may not be so.

      For instance, both you and Eric have the genuine sentiment of being right, yet you are at loggerheads on many accounts – central accounts to the matter being discussed, not peripheral ones. If we pick one such account on which you diverge, the question can be asked – which one of you is right, which one of you uses better reasoning? You may say it’s you because your brain interpreted the evidence in a way that reached a certain conclusion. Eric would say it’s him because his brain determined him to see the evidence in favor of his position. You can point to something – you may say it’s evidence in favor of materialism, he may interpret it in a completely different way. Take the famous, undying debate between atheists and theists: atheists point to the laws of nature as evidence against God; on the contrary, theists see them as evidence of God’s glory and greatness. As a result, the claim that “My reasoning is better” in this context becomes “My reasoning is better because I feel so”. You could then say that both you and Eric are endowed, as a result of evolution, with the capacity for logical reasoning, which bring us to the question of why you differ in your opinions so wildly?

      That would suggest that one of you knows the other is right but is intentionally dishonest for whatever reason. This kind of difference of opinion can be observed even in the highest-flying circles of academia – both Stephen Hawking and Roger Penrose are brilliant physicists, determinists and atheists, born in the UK in around the same generation, but Stephen Hawking is philosophically a positivist, while Penrose is a platonist. Another similar difference of opinions is the famous debate between Einstein and Bohr: determinism vs indeterminism; the father of general relativity vs one of the fathers of quantum mechanics.

      Anyway, I enjoyed the exchange you had with Eric on this subject; very interesting, I must say. I just felt the need to speak my mind on some issues I deemed to be a bit off-target.

    • Saturnian Bartender
      I find many points you make here very convincing.
      As I see it, materialism relies on an axiomatic statement that all there is is the natural and hence no supernatural exists. If you run with this, as Pavlos’ very convincingly does, you arrive at existential nihilism. I thought similar to Eric that that was unlivable and would cause one to question whether the axiom is a reasonable starting point. I have to admit I am impressed by Pavlos’ assurance that one can indeed live with such a worldview, in spite of the fact that he seems to fully realize the consequences. I could not. What baffles me most is that one would actually carry on a deep discussion like this having to admit that every bit of it would be unaffected by ones agency (in fact the concept of agency would not even make sense anymore). What I would type at this moment would be fully determined (give or take a little randomness from quantum fluctuation perhaps) and all I can do is watch it (the movie) unfold and be content with the illusion of “me” “reasoning”. As you pointed out neither Eric nor Pavlos could be “right”, because that concept also loses meaning as nothing has any meaning and hence meaning itself becomes an empty concept. Hard to swallow for me. At some point appeal to consequences may become valid if one is forced to enter the realm of absurdity.
      I will go back to my own axiomatic system now which leads me to go to Mass to begin the Easter Triduum to celebrate the resurrection of Christ our Savior and Lord. And I hope in the process the transcendent as well as the material aspects of what comprises me will be nourished. I wish the same for everyone here.

  21. These arguments are all really well reasoned and thought out, Atheist and counter-Atheist. I am christian, but when I discuss religion with my friends (most of whom are atheist) they just give simple and unreasoned responses which don’t really prove or imply anything. Eric, you are probably the best Christian apologist I have ever witnessed, but Pavlos, your atheist arguments are well thought through as well, if only my friends showed similar respect and reason. This whole thread has been insightful, and very interesting to read.
    Greg

    • I think, without meaning to, you have just stated that nobody with great faith has high intelligence and great reasoning skills. I don’t disagree, I just think it rather ironic.

  22. Hi Eric 😀 Wonderful blog! Where on earth did the notion that atheists are smart come from? Look at C.S Lewis for example. He is a genius and he used to be an atheist. But he found Jesus and never looked back. Jesus says in the bible “Seek and you shall find”. I know from my own experience that this is true. God knows what we take an interest in and speaks to us in a way only we would understand. I have known so many brainy christians/pastors I have lost count. Godbless! I will keep praying for all of you 😀

  23. Pingback: Atheism on the Ropes: Materialism’s Failure to Provide an Adequate Account for Value | Eric Hyde's Blog

  24. Great summary of Lewis’ argument! Besides the self-refuting part of the materialist account of the universe and human reason, there is the problem of explaining how something purely physical could give rise to something with at least something of the metaphysical about it.

    Supposedly mere physical matter and physical energy, with no other inputs, randomly combined to create Minds, like ours, that can apprehend and analyze metaphysical concepts,like “God,” “Truth,” “Justice,” etc. I don’t see how in the world science, being what it is (using physical instruments to observe physical events involving physical matter and energy) can possibly prove that something physical could be all that is responsible for something non-physical, such as a comparison between concepts of justice.

    Here’s a link to an article that explains what I was trying to say a little better and in more detail.
    http://www.meditations-on-life.com/why-i-am-not-an-atheist-christianity-atheism-faith-reason/atheism-vs-christianity-atheist-faith-vs-christian-faith/

    • Hi feivelsguide2014,
      I fear you may have completely ignored or perhaps misunderstood the theory of evolution. There is nothing “random” about how we evolved to the point of intelligence where we can grasp metaphysical concepts. I’m pleased that you are as impressed and awestruck by the existence of abstract thought and philosophy as I am but I’m disappointed you use it as “proof” for the existence of God.

      May I offer just a couple of points for you to think about?

      1) Science is not attempting to prove “something physical could be all that is responsible for something non-physical”. I feel like perhaps you have misunderstood what ‘Science’ is. The application of science is simply about trying to find the truth and find support for our theories on the workings of the universe. It is important to realise that the scientific method never claims to prove a theory to be 100% correct. It can however be used to prove something to be incorrect and it is through observation and refinement that most theories come to be accepted as true. So, it is true, I cannot use science to prove God does not exist but I CAN (and do) use science to prove that the Earth is 4.5 billion years old and that it is most certainly not the centre of the universe.

      2) Given the above, can you explain to me why you have made the logical leap from “science can’t currently explain conciousness” to “God must have created us”? I don’t mean to be dismissive but it comes across as very one sided there. You are willing to hold “science” under enough scrutiny to totally dismiss it as able to eventually explain life in full (which I’ve no doubt it can) but do not do the same to the theory of intelligent design. This is despite the fact that science has proven the Bible to not be factual (Adam and Eve, Noah etc which incidentally I don’t think were ever supposed to be taken literally).

      I’m afraid when I see people use the “I don’t understand it so it must be God” argument I just think they’re being incredibly ignorant or incredibly lazy. There are very many, often very compelling, arguments for the existence of God which you could use and we could debate but you have just taken the ‘de facto’ route.

      A final point on this article too; Ignoring the fact that atheism is the lack of a belief in something and so cannot be ‘disproved’ without proving the existence of God, it relies on the premise (conveniently ignored here) that “thought” is not a physical “mechanical” process. Well, naturalism doesn’t shoot itself. Thought is merely a process of chemical and electrical signals in the brain. It is only when you take a transcendental view on thought that a paradox appears. If you accept (and I’m not saying you should, I’m just saying you could thus rendering this article pointless) that the brain is just a machine then there is no paradox and no argument. Basically, a fundamental misunderstanding of the philosophical AND scientific perspectives mentioned here has given rise to a non-argument. Oddly enough, there IS a philosophical debate on this matter, it just doesn’t have any effect on Atheism.

      Regards:
      http://godreligionandme.tumblr.com/

    • “Thought is merely a process of chemical and electrical signals in the brain.”

      Then no thought is “true” but rather whatsoever the total system determined our brain to think next. By extention, science cannot discover the “truth” about anything since it relies on brains which produce ideas based on mechanical firing of neurons, firings which tract not with some mythical notion of “truth” but with natural cause and effect. Science relies on objective distancing to make observations of nature while excluding one’s own biases. How can one make legitimate observations of nature if nature is controling every aspect of the observing process? It’s hopeless question begging. Indeed, what exactly are we doing debating anything at all? We are only parroting whatever the total system determines us to say. If we have no free-agency (i.e., an antithetical phenomenon to mechanical processes) its just one bewildered ape arguing with another over truths that don’t exist.

    • Hi Eric,
      Interesting article and comments.
      You write,”By extention, science cannot discover the “truth” about anything since it relies on brains which produce ideas based on mechanical firing of neurons….”

      Science has never been about “truth”. WHAT IS TRUTH?
      A statement that corresponds to fact or reality?
      The word “truth” is a concept which has been conceived by humans for use as a conceptual label of validation on statement types known as propositions. Propositions are statements which propose an alleged case or scenario.
      But since truth ultimately stems from the validation of propositions, it necessitates an OBSERVER who must VALIDATE the proposition before they can label it as ‘true’ or ‘false’.

      It is obvious that the word “truth” is ultimately dependent on a dynamic process that an observer must perform before labeling a proposition as true/false.
      Since the concept of truth is ultimately dependent on a human’s subjective use of their limited sensory system, it is easy to understand why all truths are subjective; i.e. opinions. Truth is an observer-dependent human-related concept that is inherently subjective. As such, it necessarily resolves to none other than opinion. What is TRUE to you, is a LIE to someone else.

      This is why science has always been about reasonable and rational explanations which we can test and are falsifiable.

      You state, “But any thought which is not guided by what is “true” but guided rather by mechanistic, physical necessity is not rational……”

      Lets fix this “true” statement.
      “But any thought which is not guided by subjective opinion, but rather by mechanistic, physical necessity is not rational……”
      And subjective opinion is rational… always?

      Are you sure about this?

      The start of the summary of the argument reads:
      1) No belief is rationally inferred if it can be fully explained in terms of non-rational causes.
      2) If materialism is true, then all beliefs can be fully explained in terms of non-rational causes.

      ‘Causes’ plural of ’cause’.

      What is a cause? Cause is a verb, a doing word as we were taught in grade school.Sure we reify verbs into real things in casual speech. We may ask a runner,”Did you go for a run”? Or: ” How was your run”? But we all understand that “a run” is not a real thing and does not exist, it is just what the runner = an object did.
      Only an object acting on,(causing motion to), another object can be the cause of something.

      A non – rational cause?
      A, not able to be explained by reason, object, causing motion to another object.
      In sensible English a “non-rational cause” reads: An object that causes motion to another object that is not able to be explained by reason.

      An object is that with shape/form and for us to logically state that an object exists, we must be able to point to it, identify it as something with shape/form: therefore it must have a location.

      Shape/form + location = exist

      It therefore follows that a ’cause’ is an identified shape/form in a location acting on another shape/form with a location.

      Therefore a ’cause’, or causes cannot be non – rational.

      Beliefs:
      A belief is an acceptance that something exists or is true(true as in subjective opinion :), especially one without proof.
      How does one “fully explain” something that by definition is just accepted,especially without proof, and not able to be explained by reason ?

      The summary of the argument:
      1) No belief is rationally inferred if it can be fully explained in terms of non-rational causes.
      Definitions:
      Belief, see above.
      Rationally: reasonably; sensibly
      Inferred: conclude (something) from evidence and reasoning
      1) in English;
      “No acceptance that something exists, especially without proof, is reasonably or sensibly concluded from evidence and reasoning if it can be fully explained in terms of an object that causes motion to another object that is not able to be explained by reason.”

      This argument is based on sloppy English: Us working class men can forgive philosophers a lot of things, but not sloppy language.

    • Hi again Eric,

      Please forgive my confused writing as I am writing as I think and read your stuff :).

      You write, “any notion of good and evil, right and wrong, love and hate, etc., must also be treated as mythical. At best they are nothing more than helpful categories of thought, but categories with no existence (or ontology) of their own.”

      Why don’t people know the difference between a concept and an object?
      In physics we define an object as that with shape/form, or as everything that is not a concept.
      A concept is relational, a relation between objects, or we could say that a concept is everything that is not an object.

      Everything resolves to being either a concept or an object in our language.

      We point to objects and name them. We may even attempt to DESCRIBE them. We do not point to and name concepts. The best we can do is DEFINE concepts and give them labels to be helpful to us.

      For something to objectively exist, it must have shape/form: in other words be a some-thing rather than a no-thing and it must also have a location.
      Shape/form + location = exist.

      Theists rightfully use this same logic to establish their hypothesis for the existence of God. He is a something, rather than a nothing; and is somewhere or everywhere, rather than nowhere.God exists whether there is a human ape to experience Him or not.

      Concepts do not exist. They are relations between objects.

      Good, evil, right, wrong, love, hate are concepts. If you disagree, please direct me to pictures or diagrams or sketches of these things on the internet, or at least DESCRIBE them. Remember we only DEFINE concepts.
      Concepts are just subjective ideas in the heads of human apes. No human apes, no human ape love, hate,right,or wrong.No objects = no concepts.

    • Sean, thank you for your reply. I’m curious if this is the first time you’ve approached the argument from reason because many of your points don’t seem to fit with the argument. For example, you seem to misunderstand the general argument that is being made about non-rational causes. I quoted Reppert at length because I believed he gives one of the most succinct and easy to follow introduction to the argument. Let me pull from William Hasker to see if it will make more sense for you.

      He argues that one must have good reasons for the beliefs they hold, but under the assumption of a closed causal universe (i.e. the basic position of materialism/naturalism), no one ever accepts a belief because it is supported by good reasons. “In a physicalistic world, principles of sound reasoning have no relevance to determining what actually happens.” The entire process of thinking – or as you call it the “dynamic process” of a subjective observer – is governed by and explicable in terms of the ordinary laws of physics and chemistry. The mental properties of an event are irrelevant to its causal influence. Once a complete mechanistic explanation has been given for the act of thinking, no room is left for the reasons one might have for an action to play a role in bringing it about that one performs the action. If this is so, then the doctrine that all actions have complete neurophysiological explanations entails that no one ever does anything because he has reason to do so – that is, no one ever performs an intentional action. But this explanation is inconceivable not because it is self-contradictory but because the absurdity lies in the human act of asserting the doctrine. The occurrence of this act of assertion is inconsistent with the content of the assertion.

      If none of this makes sense for you try to imagine the argument like this: here are two different explanations of how Sally came to believe a particular proposition is true:

      “Sally believes so-and-so because she sees that it is supported by sound reasons.”

      Compare that with: “Sally believes so-and-so because of such-and-such antecedent physical conditions.”

      Now, which one of these reflects the materialist/naturalist point of view? The second one of course, and this physicalist description of the event of reason precludes the former, or, at the very least, renders the process of rational inference irrelevant to the final outcome of belief.

      The whole Wittgensteinian way in which you attempt to move around the argument doesn’t work, because you introduce unnecessary confusion in the mix by reasoning yourself out of things like “non-rational causes”, which is merely a description of the assumption materialism takes concerning all events as being produced not by something rational but by physical nature, i.e, that which is non-rational/non-thinking.

    • Thankyou for your reply Eric.

      I am confused by the confusing language used, I am not trying to move around the argument at all. I just am struggling to understand the gobbledegook used to present the argument.

      Thank you for explaining it so simply for me.It would be nice to see definitions with some of these arguments.What exactly is a good reason, as opposed to a bad one? I am not so sure that one must have good reasons for the beliefs they hold. They could be motivated by their desire for things to be a certain way. People speak of these things. “If everybody was like that, the world would be a better place”. Would it, really? The guy’s a simpleton!

      You write,”The entire process of thinking – or as you call it the “dynamic process” of a subjective observer – is governed by and explicable in terms of the ordinary laws of physics and chemistry.”

      Laws in science are just human ape concepts that are our best understanding of what we observe. We are still struggling to agree on a theory for gravity, and we do not have a clue as to how certain things work; we don’t even have a clue as to what we are clueless about.

      Neuroscientists are telling us that they have found that our brains have actually made the decision for us before we consciously make a decision. Here is a piece of one such study.

      http://exploringthemind.com/the-mind/brain-scans-can-reveal-your-decisions-7-seconds-before-you-decide

      If, and admittedly its a big IF at this stage, this is how our minds work, then the argument that one must have good reasons,(a cause, or explanation), for the beliefs they hold, or free will doesn’t wash.
      and then the religious will all become Calvinists, and the atheists will “believe’ that their unconscious minds are superior to the Calvivists unconscious minds 🙂

      Surfing around on the net, one finds studies confirming that our brains are ‘deciding’ things before we consciously register that we have made a decision, and also lots of evidence on consciousness emerges in studies done on brain damaged persons.

      Perhaps your argument should address those issues which could be a fly in the ointment.

      To speak of good and bad and truth and evil is meaningless. That is just what we individually, or as a group decide. Hitler dressed up millions of people in costumes and lead them to believe that they were good and right and opposing evil, and millions went off willingly to die for that great cause.

      I personally think we have very limited free will. We appear to have the ability to choose to absorb information or not. We appear to have the ability to choose to seek understanding of information or not. I don’t think that we choose our beliefs. I think that they stem from our best understanding of the information presented to us.

      There are definitely huge gaps in our understanding of how the world and indeed we humans function. I think that being a skeptic is the way to go. Whether one believes in materialism or dualism etc etc really doesn’t matter. We do not know enough, to know. So why become a believer?

      I don’t want to ask this but have to. Could this be a God of the gaps argument?

      What a pity mother nature never went to school and studied philosophy, it just is,it doesn’t care what we think
      ” Wittgensteinian way” I’ve never heard of him, but just reading the wiki thing looks like a man after my own heart.

      Once again thank you for taking time out with me on this.
      Looking forward to your reply.

  25. None of this drivel I have just read reveals truth to, or proof of, god’s existence. Aside from deliberately confusing a MULTITUDE of actual theory’s and facts regarding the position of Atheism, you religious types have STILL YET to account for a very large number of opposing doctrines which all claim their god is the proper god. Until the opposing doctrines can ALL agree on what god is definitely is THERE IS NO GOD DEBATE.

    This is a fundamental flaw in religious apology that continues to go unanswered by the legions of religion- how can you argue for the existence of God when you among yourselves cannot even agree what/who god is.

    Do not insult me with pretentious statements such as the opening of this blog- you sir, undermine your very position in your very poor attempt at reason. Your reason cannot be followed by any reasonable person. As a religious person you already, by definition, cannot be trusted as an authority for reason because the very basis of EVERY monotheistic religion is faith; faith itself does not require reason, if it did, it could no longer be called faith.

    • Den, thank you for your post. Before I respond in full, let me see if I understand your main atheist apologetic. Your argument is that ALL theists must agree on “what god is definitely” otherwise there is no need for debate. Does this accurately reflect your view?

      Also, if you could include THE single agreed upon atheistic position and/or philosophy, which would validate atheism according to your reasoning, it would be helpful.

      And, if time permits, examples and arguments regarding your many assertions would be great. Assertions are fine, they’re just not arguments. If you would like to show me what reason looks like maybe try to demonstrate it a bit in your response.

      Cheers.

    • I know this is off subject but try to read this in Planktons (from spongebob if you havent heard him look him up) voice when he’s mad, it’s pretty funny.

      As for the post I can agree on most of your things but the way you blatantly explained it wasn’t necessary at all, you attempted to make Christianity/Religion look like it was a joke written by children, that was totally uncalled for and rude.

      But it was worth the good laugh thinking of plankton narrating this, good day.

    • It is in the diversity of the concepts of “gods” where we can reasonably assume that divinity exists, for the desire to worship someone, or something began somewhere. There is a living God that we once had a relationship with and human kind, in their various ways, have attempted to return to Him.
      The reason there are many concept of God is because we as humans try to define Him instead of letting him define himself.
      Does “love” cease to exist because everyone has a different concept of what it means to be loved? Prove the existence of love.

  26. I would like to commend you on your intellect and argumens Eric. You are one of the best defenders of theism I have read.

  27. I agree with you article. I have thought about this from a slightly different angle lately.
    If one denies the existence of any spiritual, transcendent part of reality it becomes really difficult to defend a concept of free will in the sense that we truly have the capacity to shape the future. If the world follows fixed laws in a deterministic fashion there is only one possible future. What we experience as our ability to freely decide on a course of action is then just an illusion brought on by the complexity of our brains. Due to us being self-aware we would be observing our brains executing their programmed algorithms based on external input at the time and perceive this feedback as free decisions. Now some have argued that free will is indeed compatible with determinism, but I find this position untenable. With hard determinism the future is fixed, perhaps by invoking chance at the quantum level we may introduce some randomness, but in any case there is no way for us in this system to actively shape our future. We cannot choose a different future because no option for one exists. Every word I type now is fixed, every thought predetermined, every feeling and emotion just an ongoing chemical reaction and neurons firing. What we are and what we do would be a consequence of genetics and our past experiences. Like a chess computer that makes the same move every time in a given position so would we. In fact, if we could build a more complex computer system with a larger capacity and faster processing speed than our brains we would have created the new apex of evolution. I take that back, in this line of thought humans creating anything in the true sense of the word is just an illusion, of course, because the future is predetermined.
    So if an atheist asks me for proof that part of me is transcendental, or in other words, that I was given a soul, I like to ask if he or she is ready to believe in the alternative. I my opinion, one would have to admit that free choices are an illusion or provide an explanation as to how they can be compatible with determinism. I have not heard a satisfying answer.

  28. @ CatholicScientist,

    You write,”So if an atheist asks me for proof that part of me is transcendental, or in other words, that I was given a soul, I like to ask if he or she is ready to believe in the alternative.”
    Why only belief in “the” as in singular, one alternative? In fact why any belief at all? Why can someone not suspend their judgement until we know more and are able to come up with a rational reasonable explanation?

    Determinism is not foreign to theology. Augustine did not believe in free will, he did mumble on about us having some limited form of choice.Calvin, was a hard determinist, stating definitely that man had no free will. Luther, did not believe in free will, but in his predestination, There are Christian theologians that have grappled with the problem of free will putting too tighter limit on God’s omnipotence. So here we have Christians believing in “the alternative” as you put it, and a soul.

    This argument against atheism fails because it is not an argument against atheism at all, but rather one against determinism. The atheist or agnostic does in no way HAVE TO make a choice between free will,( as is associated with religion), or determinism,(which is usually associated with moral permissiveness by theists).

    There may well be compatibility between free will and determinism. We are only just scratching the surface on understanding how the brain works. Here is an aforementioned reference showing that decisions are made by our brain before we consciously make a decision.

    http://exploringthemind.com/the-mind/brain-scans-can-reveal-your-decisions-7-seconds-before-you-decide

    If our brains do in fact work just as this experiment shows. How does this fit in with your ‘free will’ ?

    I do not know what the big deal is. Until we know more, I am quite happy with what appears to be a causal universe and certainly enough free will to make me think that I have it.

    You state ,” If the world follows fixed laws in a deterministic fashion……..”
    Well it certainly does appear to do so, however we have chaos theory etc on a, shall i dare say, micro level. But we do not know enough to say that the world definitely does ALWAYS follow fixed laws that work in a deterministic fashion, so called laws of science are just our best understanding of how things work.Mother nature never took a physics class or studied philosophy, she just is, and we have a hard task trying to understand her.

    CathiolicScientist I think that the burden is still on you to explain to us what a soul exactly is; How you know you have one;Where exactly it is located and its purpose, nature etc.

    This idea of yours that :determinism sucks because it makes no sense, therefore so does atheism, therefore my soul and God do exist…………….does not make a very good argument at all.

    • If I may butt in, I followed the link, read the original source of the experiment, and have actually been following this stuff for at least the last year or so and I still don’t find it very challenging in the sense of it being a proof against freewill. The conscious and unconscious aspects of our mind is, of course, nothing new. Unconscious has always been understood as something irrational, something more along the lines of pure emotion as C. Jung put it. It is a part of ourselves which contains both personal and universal aspects which was highly influenced in our childhood and goes to the core of who we are. We can never “get rid” of the unconscious material but we can alter it, which is at the heart of the entire project of psychoanalysis. That there are processes influencing our decision on an unconscious level before we come into conscious awareness of it is nothing that we needed this series of experiments to demonstrate. This has been know for at least 100 years now.

      What is relatively new (minus Freud’s very early deep commitment to philosophic naturalism applied to the human psyche) is the conquest to prove that the mind is a mere aspect of the brain, something encoded in brain activity – that there is a one-to-one correspondence between mind and physical matter (i.e., neural structures).

      That the mind is “realized” by the human subject through brain activity is true, but that the mind has its pure origin in the brain is something far from being demonstrated. In fact, in the closing discussion of the research paper which the article in the link is referencing the question is raised as to what is the neural origin of the information contained in the predictive spatial patters. The paper goes on to hypothesize what it might be but concludes that the research has not addressed the question.

      Here is the logical problem with the whole notion of one-to-one correspondence of mind and matter, and the easiest way I can show it is in a series of two assertions and a question:

      1. Amy believes in xyz because this belief is based on good reasons.
      2. Amy believes in xyz because of an antecedent physical condition which caused the belief.

      Which statement is true according to a thorough naturalism?

      In a physicalistic world, principles of sound reason have no relevance to determining what actually happens. Said differently, according to a mechanistic closed-causal universe the mental properties of an event are irrelevant to its causal influence.

      Based on this reasoning Amy would believe what she believed based wholly on antecedent physical conditions and not on reason. Since we know manifestly that beliefs are based on reason, and the actions that flow from those belief, then something is still a bit mysterious about the human being and his logical faculties as far as physicalism is concerned. Either that or there is no such thing as reason, and if that is the case then there is no such thing as science, since science requires that beliefs be produced and sustained by a reliable belief-forming process.

      I like how William Hasker puts it: “Whether or not a given event has a mental description… seems entirely irrelevant to what casual relations it enters into. Its causal powers are wholly determined by the physical descriptions or characteristic that holds for it; for it is under its physical description that it may be subsumed under a causal law.”

      He goes on to utterly decimate this philosophy in his book The Emergent Self, one of my favorites.

    • Thanks for your response Sean.
      I’d like to reply with a few thoughts for now (not much time today):
      ” In fact why any belief at all? Why can someone not suspend their judgement until we know more and are able to come up with a rational reasonable explanation?”
      You could, I suppose. However, I’m afraid that would mean suspending judgement indefinitely on a lot of very important matters. Do you believe that by using the scientific method we will be able to fully explain reality at some point?
      “Determinism is not foreign to theology” I cannot speak for the theologians you listed. I’m familiar with the teachings of the Catholic Church. (http://www.vatican.va/archive/ENG0015/__P5M.HTM and following paragraphs). No determinism there.
      “This argument against atheism fails” I did not make an argument against atheism. I tried to point out some consequences of denying transcendence as I see them. Some people may be able to live with these consequences.
      “There may well be compatibility between free will and determinism” Yes, some people argue that is the case, for example Daniel Dennett. As far as I understand it, it requires some form of random generator function in our brains. Like seeding an algorithm with a first random step. I don’t see how this actually provides any space for free will, but admittedly I’m not an expert in this area.Maybe someone who understands this better can explain this.
      “Mother nature never took a physics class or studied philosophy, she just is, and we have a hard task trying to understand her.”
      Who is this “Mother nature” you write of in such a personal way? I have noticed that people often ascribe personal or even divine properties to the created universe. This actually sounds like a veiled theist position to me with the divine being manifested in nature (Immanence). Did I misunderstand this?
      “This idea of yours that :determinism sucks because it makes no sense, therefore so does atheism, therefore my soul and God do exist…………….does not make a very good argument at all.”
      I agree. I did not make that argument. I do not think there is proof for God in a scientific sense. If there was, I believe It would actually take away our free choice for or against God. I think it is reasonable to believe in God and there is evidence for transcendence, but no hard proof. But again, this has been expressed better elsewhere: http://www.vatican.va/archive/ENG0015/__PA.HTM
      (Eric, I hope it is ok to post these links here. I’m new to this excellent blog and don’t know the house rules 🙂 )

  29. I really enjoyed the topic discussed here. I even read most of the comments (did not have so much to do today, lol).

    Eric, this was a good article. Keep up the good work!

    God bless.

  30. I take it you have not read the selfish gene by Richard Dawkins? This concurs with your argument but leads us even further away from God. We are indeed acting on the “orders” of our genes to ensure their survival, not our own which as you say, is impossible.

    • Hi Mike, thanks for the post. I’m well familiar with Dawkins selfish gene theory. I’d highly recommend W. Hasker’s “Emergent Self,” T, Nagel’s “Mind and Cosmos,” or D. B, Hart’s “God,” to hear a thorough refutation of Dawkins on this point.

  31. I was running through different responses to your blog and, maybe I missed it, but I had a reasoning that I have not yet heard anyone else submit.
    While I understand skepticism from those who are not of a mind set to believe in a god that is not a quantifiable reason to pronounce that there IS no god.
    I was raised as a christian (loosely) and I can understand how people who are not strong in a faith can simply scoff and look at those persons who are forthright and true to their religion as fools who bow to the air and sing praises to nothing
    As a non-christian looking from the outside in the behavior of the church can appear to be very hypocritical with so many falling to the very sin that they purport to loathe.
    I myself saw firsthand how people, who call themselves Christians, don’t always act as a christian when everyone is not looking, It jaded me somewhat, but that does not mean there is no case for god.
    My concern is that the Bible was written by men over many many years and then re-written many times over.
    God is trust worthy and never fails. Mankind does not have those characteristics in great abundance either then or now.
    When compared to the image that we know as god we fall very short indeed.
    My point to this is, to equate god with a book that a weak and faulty mankind crafted at his behest or a religion (pick one) that states” do as I say not as I do” is not logical and in my opinion foolish at worst.
    Suppose a thousand years ago I show up in a Mayan city and I bring along with me a hover board and show the Mayans what great powers I have. Along with other things that would amaze such ancients. It would be interesting in our time, but it would be amazing and a feat of awe and wonder to those who do not yet possess the science or the understanding to comprehend what they were seeing.
    The case can be made that we simply can’t comprehend god as he is.
    We can’t believe a being can do all that the bible says he has done in the time allowed or over billions of years either.
    Is it possible that we:were, are, and will always be his young children never totally understanding?
    I suppose it’s possible, but just completely disallowing the idea of god as something larger than us is foolish simply because we may well be primitive and can’t prove he exists.
    It is true that, at this moment in time, we cannot prove definitively that god does exist. We cannot prove otherwise either.
    And now on a parting note for those who are simply just god haters or too blinded by science fact that they refuse to believe in alternate theories of how the universe began as seen through a faith based vision,if there is nothing after death then there is NO point to life as I will explain in my estimation.
    It has been surmised that this planet will die eventually billions of years from now. Even if we were to assume that the human race would make it that far into the future , what would happen to us? No matter if we do make it, science has theorized as that as the universe is expanding it will eventually contract in on itself. All our science, all our learning, loving, thinking, playing, building, for generation after generation will CEASE to exist and there is no escaping the universe.
    If we are indeed to pass on to future generations our abundant wealth of knowledge where will it go or be used when the universe becomes nothing once again?
    It seems pointless to me that constant progress as a means for mankind to better itself will sustain and move on even though it all ends.
    Does this not make a very good case for one life to end and then another begins as death may well create life elsewhere?
    Maybe the bible gets it all wrong. Maybe it gets it partly wrong.
    Maybe if god does really exist (and I believe god does) then he has patience, infinite patience at what fools we can be and how far we strayed from who god is and what was originally planned for us.
    Living only to know that you end, your loved ones end, it all ends eventually, seems to be a bleak exercise in futility.
    Everything exists for a reason.
    We all are here for a short time, but I believe it’s for a REASON.
    We are not simply some complex organic system that evolved here to live and die with no meaning behind it.
    We exist, we live.
    We love.
    We are loved.

    scott

  32. Hi Scott,

    Thanks for your post. I too was heavily jaded at points in my journey for reasons similar to what you gave. The peak of my jadedness landed me in the Orthodox Church. When I converted to it from the Evangelical world it was either Orthodoxy or bust. I figured if the Orthodox Church is not the real Church then its all a parlor trick.

    Anyway, I’m curious about this part: “My concern is that the Bible was written by men over many many years and then re-written many times over. God is trust worthy and never fails. Mankind does not have those characteristics in great abundance either then or now….My point to this is, to equate god with a book that a weak and faulty mankind crafted at his behest or a religion (pick one) that states” do as I say not as I do” is not logical and in my opinion foolish at worst.”

    Of course the Bible was written by men but it was written by men inspired of the Holy Spirit. That is a statement of faith, take it or leave it. But even so that doesn’t mean that it is flawless. The Bible is like Christ: all God and all man at once. Christ was flawless yet His flesh was subject to corruption just as ours is. To demand that a text be wholly divine is ridiculous, and yes many Christians treat it as such. Many Christians treat the Bible as Muslims treat the Koran, but they shouldn’t, and the historic Church didn’t. Also, I’d be slow to hold the idea that the Bible was re-written many times. Such a position simply doesn’t hold up when one begins studying the writings of the Church Fathers stretching back to the 2nd century. Their quotation of Scripture is a near perfect rendering of the text we use today almost 20 centuries later. The Church Fathers quoted enough Scripture that one could almost reconstruct the Bible word for word if all copies of it were to suddenly disappeared. There is no book in existence that remotely compares to the Bible in its ancient composition and its centuries of accurate copying and translations. Just thought that was an important point to tackle.

    Cheers!

  33. Believer: “God exists!”
    Atheist: “Can you prove that?”
    Believer: “No.”
    Atheist: “Then I don’t believe you” <- This is atheism. This doesn't require faith. This is not a religion. This is not a belief. This is not a claim.

    Thats the fundamental problem that most theists dont understand. Atheism is something you dont do. Its a useless word. We dont have a word for people that dont play golf or dont collect stamps. Atheism is exactly like that. But the article adresses atheism the whole time like its some kind of counterclaim to theism or somehow connected to science. Its not. If the bible would have gotten anything about reality right, science and religion would be one and the same. But they are completely different/opposites. Science also cant adress the supernatural because there is no method in existance, neither scientific or unscientific, to measure or detect any supernatural force which is the foundation for religions like christianity.

    "Also, I’d be slow to hold the idea that the Bible was re-written many times. Such a position simply doesn’t hold up when one begins studying the writings of the Church Fathers stretching back to the 2nd century."

    Actually you dont have original texts from that period to study and the current bibles are evidently edited over and over again. If you had, you would not need different versions of the same book. The people in the bible didnt have a bible. It was written decades and sometimes centuries after the events supposedly happened. Not to forget that there isnt a single verse written by jesus himself who is still controversial when it comes to his existance who cannot have existed because we know today that genesis is a pure fabrication written by people that werent there to begin with.

    " There is no book in existence that remotely compares to the Bible in its ancient composition and its centuries of accurate copying and translations."

    Right, there is no other book like it. There is no book as contradicting, inaccurate, self-refuting and more edited as the bible. It changed so many times in the last 1.000 years which explains why there are different versions to cherry pick from. It created over 50.000 different denominations because its so inaccurate in design as every denomination interprets (!!) verses differently. Everything in the bible represents the barbaric centuries in which it was written and edited, nothing shows a superior intellect or knowledge that mankind didnt figure out on its own, nothing about science, the cosmos itself or even a credible source.

    Its a collection of superstitious claims, contradicting accounts from different authors, made up stories and alot of hate. It promotes rape, genocide, murder, basically every immoral action you can think of. If thats the best a supreme being can do, I am not impressed. But its exactly what you expect from ignorant people that use fear of eternal torture and brutal armies to convince gullible and ignorant people to fall for all the false promised hope.

    And the best part is: The bible is the claim, not the evidence. Claiming the bible is right because it says so in the bible is circular reasoning. Nobody has to disprove it either. Theists simply fail to demonstrate ANY supernatural force and without that, the whole superstitious bible falls apart completely.

    We dont know everything about everything. We dont know everything about anything. But even tho we dont know absolutely everything, we do know some things absolutely. The bible and all the religions founded on that mythology are evidently absolutely wrong and physicaly impossible and logically inconsistant internaly. So we can honestly say THAT god does not exist. *Aron Ra

  34. I’m having a hard time following your argument. Isn’t an ant a “free agent” in the sense that it makes decisions freely, subject of course to far more limitations, since an ant has 250,000 brain cells compared to our 86,000,000,000?

    You may argue that an ant’s brain is so simple that its behavior is akin to a computer running a program, and it doesn’t really have thoughts or consciousness, and I’d probably agree with that since computer simulations of the brains of very simple creatures have already been created. But that further reinforces the point I’m making here.

    My dog certainly seems to have thoughts and free agency. My dog’s brain, although far less complex than mine, is far more advanced than an ant’s brain.

    An ant may not possess any consciousness or thoughts in the sense that we do, but a dog certainly does. This suggests that being human, having thoughts and free agency doesn’t reveal some sort of deep fundamental philosophical crisis about the need for the supernatural, but it’s simply a useful emergent property of an advanced brain. Consciousness, deep thoughts, nuanced decision making, etc., are simply evolutionary advantages that mammals, especially humans, have over less complex creatures.

    It doesn’t mean that whatever laws of physics make the ant brain work are different than the ones that make our brains work. Just because far more complex actions can emerge from a human brain than an ant brain, that is due to the complexity of the human brain, not because it possesses some magical properties.

    Ants, dogs, and humans can all make unpredictable decisions, just like the Jeopardy champion software program (Watson) written by IBM would find answers to things in ways that their creators did not expect. In a sense, the software has “free agency” because it will behave in ways the designers don’t expect. It just doesn’t possess a consciousness like we do. I’m a software engineer and it happens to me all the time. The more complicated a program I write is, the more ways it will behave in ways I did not expect, and IBM’s Watson is an unbelievably complex system. And it’s still orders of magnitude less complex than the human mind.

    I guess you lost me right around here:

    But any thought which is not guided by what is “true” but guided rather by mechanistic, physical necessity is not rational.

    Even the simplest life forms will behave rationally, including life forms without any brains at all. Bacteria make rational “decisions” all the time. They sense properties of their environment, and then react to those properties in ways that are beneficial to their survival and reproduction.

    So we see that you life forms don’t even need thoughts at all to take rational actions. And we see that complex systems without consciousness can behave in ways that mimic free agency even though it’s a computer program.

    Beliefs are true if they are supported by evidence. Since beliefs in the supernatural are not falsifiable, they can’t be categorized as true or false, unless they include aspects of the natural world. What does a belief being true or not have to do with thoughts? Thoughts don’t make beliefs true or not true.

  35. If this is “not an argument *for* Christianity, but rather an argument against atheism” – if so, then it should work equally well for discounting atheism in favor of the tooth fairy , unicorns and elves. Sorry, it does not.

    • Hi veggiedude. I’m not sure I understand. Are you saying that tooth fairies, unicorns and elves are not entities but rather metaphysical philosophies? Seems you’re making a categorical mistake.

    • “Are you saying that tooth fairies, unicorns and elves are not entities but rather metaphysical philosophies? Seems you’re making a categorical mistake.”

      One persons entity is another persons metaphysical philosophy and visa versa. No? Well here’s the finest argument against an all knowing, powerful and omnipotent god: if I were to meet your god one day (this is hypothetical) I could defeat it with one sentence. Yes, little mortal me, a puny human being could defeat your god with mere words! I would say “if you are the all knowing, all powerful, and omnipotent god of the universe – then create a mountain so strong and so powerful that even you, god of the entire universe, could not yourself move”. If god succeeds in building such a thing, that even he can’t move, how can be an all powerful god? He cannot move a mountain! But on the other hand, lets say since he is the all-powerful, all knowing and omnipotent god, yes, even he can move the most powerful mountain ever in creation… then I win and he has failed, because he could not create an all powerful mountain!

      😛

  36. Eric Van Bezooijen, Pilot whales have almost double the neocortical neurons that humans do. Are you prepared to say that pilot whales understand God better than theists do?

    “Are you saying that tooth fairies, unicorns and elves are not entities but rather metaphysical philosophies? Seems you’re making a categorical mistake.”
    They are all mythological/supernatural/superstitious creatures and gods are exactly the same. The only difference is the amount of power humans attribute to each of them. So evidence for all of them is exactly the same: books and bad arguments.

    • Thomas, Christianity is not God. It’s a religious understanding of God. Do unicorns and elves have doctrinal statements about God? If so, then yes they would be included in the variety of alternative choices one could make concerning God after abandoning non-belief.

  37. Hi Eric Van Bezooijen, thanks for the well articulated response. Let me start with a line or two from your post which I think is important to your overall point:

    “An ant may not possess any consciousness or thoughts in the sense that we do, but a dog certainly does. This suggests that being human, having thoughts and free agency doesn’t reveal some sort of deep fundamental philosophical crisis about the need for the supernatural, but it’s simply a useful emergent property of an advanced brain. Consciousness, deep thoughts, nuanced decision making, etc., are simply evolutionary advantages that mammals, especially humans, have over less complex creatures.

    It doesn’t mean that whatever laws of physics make the ant brain work are different than the ones that make our brains work. Just because far more complex actions can emerge from a human brain than an ant brain, that is due to the complexity of the human brain, not because it possesses some magical properties.”

    Let us assume that all this is correct. Our brains are no different than an ant or a dog save its complexity. This complexity creates emergent properties of the mind that surpass other animals by leaps. Okay. So far nothing has been said that tackles the issue in the article, though it supports your view that the brain/mind is merely a super computer. Where the problem lies is directly where you believe it doesn’t lie – that our conscious powers reveal to us “some sort of deep fundamental philosophical crisis about the need for the supernatural” exists. This is not something a dog or an ant deals with. We are different, VERY different. The ant does not ponder its own existential crisis, thus it has no need to worry about what power is generating his thinking – mere biology, or biology plus agency deriving from somewhere beyond biology.

    If our rational powers are “simply evolutionary advantages that mammals, especially humans, have over less complex creatures” then we could never claims such a view without hopeless circular reasoning. Here’s Thomas Nagel on this: “The judgment that our senses are reliable because their reliability contributes to fitness is legitimate, but the judgment that our reason is reliable because its reliability contributes to fitness is incoherent. That kind of judgment cannot itself depend on this kind of empirical confirmation without generating a regress: to make the judgment is necessarily to take it as having authority in its own right.”

    So, the problem is more complex than your allowable complexity allows for. Take an example very close to home – your own response to this article. Is it that you believe what you do on this subject because it is supported by sound reasons, or – big “or” – do you believe what you do on this subject because of such-and-such antecedent physical conditions?

    If you choose the latter you do need the former. If all events are fully explainable in physical terms, and if thoughts and beliefs are events, then there is no need at all to refer to “good reasons.” This is why philosophical materialism slits its own throat.

    If that is too confusing I’ll ask a question that I think is easier to get: If YOU did not write your post, who or what did? If you did write it, define what “you” is, i.e., what is your vision of what you are as an entity. If “you” is nothing more than molecules in motion why should I or anyone else take anything you say as relating to truth and not merely a parroting of whatever the total system would have you write at this time?

  38. “Christianity is not God. It’s a religious understanding of God.”

    And yet you wont find two christians that can agree on their gods attributes. 50.000 different denominations cant be all right, right?

    “Do unicorns and elves have doctrinal statements about God? If so, then yes they would be included in the variety of alternative choices one could make concerning God after abandoning non-belief.”

    Thats makes no sense at all.

    “let me also point you to this article, specifically argument #4 concerning the conflation of God with said fairy tales”

    This one?
    “4. Believing in God is the same as believing in the Tooth Fairy, Santa Clause, and the Flying Spaghetti Monster.

    What I love about this well-worn atheist ‘argument’ is that it actually serves to demonstrate how vastly different a belief in God is to these myths and imaginations. When one honestly assesses the Judeo-Christian doctrine of God he will find multiple thousands of years of human testimony and religious development; he will find martyrs enduring the most horrific trauma in defense of the faith; he will find accounts in religious texts with historical and geographical corroboration; etc (these fact are of course not ‘proofs,’ but rather ‘evidences’ that elicit strong consideration). Pit this against tales of the Tooth Fairy, Santa, and Spaghetti Monsters and one finds the exact opposite: no testimony or religious refinement, no martyrs, no historical and geographical corroboration, etc. Instead, one finds myths created intentionally for children, for point making, or for whatever. It’s strawman argumentation at its worst.

    Again, just to be clear, testimony, martyrs, geography, etc., are not “proof” that God exists, but rather proof that comparing faith in God to faith in fairies and Santa is totally different.”

    Yeah, using the bible to prove the biblical claims are accurate or factual only weakens your point. Circular reasoning fallacy shoots you right in the knee. And the claims of tooth fairies, unicorns and other creatures like that also come from books. If you are using the bible to validate your claims, Im using harry potter books to prove that harry potter exists. Just to show you how silly your argument is.

  39. To Zasz above (Thomas?)

    Thank you for your post. You dedicated a lot of thought and time to it so let me attempt to do the same. I’ll quote what you wrote then reply:

    “Believer: “God exists!”
    Atheist: “Can you prove that?”
    Believer: “No.”
    Atheist: “Then I don’t believe you” <- This is atheism. This doesn't require faith. This is not a religion. This is not a belief. This is not a claim."

    But Zasz, can you "prove" everything you "know" to be real? Can you prove that good and evil are real things and not mythical mental constructs? If you've already crossed that bridge then can you prove your own belief about how the universe came into existence and what sustains its being?

    Then:

    "Thats the fundamental problem that most theists dont understand. Atheism is something you dont do. Its a useless word. We dont have a word for people that dont play golf or dont collect stamps. Atheism is exactly like that."

    Well not quite. If nearly every person in every culture in every period of human history were golfers and all of the sudden they met a group of non-golfers you better believe they'ed be called something. Maybe "Agolfists".

    Then:

    "If the bible would have gotten anything about reality right, science and religion would be one and the same. But they are completely different/opposites. Science also cant adress the supernatural because there is no method in existance, neither scientific or unscientific, to measure or detect any supernatural force which is the foundation for religions like christianity."

    How would material tools go about measuring that which created and sustains it? Whatever brought the material universe into existence surely wasn't material for the simple fact that whatever is material already exists as material, hence it would have only brought itself into existence which is a logical absurdity.

    And, are science and religion "opposites"? Seems pretty bold statement. I see them as describing the same reality from different angles and for different reasons/agendas. Religion describe reality from the inside and science from the outside. It goes without saying that there is a marked difference between observing something from the outside versus from the inside. As Gordon Allport said, “To stand wholly outside the phenomenon is to understand it less well than by entering in.” No one ever "entered into" life via scientific objective distancing.

    Then, you quote me:

    "Also, I’d be slow to hold the idea that the Bible was re-written many times. Such a position simply doesn’t hold up when one begins studying the writings of the Church Fathers stretching back to the 2nd century."

    Then respond:

    "Actually you dont have original texts from that period to study and the current bibles are evidently edited over and over again. If you had, you would not need different versions of the same book. The people in the bible didnt have a bible. It was written decades and sometimes centuries after the events supposedly happened. Not to forget that there isnt a single verse written by jesus himself who is still controversial when it comes to his existance who cannot have existed because we know today that genesis is a pure fabrication written by people that werent there to begin with."

    There's a lot to unpack here and not sure where to start. My original point was that most of the Bible could be reconstructed based just off the quotations of the early Church fathers, many of whom wrote in the first few centuries after the resurrection of Christ. These works correspond exactly with the texts we have today (I'm speaking mostly of the LXX). To address the rest of your point would require a lot of time, particularly if you are the type who argues that Jesus didn't exist. And why his existence is proven wrong by the assumption that Genesis is fabrication is difficult to track with.

    So then you quote me:

    " There is no book in existence that remotely compares to the Bible in its ancient composition and its centuries of accurate copying and translations."

    Then this:

    "Right, there is no other book like it. There is no book as contradicting, inaccurate, self-refuting and more edited as the bible. It changed so many times in the last 1.000 years which explains why there are different versions to cherry pick from. It created over 50.000 different denominations because its so inaccurate in design as every denomination interprets (!!) verses differently. Everything in the bible represents the barbaric centuries in which it was written and edited, nothing shows a superior intellect or knowledge that mankind didnt figure out on its own, nothing about science, the cosmos itself or even a credible source."

    It would be fun to go through whatever examples you have to substantiate your claims above. I assure you it would not be fun because I'd never heard them before – I assure you I've heard them all – but because I could help you – someone who has clearly spend a lot of time dealing with this topic – to form some clarity on the Bible and the history of the Church from an Orthodox Christian perspective, which it sounds like you've not yet had. But yes, if the Bible were all there was to the Christian faith, as many Protestants would have you believe, much of what you say, well, at least the latter half, would hold significant weight. Everything you comment on have many unexpected layers of understanding that must be crossed before you can speak to it intelligently. It's sort of like if I were to kick in the doors of a astrophysics classroom and try to school the professor based on my own YouTube and Facebook exposure I've had on the subject. Until I dedicate myself to understanding astrophysics from those who have spend their lives invested in it, I'm going nowhere fast.

    Then:

    "Its a collection of superstitious claims, contradicting accounts from different authors, made up stories and alot of hate. It promotes rape, genocide, murder, basically every immoral action you can think of. If thats the best a supreme being can do, I am not impressed. But its exactly what you expect from ignorant people that use fear of eternal torture and brutal armies to convince gullible and ignorant people to fall for all the false promised hope."

    Yah, this will take some time. But you're right, there are a lot of made up stories. Made up for the purposes of teaching higher spiritual truths. Take for instance when king David says that one should kill the the Philistine while still a baby by "dashing it against the wall." The Church has always understood this as teaching that one should kill "sin" while it is still in its infancy before it grows up into an enemy that can not be overcome.

    Then:

    "And the best part is: The bible is the claim, not the evidence. Claiming the bible is right because it says so in the bible is circular reasoning. Nobody has to disprove it either. Theists simply fail to demonstrate ANY supernatural force and without that, the whole superstitious bible falls apart completely."

    You must be speaking of something said somewhere else because I've never made the circular argument that it's true because its in the Bible. But if supernatural force is what you're wanting demonstrated why not start with something simple like love and goodness? You take these for granted but what relation in any way shape or form do these have with mere physicality? If they do not belong to the physical material realm then why to we believe so deeply in them?

    Then:

    "We dont know everything about everything. We dont know everything about anything. But even tho we dont know absolutely everything, we do know some things absolutely. The bible and all the religions founded on that mythology are evidently absolutely wrong and physicaly impossible and logically inconsistant internaly. So we can honestly say THAT god does not exist. *Aron Ra"

    You'll have to provide some kind of evidence for this otherwise its just an assertion. I can give you an immediate demonstration why philosophical materialism is internally inconsistent and "absolutely wrong." But you've already read my article. 🙂

  40. Thomas Zasz Zert,

    You said: “And yet you wont find two christians that can agree on their gods attributes. 50.000 different denominations cant be all right, right?”

    Sounds like your only reference for Christianity is Protestantism. I can find you 10’s of millions of Orthodox Christians who have almost perfect agreement on God. Google the Nicene Creed.

    And then this in reaction to my other article:

    “Yeah, using the bible to prove the biblical claims are accurate or factual only weakens your point. Circular reasoning fallacy shoots you right in the knee. And the claims of tooth fairies, unicorns and other creatures like that also come from books. If you are using the bible to validate your claims, Im using harry potter books to prove that harry potter exists. Just to show you how silly your argument is.”

    Wow, I’m speechless. Did you really misunderstand the point to that degree?

  41. “But Zasz, can you “prove” everything you “know” to be real? Can you prove that good and evil are real things and not mythical mental constructs?”
    If we can touch or measure it then its pretty much real, dont you think? Good and evil are constructs that we use to explain actions. Actions are very real. Words are not real in the same sense.

    “If you’ve already crossed that bridge then can you prove your own belief about how the universe came into existence and what sustains its being?”
    If I knew how the universe came to be, I would know it. No belief necessary.

    “Well not quite. If nearly every person in every culture in every period of human history were golfers and all of the sudden they met a group of non-golfers you better believe they’ed be called something. Maybe “Agolfists”. ”
    Odd that muslims refer to non-theists as unbelievers which is a much more accurate term while christians use atheist and cant define the term correctly most of the time.

    “How would material tools go about measuring that which created and sustains it?”
    Do you have non-material tools to measure a non-physical thing? You make the mistake to assume that it was created by an agent who then constantly has to “fine tune” it who should be capable of creating a perfect reality but utterly fails at it. Thats the cosmological argument you are thinking of, arent you? Its just an assumption, nothing more. Its even refutes itself because it violates its own premises. The universe as we currently know it doesnt require to sustain itself. It just is.

    “And, are science and religion “opposites”? Seems pretty bold statement. I see them as describing the same reality from different angles and for different reasons/agendas”
    So far we only discovered natural causes for everything. Religion only claims supernatural causes but is unable to show anything. So yes, they are opposites.

    “My original point was that most of the Bible could be reconstructed based just off the quotations of the early Church fathers, many of whom wrote in the first few centuries after the resurrection of Christ. These works correspond exactly with the texts we have today”
    The people from that time period have not a single historical document describing those events. The only collection of claims of those events are in the bible. The first texts claim to be from many decades after the events and many are just hearsay from people that didnt witness the events. Also we know from historians that many texts were assembled nearly 1.000 years ago to completely rewrite most of the bible. Some letters were even included that talk about events from 1.000 years ago, not 2.000 years ago. And there are even fabrications.
    Nothing outside the bible confirms the bible. Not a single historical text. The bible is not historical, its religious made up texts.

    “And why his existence is proven wrong by the assumption that Genesis is fabrication is difficult to track with. ”
    To make it short:
    Genetics has proven the adam & eve story in genesis to be impossible. Without the adam & eve story, there would be no original sin. Without original sin, there would be no need for jesus. Without jesus, the christian religion falls apart completely. The whole religion is based on and relies upon a story that could not be true. All evidence points to this fact. Game over.

    “I assure you it would not be fun because I’d never heard them before”
    Yeah, most christians dont care about it. Its called being ignorant. They dont know therefor it cant be true that the bible consists of over 150 contradictions. Just an example:
    God cannot lie
    Heb 6:18
    God lies by proxy; he sends forth lying spirits to deceive
    2 Thes 2:11/ 1 Kings 22:23/ Ezek 14:9

    The full list is easy to google.

    “Everything you comment on have many unexpected layers of understanding that must be crossed before you can speak to it intelligently. It’s sort of like if I were to kick in the doors of a astrophysics classroom and try to school the professor based on my own YouTube and Facebook exposure I’ve had on the subject. Until I dedicate myself to understanding astrophysics from those who have spend their lives invested in it, I’m going nowhere fast.”
    Isnt that exactly what you are doing here? You dont have a degree in history, philosophy, any scientific field or even theology and yet you claim so many things that others with the same degrees tried before and all ended up being ridiculed for using the same old arguments and fallacies filled with magic and nonsense. And they all sold their books on amazon crying why scientists refuse to accept it while atheists constantly have to explain what the bible says to christians that hardly read their own bible.
    Ive studied more of the bible than the overwhelming majority of christians.

    “Yah, this will take some time. But you’re right, there are a lot of made up stories. Made up for the purposes of teaching higher spiritual truths. Take for instance when king David says that one should kill the the Philistine while still a baby by “dashing it against the wall.” The Church has always understood this as teaching that one should kill “sin” while it is still in its infancy before it grows up into an enemy that can not be overcome.”
    You are just interpreting the verse in any way you want. Its a barbaric verse but for you its just a metaphor. Ive heard those excuses before. Its just dishonest and the reason why over 50.000 christian denominations exist.

    “You must be speaking of something said somewhere else because I’ve never made the circular argument that it’s true because its in the Bible.”
    You claim it as true because you constantly appealed to the bible because the bible is the only book making those claims. If you think its not true, why believe and defend it?

    “But if supernatural force is what you’re wanting demonstrated why not start with something simple like love and goodness? You take these for granted but what relation in any way shape or form do these have with mere physicality? If they do not belong to the physical material realm then why to we believe so deeply in them?”
    Emotions are not supernatural. They are physical, demonstrably. Dont you have something better?

    “You’ll have to provide some kind of evidence for this otherwise its just an assertion. I can give you an immediate demonstration why philosophical materialism is internally inconsistent and “absolutely wrong.” But you’ve already read my article”
    If you could prove that the supernatural exists, or any other theist, then you would have a case. Every theist failed. So why should I believe any of YOUR claims? People that fail to prove their claims are by default wrong.
    Yes, Ive read your article but the amount of misinformation is mindblowing. Since there are no methodes to detect any supernatural force, materialism is all we have. But you can always try to disprove anything you think is wrong if you know how to do that or prove a supernatural force and claim your nobel prize. The james randy challenge offers 1mil $ for that too.

    “Sounds like your only reference for Christianity is Protestantism. I can find you 10’s of millions of Orthodox Christians who have almost perfect agreement on God.”
    Christians are christians regardless if they call themself protestants, baptists, catholics or anything else. They all have the same book. If there is ONE god, there should be only ONE version of christianity. But 50.000 denominations who all claim to be correct complety destroy any argumentation about religious truth.

    “Wow, I’m speechless. Did you really misunderstand the point to that degree? ”
    If you have other material than the bible, feel free to point to it like a method to detect or measure the supernatural or a historical text that confirms biblical claims or the original texts that can be accurately dated to the first centuries that confirm whats written in the modern bible(s). So far every christian/theist fails at that.

  42. Thomas Zasz,

    Forgive me. I honestly mistook your first post as someone who was interested in learning about a topic he has very little understanding of. There are many things I learn from atheist on my blog about their take on reality, and it is often fascinating. From you I’ve only learned that you’ve definitely seen your share of Sam Harris YouTube videos. But I’d feel this was a complete waste if I didn’t send you off with at least a piece of advice. Please, please, please, the next blog thing you get into focus on backing up your assertions with something other than “this is the truth of the matter because I say so” approach. Find some evidence for your claims. Don’t just say it, demonstrate it (though you did note a few verses of Scripture to attempt to make a point about God lying). If nothing else demonstrate that you know something about the subject which you claim to have an authoritative voice on. What you know about this stuff may seem to you amazing, it may feel like you have the greatest rebuttal to religion at your finger tips, but for those who have spent half a lifetime studying it your show is one step below embarrassing. Do something very few atheist or Christians do on these forums – ask questions. Oh, and before you tell the person what sort of education he has, check his “About” page first. Good luck.

  43. I’ve skimmed the commentaries and it doesn’t appear that anyone has said “uncle” yet. No one’s opinions appear to have changed – arguments that are supposedly compelling and persuasive have failed to compel or persuade. And so we can question how rational humans are given that everyone presenting an argument believes it to be powerful and everyone that refutes any such argument finds their responses compelling.

    We are evolved creatures – that evolution would seemingly favour cognitive abilities that allow one to model one’s environment in a somewhat competent fashion and to make reasonably accurate predictions – and the weaker one is at those 2 goals, the less one is likely to survive. And so we have decent cognitive skills on par with what one would expect for an evolved entity – and we use scientific method to achieve rigour and system where our native cognitive abilities would normally fail to achieve greater predictive power. The test of whether or not we are succeeding or not is fairly straightforwards and simple – science works whether we believe in it or not.

    No one needs to say that we know with absolute certainty that we are capable of accurately determining the nature of our environment – we could be brains in a vat or something utterly unimaginable. We don’t get to tell the universe how to behave – at best, we’re limited to rational thinking which consists of a) observations (including those procured by the manipulation of our environment b) deductive reasoning c) inductive reasoning and d) abductive reasoning. There’s no plan B where irrational reasoning or logical inconsistencies leads to better predictions or descriptions of the universe. Faith for example is not a path to knowledge. If I have faith that there is no supernatural deity engaged in our universe and who created it and you hold the opposite faith belief, there is no vehicle or method to resolve our dispute or arriving at the truth. That’s kind of why we want to have the power to justify our beliefs by reference to observations and rational empirical methods of resolving disagreements.

    Deductive reasoning does not take one to knowledge of one’s environment by itself – in fact deductive reasoning is essentially vacuous by itself and consists of following various tautologies – we need axioms or assumptions or observations or evidence to take us any further. Inductive reasoning – which is essentially given the observation of a given pattern, expect the pattern to continue until it doesn’t. If the pattern succeeds, you win. If there is no viable pattern, you lose – and there was nothing you could do about that. Inductive reasoning is the identification of our best bet given that the future could change. Hume’s reasoning attacking the soundness of inductive reasoning is quite persuasive – and so we’re left with best guess. And abductive reasoning consists of taking all of our knowledge and beliefs and in the face of new evidence trying to work out the simplest and most compelling way of making them internally consistent and useful.

    There need be no assumption that the universe is explicable using scientific method – many of our great scientists have commented that it may well not be. Rather, the position one would take is that scientific method is our current best system and that scientific method may be be revised as need be should circumstances warrant and should it lead to better and more efficient models of the universe (which in large part comes down – can we use the results of scientific method in order to make precise predictions.

    There’s an old cartoon in which 2 physicists are standing in front of a cartoon chock full of equations and mathematical formulae – and in the middle it states “here, a miracle happens”. The point of the cartoon from my perspective is that “here, a miracle happens” completely vitiates everything that went before and came after and makes the explanation useless. Unless of course, so called miracles are totally predictable – in which case we probably wouldn’t call them miracles.

    Your argument begins by assuming that atheists necessarily rely upon naturalism and physicalism. Personally, I don’t find those 2 concepts particularly useful or helpful and I don’t subscribe to either doctrine in any committed way. Science does not assume naturalism. Scientists can cheerfully investigate ghosts and goblins and paranormal phenomena or the power of prayer to change reality without any particular ado or concerns – the problem is that these investigations pretty much invariably come up with the null result. If in the fullness of time, it turns out that we need a soul or some transcendent entity to explain the results of how the brain works or we confirm paranormal phenomena, so be it. No skin off my nose. Having said that, the likelihood would appear to be fairly low. We are made of atoms. When we investigate atoms they move and act in predictable ways at least in probabilistic terms – in study after study. atoms and electrons behave following patterns & these patterns repeat themselves and the properties of the quantum particles can be determined to the power of 10 ^-14 – so pretty incredible powers of prediction. Quantum physicists don’t talk about causal relationships so much – event A may give rise to event B 60% of the time, event C 20% of the time and event D 7% of the time and so on. Event B may give rise to event A 60% of the time, event C never and so on. An object can be in 2 places at once, it may be impossible to describe certain properties of a given particle w/o reference to a particle located at the other end of the universe and many other strange phenomena which leads me to the conclusion that the concept of physicalism is more harmful then helpful.

    so given millions of observations where atoms behave themselves in accordance with known physical laws, it would seem unlikely that atoms in our brains & our nervous system and our bodies act in a materially different manner. And yet if the atoms in our bodies pursue their merry ways in accordance with known scientific laws, then how does the ghost in the machine get to intervene? How does it communicate with the atoms? And if atoms behave entirely differently when in the brain, then… well, it’s interesting but it doesn’t imply a deity or a bunch of deities. So far, there is no evidence that atoms misbehave, but that’s the thing with scientific method – enjoy the process – enjoy the surprises.
    so looking at your argument

    1. No belief is rationally inferred if it can be fully explained in terms of non-rational causes. – so this is false – we can define rational inferences and we can see the consequences of making irrational inferences vs rational inferences. Perhaps we’re like trains going from A to B on a predetermined route – we (or some of us) are predetermined to make rational calculations and to modify our beliefs rationally in response to new circumstances. And just an off comment – to what extent are our beliefs formed behind the scene in our subconscious and what role, if any does consciousness play. We can understand based upon secular reasoning and abductive reasoning which we are clearly capable of, why we might evolve to undertake rational thinking processes. And we can easily reject the notions of causality and hard determinism & materialism and naturalism should circumstances warrant w/o leading us to deities.

    2. If materialism is true, then all beliefs can be fully explained in terms of non-rational causes. – see my response to 1

    3. Therefore, if materialism is true, then no belief is rationally inferred. this conclusion is false – see my response to 1

    4. If any thesis entails the conclusion that no belief is rationally inferred, then it should be rejected and its denial accepted. completely false – bald assertion – does not follow from 1 – 4 even if one were to grant your premises. Should we reject the outputs of computers where the results are predetermined?

    5. Therefore materialism should be rejected and its denial accepted. – does not follow

  44. I hate to run on and on and again my previous post seems to have disappeared but for the sake of argument let me propose the following challenge or wager – just for the day you will exercise your free will in order to be strongly convinced of atheism. I get the satisfaction that just for the day, I’ve won that portion of the argument and you get the satisfaction of demonstrating your free will. It’s a win win. Feel free to suggest an appropriate wager.

  45. Hi David,

    Thanks for you posts. You’ve said quite a bit. If blogging was my full time job and I didn’t have kids I would comment on every point you raised, but at present there is no conceivable way. Could you narrow down your reply to perhaps one or two of your most important points and I would be glad to respond.

    Let me hit one or two points to set us on track with this article. I do not propose that philosophical materialism is the only route the atheist can take. It is simply the only route that I find to be fully consistent with a worldview wholly void of belief in the divine. Second, as noted in the article, this argument from reason is not an argument for Christianity. Lastly, I think you are misunderstanding Reppert’s syllogism, ironically I included it only because I thought it helped to present the argument from reason better than if I hadn’t included it. In plain language it is saying that if materialism is true then there is no human free-agency. If there is no agency then one is not making decisions and forming rational inferences but is rather a physical machine parroting whatever antecedent physical condition made his brain think. If this be the case then every form of truth seeking – most in particular scientific truth seeking – is dead. Science requires one to believe based on sound reasons. But if philosophic materialism has it right then whatever one believes can be fully explained in natural-physical terms (merely neurons firing based on natural cause and effect having nothing to do with personal agency, etc.). Thus, it would not be that one’s beliefs are held for sound reasons but rather one believes such-and-such because of such-and-such antecedent physical conditions, or, as someone like Sam Harris would claim, we think only what the total system determined we could think.

    Anyway, please, what are your top 1 or 2 concerns?

  46. Whether we have free will or not is a question of brute fact. If we are to form an opinion as to whether or not we have free will, that opinion should be based upon evidence and not based upon any supposed negative consequences or our personal preferences.

    “But any thought which is not guided by what is “true” but guided rather by mechanistic, physical necessity is not rational. ”

    ” if our thoughts are the inevitable play of firing neurons in our brain set in motion by causal necessity then what we think would be the result of whatever the total system delivered to us and not because it accorded with “truth” necessarily.”

    These 2 points seem to be key to your argument and both represent bald assertions as opposed to deductions from evidence based premises. If our brains are organized through our genetic structure and environmental influences to seek truth and make rational decisions (more or less – since clearly we’re not Vulcans) then both your claims are wrong. If the problem that you are proposing was genuine, adding some sort of spiritual adviser or internal homunculus who may or may not seek truth and who may be equally blinded does not seem an obvious or sufficient resolution of the problem that you are attempting to pose.

    “And if one arrives at his philosophy not because he chose it, but rather because it was all the total system would allow, then Naturalism is, philosophically, self-defeating.”

    and yet when I asked you to choose atheism for the day, you couldn’t do it (I strongly suspect). Not because you wouldn’t wish to but you don’t get to choose your beliefs. You don’t choose your philosophy – your free will does not extend to choosing your beliefs -rather your philosophy is a product of your genes and your experience. Your brain is quite capable of analyzing new experiences and observations in light of previous experience and your genetic heritage. On point, I think – there was an interesting experiment with separated twins each of whom was evaluated for religiosity. It turns out that somewhere between 40% and 50% of how religious a given individual was is genetically predetermined. In other words 2 genetically identical twins growing up in separate environments displayed religious preferences that were deeply correlated more than chance or their environments would indicate. If a deity were concerned about religiosity, it’s clear that such a deity has decided to set up a series of handicaps.

    “I like what the late Professor Haldane of Oxford University said concerning the logical conclusion of a strict naturalism: “If my mental processes are determined wholly by the motions of atoms in my brain, I have no reason to suppose that my beliefs are true”

    this conclusion does not follow, no matter how intuitively straightforwards it no doubt seems to you. If our beliefs weren’t true or at least trueish, nature would make short shrift of us. If we are in a sense conscious machines, then we are very sophisticated ones indeed operating of the thin edge of complexity and chaos. Nothing changes in our personal experience or how we relate to the world if it turns out that free will is but an illusion – we still get to ponder and deliberate and make choices- except that it represent an overt challenge to certain orthodox religious views.

    Let me ask the following questions. 1) is a computer capable of free will? 2) is a computer capable of making choices 3) is a computer capable of exercising rational decision making . I won’t ask whether or not computers are solely the product of natural processes or whether or not they are responsive to nothing other than their hardware, programming and input information as those questions would be rhetorical.

    I want to draw your attention to recent developments in the field of artificial intelligence – including the recent defeat of the world champion go player by a computer, a feat that scientists earlier had predicted would not occur for at least another quarter of a century. If you’re not familiar with go, it’s about 2000 years older than chess, the rules are much simpler and the strategy is much more complex. Unlike chess, where Kasparov was beaten by Deep Blue using massive computer power to examine all possible moves for quite a number of move sequences, the number of possible go games rapidly exceeds the number of atoms in the universe. Expert observers referred to Google Deep play as elegant and creative. You can watch the games on youtube if you’re interested.

    By the way, I thought that my initial post had already addressed a number of your responses and that your responses basically were a repeat of your initial post rather than addressed my concerns. I won’t make an issue of it – I just don’t want to see our discussion end with each of us simply repeating our positions w/o reference to the specifics of the other’s point of view.

    There’s a well known philosophical school of thought called “compatibilism” that seeks to reconcile hard determinism and free will – you may already be familiar with it but if not, there’s an article at the Stanford Encyclopedia of Philosophy.

    I wouldn’t agree that philosophical materialism is the only route consistent with a wholly void of belief in the divine – I don’t find that proposition at all plausible and I don’t know how you would make that claim stick give the range and number of possible worldviews which are absent a deity.

    Given your time constraints, would you prefer me to limit my responses and to seek and exchange of short punchy arguments, insofar as that’s possible?

    • Hello David Aiken,

      Seems as though my request for you top 1 or 2 questions proved more difficult than expected. But I know how it is. It’s tough to narrow things down when you’re passionate about a subject. Even though time is limited it’s a joy to share this space with you because I’m passionate about it as well.

      Let me jump in and see how much I can cover. You say that much of what I wrote in the article is “bald assertion” though the examples you gave are logically sound. I do not need to give evidence based premises for a priori truths. If all of one’s thoughts are merely the end result of such-and-such antecedent physical condition, and not the end result of solid reasoning, it follows that one’s thoughts do not accord with truth, except by coincidence.

      You however make a wild assertion in your very first line: “Whether we have free will or not is a question of brute fact.”

      To believe it is a matter of brute fact is an assertion based entirely on one’s philosophical worldview. If free will lent itself to being a matter of a brute fact, requiring no interpretive lens, the scientific, philosophic, psychological, and theological communities would shout it from the rooftops. No such shouting can be heard – I listen for it all the time.

      Science, particularly neuroscience, lends a hand in describing brain phenomenon, but has not leveled to debate over free will/free agency to a brute fact. Conceiving of the brain as a machine or computer is an idea that has been debunked from various angles for decades, though it does not stop many from believing. I’m too tired at present to quote a bunch of people and besides it would take 20 pages to do so, so I’ll relent for now unless you want it later. The short answer is that the mind-body debate needs to be taken into consideration which is not as neuroscience friendly as many would wish. This is the seat of the issue though, so I would expect this discussion to ensue if we were to banter any further.

      Positing the brain as merely a machine structured by genetics and environment is not helpful either once one considers how there is no law governing how exactly genetics and environment will play out in the individual. The “individual” is a total mystery to science when it comes to how it will think and how it will behave. Statistical groups are not mysterious – the individual is. The twins study is interesting. You wanted it to be a proof that one does not choose his philosophy of life, though individual’s have free decision power over their beliefs – even if their choices are limited – as anyone can demonstrate on their own individual basis (it is in fact the free agency involved in the individual’s “what shall I do next” or “how shall I think about this” conundrum that is the most existentially sound intuition one can have. To believe against one’s ability to freely choose is better understood as a mental illness). Let us pretend for the sake of argument that the study eradicated all possible confounding variables and the entire study actually produced reliable results. It reveals only 40-50% of such beliefs is genetically predetermined (and of course let’s not forget that correlation does not equal causation). I’m highly skeptical of these results as a trained psychotherapist who has seen 100’s of studies, many of which stretch their interpretive value to the max in order to satisfy a presupposition in their own theoretical orientation. Peer review boards are more than happy to sign off on such studies if they carry whatever banner happens to be in vogue that year. I’d be happy to see the actual study if you have it so I don’t have to speculate so much. But back to the point, materialism to really show it’s stuff would need far higher figures of genetic involvement to show biological determinism. Everything – everything – one believes would have to be reducible to physical matter on some level. DNA and external physical interruptions would literally have to answer for 100% of one’s beliefs.

      Btw, you called it. I did not attempt your challenge to decide to be an atheist for a day for the simple reason that my philosophy of life is based on deep and concentrated layers of supported reasons and experience with the divine. True I was not at liberty to pretend that I didn’t have solid reason for not being an atheist, but this is to the credit of me being a free agent, not the reverse. I think this test is damaging to your case.

      I like this statement as well: “If our beliefs weren’t true or at least trueish, nature would make short shrift of us.” Does it then follow that religion is at least “trueish” since it has been and continues to be the modus operandi of the vast majority of humanity since the beginning of human history? One could go so far as to say it is THE factor separating us from every other species and demonstrates our superior higher reasoning. Clearly nature must have chosen this orientation since it has been so wildly successful for us as a species. Hell, religion even gave us science.

      But seriously, the reason this line of thought is troublesome is because it creates a logical fallacy. Thomas Nagel explains: “The judgment that our reason is reliable because its reliability contributes to fitness is incoherent. That judgment cannot itself depend on this kind of empirical confirmation without generating a regress: to make the judgment is necessarily to take it as having authority in its own right.”

      I don’t want to leave out the part where you ask pointed questions so let me make sure to cover it. You said, “Let me ask the following questions. 1) is a computer capable of free will?”

      As I understand it, a computer is capable of algorithms and is constituted by them. A human can use algorithms, but is not constituted by them. So, a computer may appear to have agent causing abilities but it is merely performing whatever algorithm was assigned to it by an actual free agent (i.e., a human).

      “2) is a computer capable of making choices”

      See answer above.

      “3) is a computer capable of exercising rational decision making.”

      It is capable of computation. It is not capable of making decisions based on its personally held values, unconscious processes, fears, and teleological motivations (unless programed for end game results of course). Can it play games (such as beating world champion go players)? Sure.

      What else, what else, what else…

      To you second to last paragraph, yes, I agree. Remember, I wrote this article based on the alternative to theism that I thought most internally consistent – for the purposes of avoiding the creation of an intellectual halfway house that would insulate me from having to take my materialistic atheism to its final and logical conclusions. Thus, I chose a strict, eliminative materialism as my whipping post. Are there other possible avenues of thought? Of course.

      That’s all for now. Please feel free to respond and correct me wherever you feel necessary. I look forward to future discussion. Cheers!

  47. Hi Eric,
    Let’s look at this argument by examining the natural world around us.

    In an experiment where when presented with a peanut floating in a tube a quarter filled with water, some chimpanzees were able to figure out that they could raise the water level, and hence the peanut, by filling their mouths with water from a nearby dispenser, then spitting it into the tube. Doing so enough times, raised the floating peanut to such a level that they were eventually able to retrieve and eat it. Also, interestingly, one smart ass chimp actually resorted to urinating into the tube, (was this a solution the human ape researchers had anticipated?)

    Human ape children were given nearly the same test; though instead of having to spit water from their mouths, they were allowed a water pitcher which they could use to pour the water into the tube. In this study, three age groups were tested, 4, 6, and eight year olds. Not surprisingly, the youngest group fared quite poorly, 8% of the 4 year olds, while the oldest group outperformed the chimps, 58 % of the 8 year olds solved the problem vs 20% of the chimps.

    http://phys.org/news/2011-06-chimps-capable-insightful-ability.html

    In another experiment a banana was inserted into a pipe and various diameter rods of various lengths were given to chimps enabling them to attempt to push the fruit out, only one rod could do the job. Attempts were counted that were made. An older chimp examined the pipe and put his lips over the pipe and blew the fruit out, a solution not anticipated by the researchers.

    Chimps use reasoned thought when they process information and use their memory, for example when finding fruit according to what season it is. Chimps are capable of generalization and symbolic representation, as they are able to group symbols together, and some chimps have even learned how to use American Sign Language. Chimps also have a “concept of self”. An interesting test that is often used is to see if an animal recognizes themselves in mirrors – chimps can do this and use the mirror to examine places on their bodies that they ordinarily cannot see.

    My theist friends assure me that animals, including chimps do not have souls and whose behavior is deterministic, the word instinct also being used to describe their behavior. Yet when I look at what these animals are capable of doing, it appears that they have free will. I will define free will here as the freedom to make choices that are not determined by prior causes or by divine intervention.

    In your previous reply to me you simplified the argument from reason for me as thus;
    1. Amy believes in xyz because this belief is based on good reasons.
    2. Amy believes in xyz because of an antecedent physical condition which caused the belief.

    Given the chimp stories above, let’s state the following.
    Amy the 8 year old human believes she can get the nut from the tube because this belief is based on good reasons.
    Amy the chimp believes she can get the nut from the tube because of an antecedent physical condition which caused the belief, as she is just a chimp!

    The distinction between the two is not as apparent as it first appeared to me; in fact it all looks rather fuzzy now.

    However if materialism is indeed the case, there should be some physical differences between the brains of the 4 year old and older children and the chimps to explain the results of the study.
    There is such a study explaining these physical differences in young human brains vs adolescents and our close relatives the great apes.

    http://news.berkeley.edu/2014/12/03/reasoning-skills/

    During my research I came across an interesting idea by Stephen Cave, developing the idea of a FQ, where he writes, “…all around us, every day, we see a very natural kind of freedom – one that is completely compatible with determinism. It is the kind that living things need to pursue their goals in a world that continually presents them with multiple possibilities. Our intuitive sense that we have free will is based upon this behavioral freedom. And unlike the old mystical idea, this natural ability to shape our future is central to our own well being and that of society.”

    https://aeon.co/essays/free-will-is-back-and-maybe-this-time-we-can-measure-it

    It would be interesting to hear your thoughts on this subject.

    Kind regards
    Sean

  48. Hi Eric. I thought I had kept quite nicely to 1 or 2 issues – I just happened to be long winded in connection with those issues. :> In retrospect, perhaps you intended that I restrict myself to 1 or 2 issues in my response whereas I took your request to restrict myself to 1 or 2 issues that you had raised. I’d like to say I’ll do better but really, the temptation to address your posts in a reasonably full manner is likely to be overwhelming, unfortunately. I have however directed your attention to those paragraphs which are the main subject of discussion. My next post will be shorter.

    “You say that much of what I wrote in the article is “bald assertion” though the examples you gave are logically sound. I do not need to give evidence based premises for a priori truths. If all of one’s thoughts are merely the end result of such-and-such antecedent physical condition, and not the end result of solid reasoning, it follows that one’s thoughts do not accord with truth, except by coincidence.”

    It’s not so much that much of what you wrote in your article was a bald assertion so much as it was that this specific proposition repeated with different wording on a number of occasions that I considered to be a bald assertion – and one that for the most part simply assumed your ultimate conclusion. It’s the key issue and one that I don’t agree with. I don’t think it’s an a priori truth or logically sound and it’s this claim that is specifically the bald assertion that I’ve taken issue with and it’s a fundamental to your argument. If it fails, your argument fails.

    Firstly your argument or conclusion is not something that can be arrived at a priori – rather you and I share common experiences in which reason and intelligence are to be found only rarely within our universe, and to all appearances, can only be found in highly evolved, highly organized biological animals with sophisticated neurological systems – and that understanding is probably fairly well internalized and goes w/o saying – and it’s that fact that makes your claim even remotely plausible. If scientific evidence for ghosts becomes available then we can modify this claim but until then that’s what our evidence both scientific and day to day experience is telling us. And more than that, it would appear that the more sophisticated the brain is in relation to body size & the larger the brain is, the stronger the ability to demonstrate such reasoning abilities. If your argument was in fact a priori, it would apply to all logically possible worlds & we could claim to know it with absolute certainty. We can conceive of a logically possible world in which every physical object contained therein is capable of Vulcan like reasoning and burning bushes were well known for their wise and intelligent discourse and there is not a priori reason why our universe shouldn’t be like that. I surmise that the inhabitants of such a universe would be absolutely astounded at your claim. I raise that issue because in the absence of clear evidence that intelligence exists o/s of physical bodies, it makes it somewhat more problematic that the creator of the universe is such a spectral being

    Ignoring that issue, If all the antecedent physical conditions were randomly selected & given the laws of nature as we currently know them, then I would agree with your proposition that it was unlikely that it would give rise to an entity that could reason. However, if those antecedent physical conditions were designed either through evolutionary processes in the case of the human or animal brain or through design in the case of a computer to give rise to the possibility or probability of solid rational reasoning, this claim would not hold true. Evolution relies upon random processes but it ultimately is not itself random. And if evolutionary processes lead to a brain capable of sophisticated conscious self aware thought processes on par with the human brain demonstrates on a day to day basis and if the test for rational thought is fairly straight forwards and simple, then it’s pretty apparent that such a brain should be capable of rational thought. Why would one doubt that? Every one of your arguments simply assumes that non-rational causes could not give rise to a rational belief & that’s not a justified assumption given the possibility that the non – rational causes were selectively chosen either by a designer or by evolution to give rise to rational beliefs. From the point of view of everyday experience, only sophisticated material bodies (through evolution or tinkering) specifically designed to think are the ones which showcase this ability. The easiest response to your argument is to apply the principle that what can be asserted w/o evidence can be rejected w/o evidence and to suggest that human beings are an excellent example of a strictly physical or natural object that responds to strictly physical or natural causes.

    I think you inadvertently missed this question from my previous post. How does the intercession of some spiritual homunculus make the claim that we are at least some of the time rational in any way more probable? Aren’t you making assumptions as to the nature and qualities of such a psychic entity? And further, how does the intervention of a supernatural deity make the claim that we are rational in any way more probable given that any randomly chosen possible deity may or may not have our best interests at heart? How do these spiritual homunculus communicate with physical bodies? Where do the spiritual bodies come from? How do they pick appropriate bodies? Why do we not find a human spirit in a crow? Are these a sort of day to day task undertaken by a deity? What physical adaptions are we going to find in the brain to allow communication between brains and these esoteric spirits. When we have too much to drink, do these spirits get drunk? What happens to these spirits when we sleep? If you split the brain of an individual in half, why does that result in 2 separate personalities or I suppose spirits? And so on. None of these questions are likely to prove helpful or fruitful, especially compared to ongoing current scientific efforts in which these questions are not addresse.? Aren’t all these claims not unlike claiming heat is caused by caloric or that sleep is caused by a sleep causing agent?

    “I like this statement as well: “If our beliefs weren’t true or at least trueish, nature would make short shrift of us.” Does it then follow that religion is at least “trueish” since it has been and continues to be the modus operandi of the vast majority of humanity since the beginning of human history? One could go so far as to say it is THE factor separating us from every other species and demonstrates our superior higher reasoning. Clearly nature must have chosen this orientation since it has been so wildly successful for us as a species. Hell, religion even gave us science.”

    I liked your response here even though I’ve suffered something of a comeuppance. I shall have to improve or clarify the argument the next time I present it. There’s absolutely no doubt that religion has enhanced our survivorability probably in large part through enhancing social cooperation especially once agriculture was discovered and societies became larger than mere tribes. As a side note, most of the early religions aspired to various nature deities ie wind gods or sun gods or the like which had no moralistic function but something of an explanatory function. That may be an oversimplification. Societies in which there was a single moralistic deity (and I’m not sure if your deity would in fact be categorized in this way) strongly (possibly always, I don’t recall) tended to be societies that were in trouble and were often found in or near deserts or other areas where food and resources were scarce and where full cooperation and the policing of resources were enforced by an all seeing deity. The other factor in common that all societies that worshipped a single deity was believed that their deity was the best one – I mean why limit yourself to a single deity if all other societies had more and better ones – and so there appears to be a correlation between a single deity and evangelicism.

    It’s a fair commonplace that religious people may well be happier than secular people and perhaps live longer – but when one factors out the social aspect it turns out that it’s social aspect that is predominately the factor for the personal benefits and that people who tend to be especially fanatical about their religions tend to be more prone to depression and ill effects. Again, not something one would expect if a deity was interested in being worshipped and was paying attention.

    At the end of the day, there may be multiple ways a species can enhance survivability – increased strength or stealth or speed for example having nothing whatsoever to do with rationality. And religion would in fact enhance survivorability – just not for rational grounds. But a) the ability to map and understand one’s terrain b) the ability to predict the future c) an appropriate theory of minds which allows one to predict & or direct the behaviour of prey and negotiate and work cooperatively with one’s associates, etc. are in fact abilities that factor well with intelligence and the ability to reason, even within lesser species.

    if you were looking to limit the time you expend in drafting a response, I would request that your response be focuse on the above paragraphs. Otherwise:……

    My first comment “whether we have free will or not is a brute fact” was simply intended to say that it’s either true or false. And we can only know whether or not it’s true or false based upon evidence. That evidence could be internal experiences or detailed systematic scientific investigations. From my perspective, the question of whether or not we have some ultimate free will remains a reasonable subject of scientific inquiry and the jury is out. We certainly have strong impressions that we have free will – and yet we can point to surprising studies that show the contrary. In any event, if a theist were to hold that free will is some metaphysical necessity by reason that a) God provided it or b) that it was part and parcel of some fair test in order to determine one’s moral fitness then scientific inquiry into whether or not we actually have free will.

    In retrospect, I can see how my first sentence might be misleading as it might be suggestive that there’s no deeper explanation – and I can only say that wasn’t intended. However I wouldn’t go so far to agree with ” If free will lent itself to being a matter of a brute fact, requiring no interpretive lens, the scientific, philosophic, psychological, and theological communities would shout it from the rooftops. No such shouting can be heard – I listen for it all the time.” Presumably we could all agree that we have free wil and everyone of us be wrong or the converse. We bend our efforts to determining the truth of the matter through discussion and investigation – and we may or may not get it right or we may or may not be able to assign reasonable probabilities for the truth of the matter.

    “Conceiving of the brain as a machine or computer is an idea that has been debunked from various angles for decades, though it does not stop many from believing.”

    Fair enough. Although I don’t know that anything hangs upon it. Nature as evolved over billions of years often uses different technical strategies than we do and quite often such strategies are superior to our own and there may or may not be technical challenges in using the same strategies as nature does. If this is intended to demonstrate that the brain/mind must therefore operate o/s of physical or natural laws, the demonstration fails.

    The recent use of neural nets in artificial intelligence is an attempt to use some of the brain’s strategies and with great success – for example the computer programs that translate Chinese to English in real time or recognize faces or win at go or are superior at diagnosing patients than physicians, etc. These programs are not simply set up with a given set of algorithms and that’s the end of it – rather they are taught and undergo learning and self adjust their own algorithms to better improve play The go playing program reviewed all of the professional games ever recorded and then proceeded to play itself repeatedly in order to develop its go playing skills. Sometimes programs are designed to use processes that mimic evolution in order to adjust their own working strategy. Robots can have what appears to be personality – and I can direct you to youtube links in which people naturally refer to the robot as he or she and otherwise personalize them. All of which makes it to my mind a little more challenging to claim that they don’t make choices analogous to our own choices.

    The question that you have to demonstrate is that the brain operates outside of 1) physical laws 2) natural laws (assuming there’s a meaningful distinction between the 2), not simply show that brains and computers use different strategies. If I were a materialist or locked into a world view in which naturalism is an essential part of my world view, then I would face something of an onus or burden of proof but that burden would be somewhat met by the state of our sciences – we have very good reason to believe that we know what the constituent elements of plants and animals and humans is – laws that apply to the universe at large – why should it not apply to us? We know the history of how neurological structures arose – and it was through physical processes – simpler brains can be replicated using computers which then recreate the same sorts of behaviour as the original animal. The human brain is sufficiently sophisticated and complex, it’s not at all surprising that different individuals will react different to similar phenomena, even twins. The key point of complex structures (which differ from structures that are simply complicated by reason that they contain a multitude of feedback loops) is that they are exquisitely sensitive to inputs – it’s one of the reasons weather is not fully predictable long term.

    If you google “twin study religiosity” you’ll find numerous articles on the study I mentioned. If there’s any trouble at all, I’m happy to provide specific links. I often prefer to provide the google variables as it avoids any issue that I might cherry pick the results. In the article at New Scientist the figure of 40% is used so possibly my recollection that it was 40% – 50% stands corrected. You can also go onto google scholar and use “separated twin study religiosity” which pulls up a number of actual academic studies.

  49. By the way Eric – please feel free to take your time before responding – blogs should be fun and not work – there’s no reason to feel pressured to respond quickly, especially when there are other substantial posts to address – weather is gorgeous – in my own circumstances, my wife has made it very clear that gardening that does produce tangible results is a priority over discussions that may well lead to no one changing his or her mind. Cheers

    • Haha, David, you may not believe in prophets but you may just be one. I was just agonizing over how to get a reply back to you of any substance and still get everything else done that I need done. This weekend and next week will be impossible to blog, hence even more pressure to push a response out now. I’ll take you up on continuing at another time, though I might make a quick stab at at least one point after responding to Sean Brennan. Appreciate your blogging spirit. Cheers.

  50. Hi Sean Brennan,

    Thanks for your post. With some time restraints kicking my butt at present I will attempt a response covering what I think is your main point. First, as side note. The historic Orthodox Christian faith, which I fall in with, has a very specific anthropology and understanding of what we mean by “soul”. In short the Greek Fathers accepted Plato’s tripartite soul as written in the Republic. Beyond that we also hold a larger tripartite of the person: spirit, soul, and body. So, though I’m sure your Christian friends meant well they may not have been reflecting the Orthodox rendering of “soul.” There would be aspects of soul that some animals certainly possess and others they wouldn’t. That’s the very short answer to that. But your main point seems to be this:

    “In your previous reply to me you simplified the argument from reason for me as thus;
    1. Amy believes in xyz because this belief is based on good reasons.
    2. Amy believes in xyz because of an antecedent physical condition which caused the belief.”

    “Given the chimp stories above, let’s state the following.
    Amy the 8 year old human believes she can get the nut from the tube because this belief is based on good reasons. Amy the chimp believes she can get the nut from the tube because of an antecedent physical condition which caused the belief, as she is just a chimp. The distinction between the two is not as apparent as it first appeared to me; in fact it all looks rather fuzzy now.”

    The best I can do to unfuzz it is to make this distinction: If both Amy human and Amy the ape’s behavior and reason processes were fully explainable in physical terms then the only possible action either would take would be determined by physical causes, not by free agency. In the scenario presented by #2 they could only have believed they could reach the nut and only have responded in a particular manner based on cause and effect chemical physics happening in their brain. In scenario #1 good reason may have led them to believe they could reach the nut but they still could have decided that they couldn’t or didn’t want to, or whatever.

    The difference is one of possible outcomes. To plug this back into the larger discussion, if the only possible outcome of my belief was whatever the antecedent physical condition determined my belief, or action, or thought, etc., to be then my beliefs are fully explainable in physical terms, no need to interject reason. The process works perfectly without it. In the Darwinian concept of life our reason faculties are present in the same sense that any biological faculty is present – to benefit in the struggle for survival and reproduction. And the entire process is governed by physics and chemistry. It is not only unnecessary to go outside physical laws, it is forbidden since nature is conceived as ‘closure of the physical,’ i.e., there can be no other causes than physical causes. “Good reasons” in this conception have no causal powers.

    Thus, if one believes in evolution he must do so based on whatever physical conditions caused the belief. The fact that he seems to have good reasons for those beliefs must be a illusory narrative semi-imposed upon our thinking to give the impression that “reasons” exist and matter in the outcome.

  51. I don’t really see how materialism can escape nihilism.
    1) Materialism assumes that ONLY the material world exists.
    2) In the material world every event or entity has at least one natural cause.
    3) Natural causation must follow natural laws
    We can safely conclude that in materialism every event and every entity is fully determined or occuring or existing randomly (but still in agreement with natural laws, this time including quantum indeterminacy).
    This include thoughts, concepts, beliefs, perceptions and so on.
    Nobody could have thought any different, nobody could have acted any different, because nobody has any control over natural laws or random events. There was never and is and will be no agent and no agency in materialism. That includes ourselves (the self as agent is an illusion).
    There is also no purpose or meaning in the classical sense.
    I have yet to hear a convincing argument that makes me think otherwise.
    If someone could convince me that materialism is true, I would immediately stop my scientific research, go home, take it easy and have a beer. Wait… I would not have a choice on what to do, so let me rephrase: ‘I’ (?) would observe what ‘my’ (?) deluded brain thinks is a ‘self’ going home and have a beer. What actually would happen is that a complex assembly of elementary particles continues to operate according to natural laws and some ethanol chemistry – until it doesn’t. And that is life on materialism in a nutshell, my friends.

  52. Starving to death is no more appealing whether you believe in theism, deism, materialism, determinism, humanism or any other ism. Materialism doesn’t imply that we don’t have emotions, wants, needs & desires, objectives and goals including social goals such as the desire to be respected amongst one’s peers. If it did, then it would be game over given that it’s easy to come up with counter examples. And people who believe in determinism don’t spend their time sitting back drinking beer as they (and you) have other more appropriate goals – it’s just that they believe that the ultimate explanation for who we choose our goals is ultimately determined by reason that we are children of natural processes and that it is in theory possible to describe our decisions as being in complete accord with our physical biology, our history and experiences and the current exigencies with which we are faced. No magic is necessary to account for our behaviour.

    Morality is simply a tool humans (and loosely speaking other social animals) use to generate results that would not be achievable if cooperative behaviour was not achievable. There’s a bit more to it but that basically the long and short of it. It doesn’t lead to nihilism – not by a long shot. We are evolved to be social animals and live within communities and that includes the ability to subsume one’s personal desires to achieve some greater good that one believes in and desires more deeply. Take a look at games theory as an mathematical model which explains why cooperative behaviour makes good sense.

    I’m not even sure that materialism implies determinism – if one assumes that quantum phenomena can be described as materialism – but that’s by the by.

  53. ‘No magic is necessary to account for our behaviour.’
    If there is nothing supernatural in humans, we are biochemical robots. No amount of verbose self delusion changes that.
    There are three options:
    a) live in cognitive dissonance as if concepts like free will, agency, good and evil, purpose, and meaning still existed
    b) accept nihilism
    c) question materialism

    Most atheists I know have rejected God for irrational reasons out of emotions linked to an oppositional personality structure. Some are angry at God after an unsuccessful attempt to control Him. More common is a general unwillingness to subordinate oneself to a higher being – it conflicts with the individualism that is so rampant in our society.
    Then the rationalization of the emotional decision starts long after it has been taken. The irony is that in the attempt to make oneself the center of the universe, the one who decides what is moral and what makes sense, the one who sets the ultimate goals, one looses everything. The attempt at ultimate autonomy ends in utter dependence on a pointless universe which exists inexplicably as brute fact.
    So option a) becomes the default position, because the other options are unsettling. Depending on the strength of the initial emotions, the path to option c) may remain blocked for a long time.

    Human freedom and dignity are divine gifts which a mindless universe cannot bestow.

  54. you could, you know respond to the actual arguments presented to you as opposed to this wild flailing about with new rather wild and somewhat annoying claims.

    “If there is nothing supernatural in humans, we are biochemical robots. No amount of verbose self delusion changes that.” – says you. It’s not true, especially if we think of current efforts to build a robot. We, as sophisticated biological beings are much more advanced with conscious awareness and everything. Robots don’t come with that particular characteristic, at least so far.

    “There are three options:
    a) live in cognitive dissonance as if concepts like free will, agency, good and evil, purpose, and meaning still existed
    b) accept nihilism
    c) question materialism”

    False dichotomy – prove it. There’s a useful place for the concept of free will – when one is not under duress. I have no difficulty using the term, agency. Good and evil are determined by human needs and wants & there’s well advanced secular & scientific thinking on issues of morality. Meaning – well, we’re able to communicate – so I am able to express my thoughts in a way that you understand – that’s meaning. I have purpose in my life – but it’s a purpose I choose. It matters not to me that there may be some underlying physical explanation – that understanding has no impact on my day to day choices. There is no apparent purpose to the universe at large – and that’s not for want of looking. If there was such a purpose – if scientists could use teleological reasoning as a way of teasing out the universe’s secrets, then we would anticipate seeing instances of it.

    Atheism predates Christianity by thousands of years. I’m looking forwards to you providing an actual argument supporting the claim that your deity exists – although I think you’d be taking us far afield from what this particular blog is about. I’ve encountered numerous versions of your God, I’ve studied theist and atheist arguments at some length – and frankly theist claims have proven so far to be pretty dubious.

    Your last statement is a bald assertion that isn’t true. I think the last word here should go to Eric as to whether or not he wishes us to pursue a very broad off topic argument. I’d like to give Eric an opportunity to respond to my last post to him. There’s an interesting discussion with Sean that’s on topic. I think your arguments and my responses are the wrong way to go. I don’t know Eric well but I expect to be around for a bit – and you’ll get your chance to pursue your issues another time.

  55. I am sorry that my post makes you uncomfortable. You haven proven my point, albeit unwittingly, with your fourth paragraph. You apparently are well aware that you have lost free will, good and evil, meaning and purpose (and back to the original topic – the reliability of your own reasoning) in your worldview and resort to redefining these concepts into something else. Purpose it what I choose it to be? Please. Reality is not defined by your perception of it.
    In any case, I’ll bow out here. Before you get too stressed out, remember that in your worldview I could not have written a different post. Every word that you read is a direct consequence of my genetic makeup and prior experiences and maybe my hormone levels that day and what I had for dinner. It just happened for no reason and I could not have changed any of it. Your responses including all your reasoning are also fully automatic or random, so sit back, relax and watch the show. We are just two biological machines who can’t help interacting in a strange impersonal way via some complex electronic machinery, namely the internet. Some synapses firing and electrons moving about, that’s it.

  56. your post made me annoyed because it was insulting – nothing more – the interesting thing is from my perspective is that when there is a discussion in which there is something of a team – and it would seem that one side is losing, (which I think Eric is) it’s quite common for someone else on his or her team to step up the plate and attempt to change the topic and/or engage in ad hominems – which is what you’ve done. I haven’t claimed to be materialist or a determinist – in fact I indicated in my very first post what my actual position is – which is pretty much that I’ll go along with whatever the evidence indicates – and I I stated that I don’t view it as a subject that can be resolved a priori or because it happens to fit your fancy and that I very expressly stated in no uncertain terms that I do not view myself as either a materialist or naturalist – I don’t view either term as particularly useful.

    I think we can say that in broad terms the fact that you responded and the manner in which you responded were pretty highly predictable. You appear to have covered all the bases that you were capable of w/o in any way relying upon any scientific principles or evidence and your arguments conveyed the message of arrogance and sophomoric humour (which is pretty much the way you’ve presented yourself throughout this discussion). My reaction in the first place of being annoyed as a result of your insults were not a matter of personal choice. I’m not choosing to be annoyed or exercising my free will not to be annoyed – I’m just annoyed. And I predict that you will feel annoyed when you read my post no matter how much you utilize your supposed free will to the contrary.

    We seem to be engaged in some sort of zero sum pissing match which will leave both of us annoyed which I compare quite unfavourably to the way that everyone else who has been engaged in this topic have behaved. Your first paragraph, by the way is complete nonsense – rather than review every sentence, take this one: ” Purpose it what I choose it to be? Please. Reality is not defined by your perception of it.” Not what I said. But it doesn’t surprise me in the least that your reading comprehension is blinded by your biases.

    I applaud your decision to bow out. It will be interesting to see if you meant it.

  57. Having read through all of these posts including those of Catholicscientist and Davids. There is something I would really like to sort out with these discussions; definitions that can be used consistently throughout our discussions.

    An awful lot of confusion arises with words like “exist”, Re;CatholicScientist above;

    QUOTE >”There are three options:
    a) live in cognitive dissonance as if concepts like free will, agency, good and evil, purpose, and meaning still existed.
    b) accept nihilism
    c) question materialism ” <

    To "exist" is TO BE; To be is to be SOMETHING in particular. Free will,agency,evil,purpose and meaning are not "PARTICULAR THINGS",( Particular- as in an individual item, as contrasted with a universal quality.) They are, as you state; concepts.

    Existence just is. It does not have a reason,( reason = a cause, explanation, or justification, only EVENTS have a cause, explanation, or justification) Existence is what is and always present tense.What I have in front of me is not a "used to be" a oocyte,(egg yolk formed by a chickens ovary) or the beginning of an omelette. What exists before me is a chicken egg.

    We live in a world where everything in our language resolves to being either an object, or a concept.

    An object is something with shape/form. EG; dog,chair,moon,cup,egg.
    An object + a location = exist. As in objective existence; IE exists whether a human ape is there to observe it or not.
    How can we rationally state that SOMETHING exists without it being a some – thing, rather than a no – thing, and being located somewhere,(or everywhere?) rather than nowhere?

    Concepts are notions in human heads that are relations between objects and/or other concepts as perceived by the human ape. Eg agency is the capacity of an entity/object to act upon other entities/objects.It is not a particular thing, it is a relation. Free will is is a condition by which a living entity has options/actions available to choose from and the freedom to carry out those options/actions.Again a relation, not a particular THING.

    So why should a materialist deny the relationships that he perceives between objects? How does materialism prevent the materialist from seeing something as "good".?
    Good = to be desired or approved of; having the qualities required for a particular role; benefit or advantage to someone or something; that which is righteous in somebody's opinion.

    If some lunatic and his followers are intent on wiping us off the face of the earth and we kill him. Is that good? Some would say so.Some might even claim "Thy shalt not kill"and its evil. Who decides? You? Me? The Pope?

    Where and how do we get to this, "If there is nothing supernatural in humans, we are biochemical robots"?
    None of the concepts mentioned by Catholicscientist require anything super natural. Bobo the chimp has a concept of self, free will, agency and a chimp concept of good and bad.

    Does the love and the care of the super natural theists speak of extend to our close cousins the apes aswell?

  58. Sean,
    I could have worded that more carefully, thanks for correcting me. A concept is an abstract object conceived to describe an aspect of reality. This can be quite concrete, as in the concept of a computer that describes the actual computer that exists in realty. There is a match between the entity in reality and the concept of it, ideally. For example, if a programmer develops an algorithm for the purpose of sorting data, that is it’s actual purpose. If a computer running this algorithm were capable of artificial intelligence, it might be able to form a concept about the purpose of this algorithm, but it does not necessarily match the actual purpose. The theist conceptualizes that there is in fact an objective purpose to our existence, there is objective morality, and we do actually have the ability to start a new causal chain (agency). In materialism these classical concepts are redefined to describe something different. You will find a large variety of concepts in materialism regarding free will or agency, for example. Sam Harris calls it a self delusion of the mind. Daniel Dennett talks about choices that are taken without any forceful influences of other humans, but when asked will deny the ability to have acted differently in any given instance. Compatibilists have a variety of definitions of what they think is possible under materalism.
    You write ‘ Free will is is a condition by which a living entity has options/actions available to choose from and the freedom to carry out those options/actions.’
    You would have to explain what ‘available options/actions’ are under materialism and what ‘freedom to carry out’ means. Depending on their ontological position, people may not agree on what these concepts describe. Daniel Dennett’s definition of ‘freedom’ is radically different from the the Biblical concept of freedom, for example.

    I can’t address all your questions but the one here:
    ‘Where and how do we get to this, “If there is nothing supernatural in humans, we are biochemical robots”?’
    is easy:
    Under robot I understand an agent that is controlled in its actions by a fixed program (including random generators). Under materialism we cannot but follow natural laws in all our actions. What we do and think is fully determined by genetic makeup and antecedent causes. Hence we are biochemical robots, and chimps are as well. In materialism there is no ontological difference between ants, chimps, or humans, they are all social animals with varying complexity.
    In theism there is such a difference between humans and animals. Humans can interact with the divine, or you could say humans have a supernatural dimensions. Animals do not. Hence, they have no culpability. You do not fault primates for their frequent infanticides. I can’t just eat my neighbors kids, on the other hand, even if I wanted to.

  59. “Under robot I understand an agent that is controlled in its actions by a fixed program (including random generators”

    That’s how some robots work, not all. An artificial intelligence neural net uses a different technique in which the actual algorithms evolve in response to learning – they are not programmed – they learn by example – see google deep mind that played and defeated the world champion. That same program) played a much poorer game 6 months before. With Deep Blue (the computer program that defeated Kasparov), it would have been possible for a programmer to work out the move that Deep Blue thought best by working through the algorithms manually – such would not have been possible with Google Deep mind.

    I suspect that one of the main challenges for theism and particularly you and Eric is to make a compelling case that homo sapiens are o/s of nature whereas other animals are part of nature. Where do you place homo erectrus, hom ergaster, homo antecessor, homo Neandertals, and all of our other ancestors? Humans have developed through evolutionary processes over extended time periods with only minute changes between generations. Other species are clearly capable of consciousness so that isn’t the distinction but apparently you concede that one can be fully conscious and nevertheless driven by physical processes. Other species clearly demonstrate significant intelligence and will succeed on some intellectual challenges that you, yourself will fail – google “are you smarter than a chimpanzee” for an example. You can follow along the challenge on youtube. Other species play and enjoy themselves. They can be wilful and stubborn. Other species form life long partnerships. Other species demonstrate empathy and or a sense of fairness. Other species are able to cooperate with other members of their species and with other species. Other species have a theory of mind – an example you can find on youtube – a wild bear cub with its head stuck in a jar came up to 2 humans for assistance. Other species can use language although in the wild it’s more of a prototypical language. Again on youtube, you can find a sheepdog with a functioning vocabulary in excess of 2000 words, if I recall correctly.

    Humans clearly have greater intelligence & communication skills – but that in itself wouldn’t in itself imply some fundamental difference in the quality of or the foundation for the reasoning. We train and indoctrinate our children to distinguish right from wrong & punish them when they do wrong – not unlike toilet training a puppy. Our moral codes vary dramatically throughout the world. Our current civilized western values are outliers amongst all of the humans that ever lived – and the values of our traditional ancestors were very different – more along the lines of if I find a stranger in the forest – kill him or run away – those were the only 2 options. One tribe – if you wronged them – their policy was to forgive you – wait a year – treat you nicely – invite you and your friends over for a feast – then slaughter the bunch of you.

    So why does an internal magic being require this education – and why does this education not easily override our moral inculcation when such moral inculcation fails. Jonathon Haidt whom you can find readily in the TED discussions outlines fairly clearly where our morality comes from – and it’s based upon 6 dimensions: – which the wikipedia entry identifies as:

    • Care: cherishing and protecting others; opposite of harm.
    • Fairness or proportionality: rendering justice according to shared rules; opposite of cheating.
    • Liberty: the loathing of tyranny; opposite of oppression.
    • Loyalty or ingroup: standing with your group, family, nation; opposite of betrayal.
    • Authority or respect: obeying tradition and legitimate authority; opposite of subversion.
    • Sanctity or purity: abhorrence for disgusting things, foods, actions; opposite of degradation.

    None of these elements requires divine intervention. And there’s a clear physical dimension to our morality that I think requires explanation by theists – a significant part of our moral reasoning (particularly amongst conservatives and theists) is based upon moral disgust – which uses some of the same brain/gustatory functions as when we experience disgusting foods and odours. And which is perhaps why we are less compassionate and less cooperative after eating a lemon or other sour food (surely that requires an explanation from the theists). It turns out (at least according to Haidt) that we make our moral judgments as a matter of primal instinctive reactions – and then rationalize them. Progressive thinkers tend to favour care/empathy plus fairness whereas conservative thinkers tend to place more weight on the other dimensions – and tend to have stronger visceral disgust reactions to unpleasant food or situations.

  60. Hi David Aiken,

    This is in reply to your post from way above. Kids are down for a nap, no telling how long, and tomorrow starts a pretty hefty week for me, so I’ll do my best with what I have. I’m thinking the copy and paste method of response might work best. Okay, here we go…

    “Hi Eric. I thought I had kept quite nicely to 1 or 2 issues – I just happened to be long winded in connection with those issues.”

    You did good, just a lot of content for one post. Sort of reminds me of the old Rodney Dangerfield movie, Back to School seen: https://www.youtube.com/watch?v=BIroRqO_BpM

    Responding to my reply to “bald assertion” claim…

    “It’s not so much that much of what you wrote in your article was a bald assertion so much as it was that this specific proposition repeated with different wording on a number of occasions that I considered to be a bald assertion – and one that for the most part simply assumed your ultimate conclusion. It’s the key issue and one that I don’t agree with. I don’t think it’s an a priori truth or logically sound and it’s this claim that is specifically the bald assertion that I’ve taken issue with and it’s a fundamental to your argument. If it fails, your argument fails.”

    Then you’d need to show why it fails not just mistaken it for bald assertion.

    “Firstly your argument or conclusion is not something that can be arrived at a priori – rather you and I share common experiences in which reason and intelligence are to be found only rarely within our universe… If your argument was in fact a priori, it would apply to all logically possible worlds & we could claim to know it with absolute certainty.”

    That something is known a priori does not necessarily mean it is infallible. In this case it means that through careful deduction and attention to the language the truth of the argument from reason can be demonstrated. I know you are not a fan of Reppert’s syllogism in the OP but I included it as a help for those new to the concept. The argument does not stand or fall on it since it can be accurately stated in other ways as well.

    “We can conceive of a logically possible world in which every physical object contained therein is capable of Vulcan like reasoning and burning bushes were well known for their wise and intelligent discourse and there is not a priori reason why our universe shouldn’t be like that.”

    Can we? I think you are expanding the argument far outside its boundaries with the idea that we can conceive of a world where rocks have intellect. Not sure how you can logically deduce this, but I’m open ears.

    “Ignoring that issue, If all the antecedent physical conditions were randomly selected & given the laws of nature as we currently know them, then I would agree with your proposition that it was unlikely that it would give rise to an entity that could reason.”

    Sounds good.

    “However, if those antecedent physical conditions were designed either through evolutionary processes in the case of the human or animal brain or through design in the case of a computer to give rise to the possibility or probability of solid rational reasoning, this claim would not hold true.

    Are you saying you believe evolution is somehow teleologically oriented? If so you are not arguing from a strict materialist point of view and are taking a path more like that of Thomas Nagel. If this is the case cool, but it seems to me incoherent without ending up with a cosmological view that has a supreme intellect at the helm, i.e., nothing sort of a God.

    “Evolution relies upon random processes but it ultimately is not itself random. And if evolutionary processes lead to a brain capable of sophisticated conscious self aware thought processes on par with the human brain demonstrates on a day to day basis and if the test for rational thought is fairly straight forwards and simple, then it’s pretty apparent that such a brain should be capable of rational thought. Why would one doubt that?”

    We wouldn’t, except that this concept of yours is not consistent with the Darwinian evolution story, a story which requires physical causal closer of the universe (sorry to have to repeat myself here). Evolution has to answer how, under its restraints (causal elements being only according to physical laws, biology orients for survival and reproduction, etc.) it can give rise to subjective consciousness – out of which the sort of reasoning we are discussing arises – and how it is reducible to something physical. There are many attempts at resolving this issue: Identity theory, Supervienience theory, Epiphenomenalism, etc., all of which have this problem with the seemingly impossible task of reducability. On top of that, if one takes this materialist course he must account for his reason’s validity but he cannot find this reliability using evolutionary processes. Nagel puts it like this: “Any evolutionary accout of the place of reason presupposes reason’s validity and cannot confirm it without circularity.” E.g., Qu: “How do I know that my thoughts are wholly grounded in physiology.” An: “because physiology told me so.” This of course is question begging, however saying rather that God is the ground of intelligence is consistent, non-question begging/circular in its own rubric.

    I want to drive this point home so let me give one extra thought to it. Physical facts are not “true” in themselves (follow me here), but are explained in terms of actual or possible experience. And what is experience grounded in? Our thoughts. And what are thoughts grounded in? Materialism answers: physiology. It is circular reasoning as soon as one asks how their own reason – if wholly determined by physiology – can say anything authoritatively about itself.

    Thus (here it comes again, sorry) materialism slits its own throat.

    “Every one of your arguments simply assumes that non-rational causes could not give rise to a rational belief & that’s not a justified assumption given the possibility that the non – rational causes were selectively chosen either by a designer or by evolution to give rise to rational beliefs…”

    I think your take here is too simplistic. It does not account for what is being discussed in full, which is why I attempted to lay out the argument in various but similar ways. If you don’t like the terminology of non-rational vs rational try objective and subjective instead. As shown above it seems illogical to believe that mere objects, that have no intellect whatsoever, give rise to subjective states of mind. What are these subjective states? An: according to materialism they are ultimately to be identified with -reduced to – something physical. Take your pick of reigning theories and you will discover what Nagel discovered, that “the materialist neo-Darwinian conception of nature is almost certainly false.”

    “I think you inadvertently missed this question from my previous post. How does the intercession of some spiritual homunculus make the claim that we are at least some of the time rational in any way more probable? Aren’t you making assumptions as to the nature and qualities of such a psychic entity? And further, how does the intervention of a supernatural deity make the claim that we are rational in any way more probable given that any randomly chosen possible deity may or may not have our best interests at heart? How do these spiritual homunculus communicate with physical bodies? Where do the spiritual bodies come from? How do they pick appropriate bodies? Why do we not find a human spirit in a crow? Are these a sort of day to day task undertaken by a deity? What physical adaptions are we going to find in the brain to allow communication between brains and these esoteric spirits. When we have too much to drink, do these spirits get drunk? What happens to these spirits when we sleep? If you split the brain of an individual in half, why does that result in 2 separate personalities or I suppose spirits? And so on. None of these questions are likely to prove helpful or fruitful, especially compared to ongoing current scientific efforts in which these questions are not addresse.? Aren’t all these claims not unlike claiming heat is caused by caloric or that sleep is caused by a sleep causing agent?”

    This line of questioning takes us deep into theology and I’m willing to engage it since it is sort of what I’m into academic wise, but it is a long discussion. Attempting a quick reply would be something like: yes, mind over nature is nothing short of a miracle. How can a mind cause physical objects to move? Analyzing the brain is only part of the work, one must include the mind. Some believe they are identical. Good luck proving it. This is at the heart of the matter, which I predicted a few posts back that we would eventually come to. Why is belief that spirit (I will state this briefly without getting into a complicated breakdown of spirit vs soul) is a better solution of agent causation than merely physical? because, for one, the entire cosmos are seated in such a view. To say that the physical caused its own existence is logically absurd. One seems forced into concession that physical is eternal, but then comes the ultimate kicker: Why is belief in an eternal non-intelligent something (nature) logically superior to belief in an eternal intelligent something (God)? Answer: its not. If the physical can produce the mental then it seems to be operating with a telos – with an end goal. Why is it not more intelligent to believe that nature is ultimately intelligent and produced intelligence of its own kind. The mental is the pinnacle of nature’s products (I’m speaking for Nagel here, not Christianity). Move the discussion to any classical theistic claim dealing with ontology and you are better off then with naked materialism. Move it over to historic Christianity and you have, in my view, your actual answer. Why Christianity? That’s the funnest part of this discussion but takes us out of the article.

  61. Hi Veggiedude,

    Your post has nothing to do with the article but I will address it for fun. You pose the ol’ “If God can do anything then can he make a rock so big he can’t lift it” thing. The Christian belief that God is all powerful does not mean that He can defeat logical absurdities. Rather it means that all power is His. He created all, is the ground and being of all, gives life, etc. The question fails immediately due to the fact that it pictures God as an object along side all other objects in the universe. It takes what classical Christian theism believes about God and makes it into a caricature – like the picture of the old bearded one in the sky – which has no relevance to the faith whatsoever. But to indulge you, yes: if God were pitted against himself in a fight it would be a stalemate.

    If you plan to argue with Christianity in the future I would highly recommend becoming familiar with it’s beliefs first. I don’t say that in any way belittling but just as a matter of fact. You’ll do much better defeating the faith if you actually attack it versus popular fantasy about it.

  62. Sean Brennan,

    I had to respond to something you said to CScientist. You said:

    “Existence just is. It does not have a reason,( reason = a cause, explanation, or justification, only EVENTS have a cause, explanation, or justification)”

    This phrase reveals some misunderstanding about what is being discussed. To begin scientific study – for the purpose of manipulating and harnessing the power of nature – one begins with “existence just is” and moves on. Well and good. But when the discussion is about metaphysics, and everyone has one, this won’t do. “Just there-ness” of the universe is logically indistinguishable from magic.

    The question of existence is not one concerning the physical origins of things, or of how one physical state may have been produced by a prior physical state, or of physical persistence across time, or of the physical constituents of the universe, but one of simple logical or conceptual possibility: How is it that any reality so obviously fortuitous – so lacking in any mark of inherent necessity or explanatory self sufficiency – can exist at all? (David Bentley Hart).

    It’s a question generated from the observation that everything natural is contingent on something else for its being – nothing is self sufficient. The logical deduction from contingency is that the Absolute, or the Necessary, by which all things have their being, by which all things are derivative. This is a very strong argument for the logical necessity of God (the Absolute/Necessary). Alternatively one can hold a belief that reality is absolute contingency, unconditional conditionality, an uncaused effect, i.e., a concept of reality that is essentially absurd.

    But carry on with your discussion. Just wanted to add that.

  63. Hi Guys,

    Eric you write, “But when the discussion is about metaphysics, and everyone has one, this won’t do. “Just there-ness” of the universe is logically indistinguishable from magic”

    The universe is not a philosophical entity. It does not exist as a being. Rather the universe is the conceptual set of all material beings. The term you use,”there-ness” of the universe”, implies that the universe is both an entity in itself and an entity humanly knowable as an entity in itself. It is not, if you believe it to be, then please direct us to a diagram or drawing of this thing called a universe which we all recognize as such.

    No one claims that the universe is an object as in a thing with shape/form. No human has even sense knowledge of the universe as a thing. The universe, as the set of material things, is an abstract concept, not an entity of human experience and knowledge – when one starts to pretend that concepts,(notions in human heads) actually exist of course talk of existence will be” logically indistinguishable from magic”, as you put it.

    I am sorry to have to go on about this but you quote,”How is it that any REALITY,(emphasis mine) so obviously fortuitous – so lacking in any mark of inherent necessity or explanatory self sufficiency – can exist at all? (David Bentley Hart).

    Reality = the world or the state of things as they actually exist, as opposed to an idealistic or notional idea of them.

    Universe is really just a word for everything = all space and matter.

    You write,”It’s a question generated from the observation that everything natural is contingent on something else for its being – nothing is self sufficient.”
    It would seem that everything we observe is made up of pre existing matter, and all causes are material things, “objects” interacting with other objects.
    IE Object (A)Object(B) = change/effect.

    I do not get to an immaterial anything causing material things to move. We just do not see that

    It would appear that the particles that make up atoms and all we observe do appear to be “self sufficient” Eric. We do not create or destroy matter, it just changes form.

    Why shouldn’t it be a possibility that matter has always been around in some form or another – Just like God 🙂

  64. Hi Eric. Welcome back. I’m very interested in your reply to my last post. Your most recent post addressed to me raises interesting issues in which we disagree. I cannot undertake to be short. However, I think what I might do is resist posting a response to sometime next week which will allow you to catch up – if that’s ok.

    “It’s a question generated from the observation that everything natural is contingent on something else for its being – nothing is self sufficient.”

    How do you know that? What would a quantum field be contingent upon?

    Mainstream science would have it that quantum fields underlie reality and provide a ready explanation as to why all electrons look the same and why all quarks look the same. The existence of our own universe may well be explicable in terms of a quantum vacuum fluctuation (of a quantum field) or based upon eternal or chaotic inflation of an underlying field under negative pressure. Time begins for our universe in the big bang – and so arguably there never was a time when there was nothing. The mathematics of quantum fields is both elegant relying massively on the notion of symmetries and powerful and extraordinarily restrictive. The standard model does not allow much in the way of wriggle room to modify the equations. Every quantum particle (an excitation of a quantum field) that scientists have discovered since the late 1960’s (and there has been a host of them) was predicted by the standard model. Not unlike I suppose how scientists were able to predict the nature and quality of all of the elements using the periodic table. Scientists don’t know why there’s is something – but they also don’t know why they should presume that there would be nothing.

  65. Sean,
    I don’t get your point. Are you saying the universe does not exist? If the universe does not exist, how then can I exist? Can you explain this a bit more?

    ‘I do not get to an immaterial anything causing material things to move. We just do not see that’
    That is a matter of interpretation, I suppose. I just made the decision to drink a cup of coffee and just had the first sip. In my worldview this decision was the beginning of a new causal chain, caused by true agency, by a mind that is not predetermined by natural laws, which means it must be my supernatural faculties moving the natural. I don’t see where the problem is there. It happens all the time.

  66. Hi Catholicscientis

    If you state that a “thing” called the universe exists, just as a cat, or a cup, or a brick exists then please show us what exactly it is that exists.Direct us to an image or a diagram of that thing we all recognize as a universe. You cannot, because it is just as I stated above, “It does not exist as a being. Rather the universe is the conceptual set of all material beings”.

    Catholicscientist you really need to define this word exist. To do so would clear everything up nicely.

    There is no shape or form to this concept we call a universe, If you think of the universe, as you would of an apple, or the artists impression of the mathematical big bang model we have all seen, with nothingness around all of the stars and galaxies; with you sitting there ‘outside’ looking in on it, you have a really lousy concept of the universe. You are taking a perspective of being outside of ‘everything’; this is not a helpful picture. Where would you even get an idea like this?

    This is what William Lane Craig and other theists do when making their Kalam argument claims, where they conclude;” Therefor the universe was created by an uncaused cause outside of space and time ”. How can these people rationally take a perspective of anything being outside of the universe? Don’t their followers and fans think?

    What could the boundary of man’s concept of everything possibly be made of, for these idiots to claim the existence of an “outside”? Lines an artist has drawn of a mathematical concept?

    The universe is the total of that which exists—not merely the earth or the stars or the galaxies, but everything. The word ‘universe’ defines what we humans refer to as everything.
    Is the universe then unlimited in size? No. All of the matter which exists is finite. What then, you may ask, is outside all of this space and matter, if it is finite? This question is invalid. The phrase “outside the universe” has no referent.

    The universe is everything. “Outside the universe” stands for and means “that which is where everything isn’t.” There is no such place. There isn’t even – nothing, “out there”: there is no -“out there.”

    Object/entity = that with shape/form
    We do not define objects, we point to and name them, Just like God did to Adam and Adam to the animals. We may however attempt to describe objects.

    Concept/notion in a human ape head = a relationship between objects and/or other concepts. We define concepts.

    Object/entity + a location = exist. Only objects exist, not concepts. EG Love is a concept, it is a notion in a human ape head. The deep affection and sexual attraction you feel for your mate you may refer to as love, but it is a notion in your head. It does not exist. It has no shape or form and it has no location in space.

    The universe is a notion,the idea of a super-set of everything in the human ape head. Space and matter exist. Not notions in human ape heads.

    To simplify this; Let us take a concept like “neighborhood”. You might think of your neighborhood as a geographical area with tree lined avenues, white picket fences and double story homes with well kept gardens,kids,dogs,cats and the jackass next door. My neighborhood might be four city blocks of high rise apartment blocks and shops at street level. A neighborhood is a concept. There exists no recognizable entity such as a neighborhood. It is a concept in our heads. What does “exist’ is the things/objects that you find in that area you call your neighborhood. The same goes for “universe”. It is a concept in a human head – not an entity which we all recognize as such. Universe = all space and matter = everyTHING.

    You state”‘
    That is a matter of interpretation, I suppose. I just made the decision to drink a cup of coffee and just had the first sip. In my worldview this decision was the beginning of a new causal chain, caused by true agency, by a mind that is not predetermined by natural laws, which means it must be my supernatural faculties moving the natural. I don’t see where the problem is there. It happens all the time.’

    To get to basics. “Nothing happens until something moves”. SomeTHING being an object. Not a notion in a human ape head. Love does not fly. Faith does not move mountains – bulldozers do that. You can make all of the decisions you like Catholicscientist but until some object moves and interacts with other objects you aint getting coffee. EG your hand moving toward the coffee pot. The rock will not break until the hammer strikes it, not the notion of a hammer striking a rock in a human ape head.

    Causality is when we have things = objects interacting with other objects to give us a change or effect.

    Object (A) interacts with Object(B) = change/effect. As Eric states this is what we observe in the world around us. As Einstein stated, “Nothing happens until something moves” It does not matter how many notions human apes move around in their heads, until we have an entity/object that exists moving and interacting with another entity/object that exists nothing happens.

    I find it strange therefor that people claim that because of what we observe a non material thing located no where in particular is responsible for starting movement. We just do not see examples of this in the world around us.
    The problem I think lies in that theists think that if they can demonstrate ‘why’ something happened as well as ‘what’ happened (as in God did it) This is causality; It is not.
    Our brains are wired to see patterns, even when they’re not really there . We are constantly looking for patterns/concepts that support our beliefs about how the world works,and if there is no pattern, then we seem to have the capability to invent one! Including turning concepts into objects and bringing them into existence to support our beliefs.
    “People assume that the world has a causal texture – that it’s events can be explained by the world’s very nature, rather than just being one damn thing after another.” Steven Pinker

    • Sean,
      I still don’t understand your argument. You seem to indicate that there are objects that exist and these need to have a shape or form (I wonder what your definition of form is, it is probably not that of St. Thomas Aquinas) but the sum of all these objects does not exist. Does that mean that my natural body does not exist because I am the sum of a large number of elementary particles?
      Also you restrict existence to entities that have a shape or form, and a location. What then would you say about an electron? It cannot be located precisely in space and does not have a shape – whether it has a form depends on your definition of form. What about a photon? We have two seemingly contradictory concepts of it, that of an electromagnetic wave and that of a particle without mass (no shape again). It moves with the speed of light, which means it is no longer bound by time. Does that mean photons do not exist, but light does? Or does it not? Reality is not limited to what you can see and feel and reality is much stranger than we can comprehend as anyone who has ever looked into quantum mechanics can attest to.

      I’m afraid your characterization of the Kalam argument is unfair. It does not use the idea of ‘outside of the universe’. I don’t think it is very helpful to put up this straw man and then claim that people fall for that in idiotic fashion while it is in fact your own faulty concept that you project. Maybe you are trying to get around the first premiss of the KCA (whatever begins to exit has a cause) by arguing that the universe does not exist?

      Now in the last paragraph you actually make the same point that I made earlier. Under materialism there is no mind in the classical sense. No free will, no agency, no reliable reasoning. Under materialism they are ineffective as they cannot move anything and per your definition they do not even exist because they have no shape or form and cannot be located in space. So that would leave me mindless.

      Your last quote by Pinker hearkens back to Hume’s interpretation of the universe as a brute fact. So again we agree. Under materialism the universe is there for no reason, it has neither point nor purpose.

      So in the end we inevitably arrive at nihilism, as promised. I am mindless, the universe is mindless, there is no other entity. Hence, there is no meaning and no purpose. It’s just ‘one damn thing after another’.

      Now let me ask you: Even if you came to the conclusion that this was true, can you live it?

  67. It seems to me that if one is going to discuss the issue of free will in a meaningful way, it wouldn’t hurt to be familiar with the experimental evidence and in a position to comment on same. There is a useful summary in Wikipedia at: https://en.wikipedia.org/wiki/Neuroscience_of_free_will

    and a more recent experiment described at:

    http://www.independent.co.uk/news/science/free-will-could-all-be-an-illusion-scientists-suggest-after-study-that-shows-choice-could-just-be-a7008181.html

    and beyond that, one has to provide some sort of explanation as to how this supposed cause ab initio works, how it relates to physical objects, what the evidence is that supports the claim that it exists, how is energy conservation affected and a host of other issues that need to be resolved. Simply saying one exercised free will by picking up a coffee cup is not sufficient – all one has done is show a correlation between intending to pick up the coffee cup and your arm actually picking up the coffee cup – especially in light of the experimental evidence that clearly demonstrates that at least some of the time we are deceived in making the judgment that our actions were the result of our will or that w/o our knowledge our will was compelled.

    As far as your discussion with CatholicScientist regarding the reality of the universe, for what it’s worth, I agree with CatholicScientist’s first paragraph – at least more or less. Quantum field theory has replaced quantum mechanics but the idea that the universe is limited to objects having form and location would not in my view be particularly compelling or that there is some sort of distinction between talking about the universe that doesn’t exist and a human body composed of numerous other items that somehow does. I don’t think we can rule out disembodied spirits simply by defining them out of existence – one has to look and see if there’s any evidence one way or another. I don’t think we can rule out that our universe has more than 3 dimensions simply by arguing that one can’t visualize such additional dimensions.

    “Under materialism the universe is there for no reason, it has neither point nor purpose.” Certainly none that we are aware of or which can be ascertained by examining the universe at large. We have a variety of myths that make a claim that the universe was put here for some purpose by some anthropomorphized being but evidence is lacking for any such claim. And why should we accede to such purpose. How does any conceivable action that we take have any relevance to what’s happening right now 3 or 4 galaxies away? And why should we assume a deity’s purpose matches our own purposes – there’s a recent parody by John Zande setting out the evidence that we have a deity that is omnimalevolent – check out The Owner of All Infernal Names; An Introductory Treatise on the Existence, Nature & Government of our Omnimalevolent Creator. In the HitchHiker’s Guide to the Universe, the Vogon’s may have had a good & compelling reason for blowing up the Earth – but that’s cold comfort to its inhabitants.

    It doesn’t stop we ourselves from having purposes & making moral judgments and aspiring to leave the world a better place – and so we can clearly reject the notion of nihilism no matter how often that CatholicScientist wishes it were so.

  68. Hi Guys,

    To speak of something rationally requires that we at least agree on what it is exactly of which we speak.I defined my terms/words which you have chosen to ignore. Please then give us definitions you would rather use.

    So here it is:
    Definition Exist ……………………………………………………………………………………………………………….
    Definition concept ……………………………………………………………………………………………………………
    Definition object……………………………………………………………………………………………………………….

    That simple, all we require is a definition that we can use consistently. Only a dishonest person would refuse to define the terms they use to justify an argument, or someone who doesn’t know what they are talking about.

  69. Sean, there is no short answer to your request for these definitions. The answers depend on your ontological position. In Catholic theology existence and God are linked and it is often stated that God is existence itself and there is no existence without God. If this interests you will find it explained by scholastic philosophers like St. Thomas Aquinas or St. Bonaventure. However, this is Eric’s blog and this would take us far off topic and I also don’t want to use it as a discussion platform for Catholicism. Also, the chances that the exchange would be fruitful are quite low, so I will end it here, if you don’t mind. All the best.

  70. Hi David and Sean,

    As I said a few posts ago this week is extremely busy for me so I apologize for being MIA but its just a fact of my life at the moment. I won’t attempt to answer everything brought up but will do my best to respond to what stood out to me.

    Btw, just want to say I’m highly impressed with both of you, your care about this subject, and your civility/level headedness. It’s a relief. Very few atheist-materialists or Christians that can undergo so many volleys without becoming hot under the collar, so thanks.

    Okay, Sean, I was hoping I didn’t have to define words like “universe” given the context that I set it in. By universe I simply meant what nearly anyone means: the total aggregate of all matter/material in existence. That’s all. It is all contingent by sheer fact that nothing brought itself into existence – nothing is eternally self-sustaining either. The laws of thermal dynamics apply to a closed system. Energy is not created or destroyed but it was created at some point, or at least the weight of nearly every corner of science at present seems to indicate so. The second law would be in jeopardy if this was not true. If the universe was eternal everything would have already achieved maximum entropy. How do I know everything is contingent? I don’t “know” everything is, but it is a logical deduction from every observable phenomenon we’ve encountered so far. I’ll answer the quantum question when I address David. The argument of contingency is fairly well known and, as far as I can tell, is fairly tight. Back to the OP, if I were an atheist this is the thing that would make me lose sleep at night. I understand if it doesn’t trouble you, but it would me.

    David,

    Thank you for the links. I was able to read through a good portion of the first one and came across many things I was already familiar with. Particularly the studies that show that one is already in process of making a movement up to 7 seconds before it reaches consciousness. These studies are interesting, but not terribly devastating to my position. As a psychotherapist I have studied much in the areas of conscious-unconsciousness, neuroscience, and sociology. I understand that man is conditioned by many factors – neurological, biological, environment, etc. The issue of free agency is not so much that man is conditioned in certain ways but that he has a choice with what to do about his conditioning. To quote Viktor Frankl, “Man’s freedom is no freedom from conditions but rather freedom to take a stand on whatever condition might confront him.”

    Man is not a lab rat or a dog that can be conditioned to behave a certain way and that’s that. Man can transcend himself and freely choose what he will do with his conditioning. I see it on a daily basis in the therapy room. I work with teens and adults who have been conditioned by genes, drugs, neglect, abuse, you name it, but they all retain an internal locus of control by which they remain responsible for what they will make of their conditioning. I would not expect such responsibility from a rat.

    The fact that a neuroscientist can predict with 60% accuracy what choice a person will make to push a button on his right or left seconds before the person becomes conscious is pretty cool. But what is at issue with this discussion of free agency has nothing to do with such things. It has everything to do with how a person comes to belief, ethics, morals as a matter of personal history – a history spread out over long intervals of time, in which one constantly transcends their own conditioning to think a certain way and form beliefs. This is what the Christian faith is concerned with: not does man have perfect free will and full control over his every last behavioral aspect, but is he/she ultimately in a position to choose whether or not to follow God or to become their own god (or any number of avenues in between).

    Your link reminded me of a study I read about some time ago, the one where psychologists tested students to see how likely they were to lower their moral standards after being influenced by the tenet of materialism. The study revealed that students influenced by materialism prior to the test were much more likely to cheat on their exam than the subject not introduced to materialism. This is a great example of free agency – the cheating students made moral choices in light of the teaching that nothing ultimately matters (i.e., materialism). They rose above their prior conditioning and adopted a new life philosophy based on reasons they were given, not based on their physiology. If they did not choose to loosen their morals and instead we blame it on some antecedent physical condition then I think we’ve created a modern myth worthy of sharing shelf space with Nordic and Greek mythology. If we blame in on biology then lets quit the name game of “good, evil, moral, immoral, should, shouldn’t.” It’s confusing and promotion of fairy tales. Quit telling your wife you love her as you turn out the lights at night. Tell her that chemicals in your brain are causing you to feel warm towards her and giving you the illusion that she matters. Besides, its far more romantic.

    As far as quantum field theory, I have read exactly one book in my life on quantum physics, something by the title of “The non-local universe”. Incredibly interesting read and I know the field has grown much since the publishing of that text. I do not claim to be learned in every possible science. Each specialty requires years of study to become acquainted enough with it to say much about it. My specialties are in theology, history, and psychology with smatterings of philosophy and neuroscience. The best reply I have to your post is that I’m pretty sure everything on a quantum level is still subject to contingency, unless one posits that the quantum world created itself out of nothing. But, I may be surprised.

    Thanks again for the banter gentlemen. If you care to suffer with me and my schedule I’m still game to volley some more if there is stuff we haven’t already butchered to death.

    Cheers!

  71. Hi Eric – thank you for the compliments. I enjoy philosophical discussions on the merits w/o resort to name calling or appeals to emotional tribalism that are able to cover the topic in some detail. I have a background that includes an undergraduate degree in philosophy specializing in formal logic and the philosophy of science. It was a long time and so I don’t claim to be current. I also am fascinated by modern physics including the special and general theories of relativity and quantum mechanics (now quantum field theory) and cosmology and read as much as I can on these topics. If you have the time and are prepared to do some modest homework including viewing some youtube links, I think we can bring you up to speed in an overview sort of way as to why scientists hold the views that they do & what those views are. So I’ve enjoyed your blog – I like the freedom to make an argument that people actually read notwithstanding that it’s somewhat longer than, say a typical facebook post.

    Our topic in this thread is now officially very broad – I could make relevant responses in the fields of ethics, physics , psychology and philosophy of science so it’s somewhat daunting to choose the most effective method of proceeding.

    I’m not fond of CatholicScientist’s argument from nihilism – it’s not true and the promulgation of this argument from the pulpit has had a profound effect on the lives of atheists all over the world including in many African and Middle Eastern countries the putting to death of atheists and in the US heavy discrimination against atheists. It might be interesting to provide you the currently prevailing humanist version of the 10 commandments which is noteworthy in how it differs from the biblical one:

    1. Be open-minded and be willing to alter your beliefs with new evidence.
    2. Strive to understand what is most likely to be true, not to believe what you wish to be true.
    3. The scientific method is the most reliable way of understanding the natural world.
    4. Every person has the right to control of their body.
    5. God is not necessary to be a good person or to live a full and meaningful life.
    6. Be mindful of the consequences of all your actions and recognize that you must take responsibility for them.
    7. Treat others as you would want them to treat you, and can reasonably expect them to want to be treated. Think about their perspective.
    8. We have the responsibility to consider others, including future generations.
    9. There is no one right way to live.
    10. Leave the world a better place than you found it.

    One key difference between biblical precepts and humanist/atheist precepts is that to the best of my knowledge nowhere in the bible are intelligence and the critical examination of evidence treated as virtues – in fact one could argue that the exact opposite would hold true. Rather than responding to this post, please leave the topic with me for a few days so that I can provide a more comprehensive response that is actually specifically directed at the issues you’ve raised.

    And finally, the theory of special relativity is one that is readily understandable by the general public and there are a number of excellent video links – it’s easy to explain if one has a piece of paper and can draw pictures which is why the youtube videos are particularly helpful. In quantum theory, you can go onto youtube and search for Professor Quantum for some useful simple animated explanations of basic features of the quantum universe. It would be my recommendation that you start with the double slit experiment. I think the effort will repay dividends.

    I’ll mention 1, well maybe 2 or 3 quick points in passing – for reasons that I can explain, gravity represents negative energy – and there is a fairly compelling hypothesis/theory that the net amount of energy in the universe is precisely equal to nothing. Similarly, the net electrical charge is nothing. The net angular motion is said to be equal to nothing. Which if true might surprise you. And a second thing that might surprise you is that quantum fields are always in flux as a result of Heisenburg’s uncertainty principle – if a quantum field was & remained exactly 0 for an extended time period, then that would mean that we could have exact knowledge of the value of the field – which is inconsistent with Heisenburg’s principle. And that’s why the vacuum is continually creating quantum particles – and the explanation for Hawking’s radiation is that whereas in normal space – quantum particles will be created – or rather a particle and its antiparticle are created from the energy of the vacuum but will normally encounter each other and then destroy themselves – in the case of a black hole, 1 of the created particles may disappear into the hole whereas the other particle may escape – leading to a constant erosion out of the black hole.

  72. Hi Sean

    Off the top of my head, possible definitions of exist:
    1) that which responds to gravity
    2) that which is real
    3) that which is a quantum field or an excitation of a quantum field or composed of such excitations
    4) that which is capable of interacting with objects
    5) that which has positive or negative energy
    6) that which is composed of fermions or bosons – this last one needs work as dark matter and dark energy are likely composed of neither

    Another answer is that we can define “exist” by examples – this tree, that cat, that bird, this grass, … and so on. As long as we can communicate and understand each other a precise definition is not required. Sometimes we communicate and context matters. If one is running a high speed particle collider, it’s sufficient to take the position that what one is observing (the various paths and stuff that is setting off one’s detectors) are actual objects that exist. For different purposes, one might reject the view that they are objects at all.

    All definitions are somewhat arbitrary and we choose them for their utility – I have a pretty good idea where your choice of definitions leads and it’s to the rejection of mainstream science on the basis of a choice of definitions as opposed to based upon observations and mathematical analysis.

    A concept is an idea or a thought. An example of a concept is a klein bottle which is a 2 dimensional bottle twisted in 4 – dimensions.

    I don’t have a huge problem with your definition of object – although in fairness any sizable object in fact occupies locations and not just a location. And it’s not clear if a location is actually an objective feature of the universe – rather it’s a location relative to some other locations. And there’s a problem with shape as the boundaries of objects are somewhat undefined. For example, the length of the coast of England varies markedly depending upon what scale one uses. And a chair viewed at the level of atoms has a very different shape than it does when viewed at using our ordinary every day vision. And of course, the shape of an object will vary depending upon your frame of reference – how fast one is moving relative to the object in question – and according to Einstein, each frame of reference is of equal merit. And what about time – does your definition of object have any temporal requirements – because the special theory of relativity establishes that there is no universal “now” and if one observer sees, for example, events separated some distance apart in the sequence, A. B, C, some other observer moving at a different velocity will see them, for example C, B, A and every sequence in between – and so a location in space-time is important. And when we get to the shape of quantum particles or even agreeing to the count of atomic particles – well, there’s problems there too – because that may well depend a bit on how it’s measured. An electron is more of an electron cloud, when it is not being observed spreading through space – and surrounded by virtual electrons that screen the actual physical electron. And of course a quantum particle may well find itself in 2 places at once. Are you familiar with the concept of the collapse of the wave function and entanglement – because that’s going to cause your notions some problems as well.

    I think your questions don’t advance the argument as much as you think, Sean. You’re attempting to use definitions in order to restrict what we might consider to be real – whereas it makes more sense sometimes to figure out what we consider to be real based upon evidence, observations and our best theories in order to arrive at useful definitions. Whereas we could once define swans as being certain type of white birds having certain properties, it subsequently became more sensible to include some black swans – as that is what our experience suggested to be wise. It’s kind of the reverse of the ontological argument – which attempts to use definitions to add God to what we would include in our reality. Neither approach is legitimate.

  73. Sean,
    I ran across a resource that you might find interesting or helpful.
    Watch the youtube video:

    The first 20 minutes should do it (although the rest is good as well (in my opinion) and the webpage that the speaker mentions.
    This may clarify some of St. Thomas Aquinas thought and the discussion about a single unrestricted first cause.

  74. Hi Eric – our topic seems to become more challenging the longer the time gap between posts – as one tends to forget what has already been covered or that which needs additional emphasis – posts that I’ve intended but abandoned as the topic subsequently developed or other matters pressed. And of course, the topic has become overly broad and one is forced to choose between a broad but somewhat superficial response or a more focused one. And of course there are quite a few points of disagreement that I would love to delve into. I’d been thinking that it may be time to wind down the discussion – and I’d come back another time as your topics of your blog generally interest me – but CatholicScientist seems to wish to continue and so, perhaps, we should have at it. I’ll stick with 1 topic for this post – but I remain enthusiastic about touching in the near future with some of our other points of disagreement.

    DUALISM / FREE WILL
    I’m quite familiar with the experiment with materialism that you discussed – it’s part of applied games theory, probably the dictator game or the ultimatum game. You’ll be pleased to learn that theists tend to be viewed by theists and atheists as more trustworthy – at least that’s my recollection of the relevant study. Interestingly, framing the subject in advance with religious ideology in advance of the experiment will make theists more trustworthy in these cooperation games but not, as best I recall atheists. Framing subjects in advance with law or civil rights ideology will make people raised in western democratic societies where rule of law is important more trustworthy but has no impact on those raised in other societies. This increase in trustworthiness from referencing secular authorities is pretty much equivalent to the effect that theists get from some reference to God. In secular societies, God is replaced by good government and when we compare the viability of religions in wealthy western communities that are egalitarian where religions are visibly failing vs countries with significant wealth disparity or poverty and lack of education where religions are growing And, also interesting, I think: if you have what appears to be eyes looking at one, even very symbolic ones (or the experimenter regularly drops by to supervise even though the experimenter has no role to play in the playing of the game), honesty and trustworthiness goes up.

    None of which of course plays any role in whether or not our beliefs are true. In the grand scheme of things, in my view what makes our beliefs more likely to be true is that they are representative of a model of the universe capable of making accurate predictions. The ability to make accurate predictions is unlikely to be explained by chance. If you can routinely predict what is going to happen, then generally one is likely to know something about the antecedent circumstances giving rise to what is about to happen. We believe we can rely upon our memory by reason, that for example I can use my memory to predict where I’ll find my keys. But I know my memory is not totally reliable – for obvious reasons.

    “Man can transcend himself and freely choose what he will do with his conditioning. I see it on a daily basis in the therapy room. I work with teens and adults who have been conditioned by genes, drugs, neglect, abuse, you name it, but they all retain an internal locus of control by which they remain responsible for what they will make of their conditioning. I would not expect such responsibility from a rat.”

    I don’t dispute your evidence – however I think the inference from that evidence that man can transcend himself is not compelling. In my perspective either a) the internal locus of control is an illusion but nevertheless is consistent with your evidence or b) we have an internal locus of control that is not an illusion but is explicable in theory in terms of the functioning of the brain is consistent with the data. Both theories are consistent with the data. There is a 3rd option (dualism) which can’t be ruled out conclusively but which certainly seems significantly less fruitful as an hypothesis – things like tumors causing a person to become violent or a disease that causes them to swear at everyone or the creation of 2 personalities by the splitting of the connection between the left and right brain – these seemingly suggest that these are defects in the brain that lead directly to defects in the soul – but how can that be if the soul is something transcendent? i think it’s fairly apparent that the brain is sufficiently sophisticated and complex that it would be perfectly explicable that it could give rise to complex behaviour other than what one might at first anticipate – such as the examples that you’ve given. Would the soul theory have lead to the prediction that scientists would be able to predict choices 7 seconds before a subject became aware of it? Would the soul theory have predicted that scientists can create images of what a subject is currently imagining based upon brain patterns?

    As best I know, most neurobiologists are not dualists – and the distinction between human reasoning and animal reasoning becomes smaller and smaller. Our brains function very much like our animal predecessors and in the case of mammals share similar structures and designs. The hard problems is “what is consciousness” and it appears that everyone is in agreement that animals are conscious and some would appear to have a sense of self as demonstrated by their ability to recognize themselves in a mirror. If you google “mouse altruism” one finds a number of studies which explore interesting precursors of morality. And if you investigate the evolution of consciousness, the theory pretty much predicts what we actually observe – that with the development of more and more sophisticated brains and neurological structures, one also observes more and more complex behaviour that is more and more sophisticated in demonstrating an understanding of the world around it. I suspect consciousness arose early in the development of life – in recent studies, fish have been shown to experience pain, to have personalities and to be able to recognize human faces – pretty good for a fish, I should think. The personalities don’t have much to write home about – some fish are brave and some cowardly which has resulted from different life experiences as young fish & there’s not much more. I think the idea that fish or small mammals or crows or dolphins or giant squid have access to magical conscious interlopers seems far fetched compared to a secular explanation. It doesn’t seem likely that there is a separate conscious mind somehow seeking out its relevant species and somehow (magically) connecting – to make the claim plausible one would want this explanation to be well supported by evidence. Scientists are currently engaged in developing hardware and software to completely mimic the neural structure of the human brain – it will be interesting to see the results. I’m curious – if in our lifetime, a computer became sentient – what impact would that have on your views?

    At the end of the day, I have no skin in the game – I’m perfectly happy to follow the scientific evidence where it leads – and to change my mind as and when the evidence supporting 1 position or another changes. I think it difficult for theists to maintain such an open mind – for them the results are fixed in advance of experiment – if one doesn’t have free will, then the deity that purportedly supplied it does not exist and we have a useful test for the God hypothesis. If one does have free will- then the games is still afoot as free will may well simply be one outcome of certain physiological structures and feedback mechanisms or dualism may hold but not a deity or we simply lack sufficient understanding of what is going on.

    Experiments attempting to ascertain whether or not prayers actually get answered is a way of testing for certain versions of Christianity – namely those versions in which God is said to answer prayers – so it’s not as if religion gets to have a separate magisteria or is exempt from critical scientific investigation no matter how devoutly that theists might wish it were so – science is more than capable of studying religious views, superstitious views, invisible stuffs (such as dark matter or dark energy or quantum fields or the top quark), basic physics, psi phenomena, etc. And given that certain religious views appear a) to be held very dogmatically and b) to be immune to rational discussion and c) are pernicious and nasty – that’s probably a good thing. The secular community takes a certain amount of satisfaction in moving the religious community slowly but surely away from historical viewpoints that in retrospect were not particularly moral.

    And CatholicScientist – unless Thomas Aquinas had access to current scientific theories regarding physics, time, space & cosmology, (which are far removed from what our naively informed senses might suggest) I’m dubious that his argument would have much merit other than it’s literary or historical value. Clearly scientists have a considerable number of ideas as to how our universe came into existence – it’s more a matter of identifying which one is true as opposed to which ones are merely plausible.

  75. I apologize for monopolizing the bully pulpit – but I don’t think my last post is likely to lead to an interesting discussion – whereas this post might.

    In old fashioned terminology – there are weak atheists – whose claim is simply that in the absence of evidence, there is no reason to believe in deities of any sort. Given that there are infinitely more things that don’t exist than do exist, the default rational position is to actively disbelieve in some proprosed entity rather than to be agnostic – a position that is of course subject to being corrected given sufficient justification. This type of atheist is simply putting the theist to the onus to provide compelling & persuasive evidence that God exists and suggests that this onus has not been met.

    A strong atheist is one that accepts an additional onus of developing a persuasive argument that God or any other deities likely don’t exist. The theist opponent usually is atheist as towards Zeus and Aprhrodite and all of the gods and deities ever believed in by humanity and all of the other logically possible deities (so we’re off to a good start) and the argument is generally whether or not some version of the Abrahamic deity exists. Notwithstanding polls suggesting that most Christians in fact believe in a anthropomorphic superman style deity with more polished manners than the one described in the Old Testament, Christian apologists generally tend towards a more sophisticated deity.

    The general properties ascribed to this more abstract deity include some variations on a) necessary being b) creator of the universe c) perfect being not unlike what Plato might have proposed (or sometimes as perfectible as possible in each possible quality) – d) source of morality e) representative of the highest possible morality and e) has some sort of intelligent albeit unintelligible plan for the universe and for us. I’m not sure to what extent the deity you espouse holds any of these properties or whether or not you are in fact an atheist with respect to deities with any of these properties – I’m curious to know.

    As I understand it, and I’m quite certain that I don’t fully understand it yet, your deity represents some sort of underpinning for the universe and is the ground of all possibility. And the obvious question is: why is any such ground necessary? Why aren’t objects sufficient in and of themselves to ground possibilities, at least from an a priori perspective? And if there is some sort of ground necessary, why should one assume that such a ground is a deity rather than something else. I should think a deity needs must be something somewhat lifelike and intelligent in order to warrant the name – and that a quantum field would not qualify as a deity even though from a scientific perspective a quantum field may be all that is required and we have very good reasons for believing that such fields exist and fill all of space. Eternal inflation would be 1 explanation of why we have a universe at all – a 2nd explanation would be the ekypyrotic universe, a 3rd explanation would be based upon string theory and the collision of branes, a 4th would be a vacuum fluctuation – and so on.

    A claim that something is necessary is traditionally understood as being a claim that it is true in all logically possible worlds. That understanding is spelled out in the Stanford Encyclopedia of Philosophy article on the subject of logically possible worlds. A logically possible world is any world, real or fictional that can be described in a logically consistent manner. For example, if we assume that some proposition is true and we also assume that p implies q, then q is true – and that relationship holds in all logically possible worlds. The claim that Santa Clause exists in some logically possible world merely means that the concept of Santa Clause is logically self coherent – and makes no claim as to the actual likelihood of Santa Clause.

    A claim that God is a necessary being is a claim in effect that God exists in all logically possible worlds. It’s a claim that most Christian apologists want to shy away from because it’s a claim that is obviously untrue. A logically possible world might consist solely of a chair or a table pr a hydrogen atom – ie no other items other than these specific ones – and there are an infinite number of logically possible worlds that “exist” in which there are no deities of any kind – and so we draw the conclusion that God is not a necessary being. And if God is not a necessary being, then that removes one of the most fundamental planks apologists use to establish that God is required in order to provide an explanation for where our universe came from – because if God is not a necessary being, than he/she/it/? is presumably a contingent being. The argument that God is a contingent being but nevertheless responsible for the creation of the universe is a scientific claim that to date has not be found to be particularly meritorious amongst the scientific community, especially amongst physicists and cosmologists.

    There are other serious problems in the God proposal – a perfect being is quite evidently self contradictory – is he/she/it a perfect square and a perfect circle? Or more realistically from the point of view of religions – does God take fairness to its ultimate degree or does God take mercy to its ultimate degree – because those conflict. And why is the universe so different than what we would expect given its creation by a perfect being – because the universe sure looks imperfect and does not show any signs of purpose? An answer that suggest that in addition to being perfect, he’s also mysterious would seem to be more of a rationalization for a failed theory rather than a serious scientific hypothesis. And if our minds are too imperfect to understand God fully, that is a misdirection – God seemingly has no difficulty in letting us understand how his universe works in some considerable detail – but remains mute and incomprehensible when it comes to his purpose? How does that facilitate some justification for following the 10 commandments or praying? How is any such deity a fruitful hypothesis to warrant any sort of scientific investigation as to aspects of reality?

    If I’ve underwhelmed in my understanding of the Abrahamic gods or the version that you represent, I’m perfectly content to be enlightened.

    some useful quotes that I think relevant to our discussion:

    “I want to put on the table, not why 85% of the members of the National Academy of Sciences reject God, I want to know why 15% of the National Academy don’t.”

    Great scientific minds, from Claudius Ptolemy of the second century to Isaac Newton of the seventeenth, invested their formidable intellects in attempts to deduce the nature of the universe from the statements and philosophies contained in religious writings…. Had any of these efforts worked, science and religion today might be one and the same. But they are not.

    Whenever people have used religious documents to make accurate predictions about our base knowledge of the physical world, they have been famously wrong.

    I simply go with what works. And what works is the healthy skepticism embodied in the scientific method. Believe me, if the Bible had ever been shown to be a rich source of scientific answers and enlightenment, we would be mining it daily for cosmic discovery

    We should not be ashamed of not having answers to all questions yet…I’m perfectly happy staring somebody in the face saying, I don’t know yet, and we’ve got top people working on it. The moment you feel compelled to provide an answer, then you’re doing the same thing that the religious community does: providing answers to every possible question.

    “I am convinced that the act of thinking logically cannot possibly be natural to the human mind. If it were, then mathematics would be everybody’s easiest course at school and our species would not have taken several millennia to figure out the scientific method.”

    “The Universe is under no obligation to make sense to you.”

    all above by Neil deGrasse Tyson

    “The Universe is not only queerer than we suppose, but queerer than we can suppose.”
    J S Haldane

    As far as the laws of mathematics refer to reality, they are not certain, and as far as they are certain, they do not refer to reality. Albert Einstein

  76. Hi David, forgive me. I didn’t respond to your last post because I didn’t see anything that was requesting a response per se. I am still around, peeking in, surveying and responding as needed and as able. This last post of yours is certainly weighty. Not only is there a lot of content but to respond to the items of most interest to me would involve a good degree of theological educating. For example your question about how could God be a “perfect being” and then giving examples of a perfect square or circle. This reveals a fairly serious deficit in understanding not only the Orthodox Christian revelation of God but classic theism in general. That’s not in any way a put down as the good majority of Christians are also out of the loop in such matters. In short, God is understood to be beyond being. He is not conceivable as one conceives of any object. Orthodox hold an apophatic theology about God’s essence. What we do know through revelation – Trinity, nature of Christ, etc.,- is made available to have a definition of what God is not, but not what God “is” except to say He is spirit. Christ revealed God in His incarnation, but the essence of God is unknowable to our minds. However, there is a difference made between God’s essence and energy. It is through God’s energies that we have full contact with Him, are united with Him by grace, made partakers of his nature, etc., i.e., the doctrine of theosis or deification (Orthodoxy soteriology). I don’t intend this to be an exhaustive answer to your question but just an introduction to state that there is much more than meets the unsuspecting eye when speaking of God’s “being” in usual parlance.

    I’m interested in this as well: “A logically possible world is any world, real or fictional that can be described in a logically consistent manner.”

    I would love an example you might have of a logical account of a possible world that can be given without a cause and/or creator – without that “necessary” thing by which everything else is contingent. This is where I believe the argument is so strong (and I’m not sure what Christian apologists are trying to steer clear of it) but I’m willing to be changed. It seems that a material world without a cause would at least need to be eternal, or somehow transcending time (nevermind the physio-spacial impossibility of space without time). Trouble is that our real life physical universe, according to the best science we have, had a starting point. Sorry if the argument has yet again swung through the revolving door but there has been no answer to this even with all the cyberspace consumed in our posts.

  77. David, let me take a quick swing at these as well:

    “why is any such ground necessary? Why aren’t objects sufficient in and of themselves to ground possibilities, at least from an a priori perspective? And if there is some sort of ground necessary, why should one assume that such a ground is a deity rather than something else.”

    1. such a ground seems necessary since its more difficult to believe that everything suddenly appeared from nothing with no cause. We also have no a priori nor a posteriori reason for believing that anything in existence is self-created and independent from everything else.

    2. The difference between existence and non-existence is an unbridgeable chasm (Kierkegaard had a great term for it, but I can’t recall it at the moment). It that’s even what you were getting at with the question.

    3. I personally assume this “ground” is God due to the teleology weaved throughout the fabric of existence, for one. For two, I believe in the incarnation, death, and resurrection of Christ. That statement is wholly unscientific and I hope to keep it that way. There is more explanation emanating from this theological proposition than anything science could ever give about existence, not because science is bad but because it is inherently limited. It studies some things, not everything. For three, I have no reason whatsoever, in light of the fact that consciousness exists, that the cosmos are essentially non-rational. The gift of consciousness and reason speak to something bound within nature that far exceeds mere physicality. I would be unable to argue for the opposite, that the cosmos are non-rational at base, though perhaps someone else could. Again, Thomas Nagel produced an excellent treaties on this from a non-theistic point of view that is worth the read, which I’ve already suggested.

  78. Hi Eric – you expressed an interest in following up with the logically possible worlds in particular – so let’s start there. I take it to be obvious that long posts are something of a fatal flaw of mine – I can only apologize once again. Please just pick the topics you are interested in – I will assume you disagree on the ones you are silent on. We can always revisit them later – should time and interest permit.

    Logically possible worlds

    A universe containing a clock that pops into existence, the clock ticks for 10 minutes and then the universe ceases to exist is perfectly intelligible and logically consistent. There are no axioms of formal logic that conflict with this universe. We don’t have to assume that there is a cause or explanation for this unprincipled behaviour by reason that neither our universe or universes in general do not owe us an explanation. If God exists (in the absence of some proof that he/she/it/? is a necessary being, than either God is the result of some previous cause or God’s existence is simply a brute fact. Perhaps in lieu of a universe containing a clock that pops into existence, there is another clock universe creating universe that is the brute fact (or necessary entity).

    Let me request that you provide me a word picture of a world in which the law of identity doesn’t hold – in other words, p = p isn’t true. Or a world in which if p implies q and if q implies r, that somehow p doesn’t imply r. Or a world in which if p is true and q is true, it’s not the case that p and q are true. Logical arguments are vacuous or tautological in nature – they don’t tell us stuff about genuine properties of the world – but they do limit how we are going to meaningfully describe them. I’m guessing that you are utterly unable to present any viable or sensible picture of a world in which classical logical principles are violated. We’re not going to find a world in which all men are mortal, Socrates is a man and yet Socrates is immortal. We’ve violated our basic linguistic principles in making any such claim and any such claim represents an obvious self contradiction. And avoiding self contradictions is at the heart of logically possible worlds.

    There are other logical systems other than classical 2 valued logics. As the name suggests, 3 valued logics have 3 truth values – perhaps True, False, Meaningless would represent the 3 values – and a formal logic would then explore how these truth values interrelate. In our own universe, we have quantum theory in which a quantum particle can, for example have spin up, spin down or a superposition of spin up and spin down – which is at least suggestive that we can sometimes benefit from a richer logic system than the simple 2 valued logic.

    Causality & Conservation laws

    You wish to introduce axioms that are not normally part of formal logic systems – you’re welcome to do so – but I don’t have to accept them as genuine axioms – which I don’t. Nor do I have to accept them as true. Seemingly, you wish to introduce some sort of law of universal causation – that for every event (effect) there is a cause – and then you wish to reject that universal principle and introduce the notion of 1st cause (n the case of the start of the universe), then you presumably wish to introduce the notion of some sort of cause that had the effect of causing God to create the universe (I know you actually don’t – you’re going to claim that God is special and not subject to universal laws and so therefore the laws are not in fact universal) and you also wish to introduce the notion that humans are capable of violating this universal law of causation by initiating new causal chains by uncaused acts of will – making the whole notion of a universal cause somewhat problematic given the numerous exceptions.

    In our actual world, quantum physics works on the principle of probability – causal relationships form no part of our description of nature at the quantum level. Now scientists do very sophisticated experiments that have given rise to their conclusions and there is some interesting and elegant mathematics supporting their claims which are noteworthy for the basic principles that they rely upon – namely symmetries which is a pleasing foundation in explaining how reality works. There are some quantum tests you can run at home – if you look at light streaming through a window on an angle for example – some of the light will penetrate the glass and you will see o/s the window and some of the light will reflect and you will see an image of inside the window – both at the same time – and which light photons reflect and which light travels through the glass is entirely probabilistic & it is impossible in principle to predict which will be which. When you hear a geiger counter clicking, it’s picking up signals from the random decay of particles – a given object (say composed of uranium) may be unstable and will have a half life of say 10,000 years. So that in 10,000 years half the particles will have decayed into something else – but which half is utterly unpredictable. Unlike the traditional view of the flip of a coin in which someone sufficient well versed in physics and the surrounding circumstances could supposedly predict whether or not the coin would land heads or tails, no such prediction is theoretically possible in quantum theory.

    You also wish to introduce a mandatory law of conservation – that things can’t spring out of nothing which I think is probably related to your causality views. However our acts of free will spring out of or at least are independent of all other aspects of the universe according to you. Your proposed viewpoint does not represent axioms of logic -in the real world we do not get to instruct the universe how to behave – we don’t get to make up rules and insist that the universe behave accordingly. In the 50’s and 60’s, the idea of a steady state universe was one of the dominant theories of cosmology – in which the universe was expanding but nevertheless staying the same – and new matter was being created at a very slow rate so that density of matter in the universe remained the same. That’s a perfectly plausible and logically consistent theory – and there is no requirement that there be some deeper explanation as to why the universe behaved that way – if the universe was a steady state universe – it could be that such was simply a brute fact with no further or deeper explanation – or with luck, we might find a deeper explanation which itself was simply a brute fact. The universe does not come with any guarantee that explanations are even in principle possible. The idea that some property of the universe is unknown or perhaps in principle unknowable does not lead to the conclusion: therefore God – that’s an appeal to ignorance and is a known fallacy.

    It appears that according to cosmologists, energy is not conserved in our universe by reason that the universe is expanding.

    http://www.preposterousuniverse.com/blog/2010/02/22/energy-is-not-conserved/

    The author here is a well known populizer of science. I was in fact going to recommend that if you get a chance, please watch the videos he did on the Higgs Boson on behalf of the Great Courses. The Great Course are introductory university level videos offered by exceptional teachers on a wide variety of subjects including physics, cosmology, religion, science in general, history, literature, mathematics, etc.

    “It seems that a material world without a cause would at least need to be eternal” That would be one of those bald assertions that we’ve discussed – I see no reason to assume this proposition to be true. In fact the very notions of time and space in the framework of quantum theory is very different than what you imagine – as you approach planck time and planck space, the very notions of time and space become problematic – space becomes more time-like and time becomes more space-like and at the end of the day space-time becomes a sort of froth with before and after and locations turning into a complete muddled mess.

    Time and Space

    “nevermind the physio-spacial impossibility of space without time” – why would that be? All one has to do is remove all of the general arrows of time. In fact in quantum theory, there are very little in the way of temporal direction and generally time can be conceived of as flowing in either direction. A positron could be conceived of as an electron travelling back in time, for example. In the world of our experience, time flows in 1 direction – an egg that is broken never leaps off the floor and reassembles itself – but that’s not the world at its most fundamental. One plausible interpretation of the general theory of relativity is that time and space exist as bulk entities and where we are right now is at a specific instance of space and time no different in principle than another specific incident of space and time 100 years ago or 50 years in the future – the idea that there is a moving present taking us from the past and into the future is controversial. In a black hole – time stops. And time does not pass for a light beam. And time slows relative to an observer the faster one moves.

    Cosmology

    “Trouble is that our real life physical universe, according to the best science we have, had a starting point.”

    yes and no – certainly the universe underwent some sort of phase transition in the big bang 13.8 billion years ago. Can we rule out that the universe existed for ever but simply changed at that point in time? No, we can’t. Or alternatively, there are other universes that “preceded” ours that gave rise to ours. An intriguing possibility is that our universe is somewhat custom designed to create black holes – the physics of our universe tends to maximize the size and number of black holes and so (wild speculation here, but not one that originates with me) what if black holes in our universe linked up with white holes in other universes and were responsible for kickstarting other universes – and that there are evolutionary pressures leading to more and more universes capable of creating black holes – and that universes containing black holes it just happens to be the types of universes most suitable for creating life. Or we have some physical principle that happens to create universe such as eternal cosmic inflation (there are pretty good arguments for this one). The theory of inflation which has not yet actually been confirmed as a scientific theory as fully proven is on the verge and is studied seriously by scientists throughout the world and naturally gives rise to more and more universes. Scientists expect to have solid proof of the theory of inflation within a few years, principally from examining the cosmic microwave background. The original impetus for the theory of inflation was not to answer questions of cosmology but rather a reasonable conclusion drawn for particle physics – but it provides a broad range of answers to what had previously been perplexing questions of cosmology.

    ” such a ground seems necessary since its more difficult to believe that everything suddenly appeared from nothing with no cause.”

    that would be an appeal to personal incredulity or an argument from ignorance, both fallacies. Having said that, there are numerous alternatives.
    The Role of Science
    >>because science is bad but because it is inherently limited. It studies some things, not everything.<< what parts of reality is not accessible to science? And what reasons do we have that these parts of reality are accessible to some other discipline?

    Nature of God / Morality
    My argument about squares and circles is not about God per se – it's about the concept of perfection and an illustration of why it's not possible to have something that is simply "perfect" – it's got to be more specific about what is perfect – and in the case of morality or simply "best", that simply isn't something that some entity could be perfect and not face contradictions. The argument is not something so facile as the idea of a rock so heavy that God himself couldn't life – that argument carries no weight with me. There are a great many theists who in fact buy into the God as perfection thesis, some of whom have accepted the ontological argument as a genuine logical proof of God that I think the argument to the contrary establishing that such a God does not in fact exist worth presenting. Can I take it that you reject the God as perfect being hypothesis?

    As an aside, CatholicScientist presents the position that there is some unique version of ethics that is real and true – a view that I reject. In my perspective, ethics are a useful tool – and just like we wouldn't say that a particular shovel is the "best" shovel, we cannot say that any given set of ethics is the "best" ethics and the Platonic view that our day to day ethics is a mere shadow of the real ethics is simply out and out false. Normative ethics are much like a shovel, some shovels are better than others given specific purposes, some are better for other purposes and some shovels (and some value systems) are just generally crappy. We don't need God to come up with a successful ethics – that's not a job we need God for nor a task for which in our experience our theory of God has been particularly helpful – if you look to the countries that are most secular – the Scandinavian countres for example – we see a commonly held world view which does not include a deity which has resulted in a very happy successful population with very progressive values. And contrast that with evangelical preachers in the US – who have contended, for example, that Haiti was struck by a hurricane by reason that they have fallen away from God and so deserved the fate they had been handed. I'm assuming that you are an atheist with respect to this very vengeful version of a Christian deity.

    I'm going to disagree with the claim that "God is beyond being" – I'm not convinced that carries any meaning using normal English. The best way that I would interpret that phrase is to read it as a) God doesn't exist or b) God is fundamentally different than all other stuff that exists but nevertheless does exist. Being is not an entity – and using the term "being" in that fashion leads to confusion and category errors. Rather things exist. I sometimes like to use the term "stuff exists" with the idea that stuff is to be understood as broader than, say "objects'. To my mind we can talk about the universe – which in my terminology represents our local or pocket universe in which it would be theoretically possible to reach any spatial point given, say, the ability to achieve infinite velocities. That universe may be infinite or it may be finite in which case it still likely has no boundaries but rather space is curved such that if were to look all the way across, you'd end up looking at the back of your head. The idea that something could be o/s of space and time or the idea that space and time are not fundamental are familiar ideas in physics. And so I see no particular difficulty in adopting the terminology that Universe+ is to be distinguished from the term Universe in that it expressly includes God. One traditional definition of Universe is the set of everything that exists – in which case if God is not an element in that set, by definition, he doesn't exist.

    If we consider Kant's distinction between phenomena and noumena, I think it's readily apparent that even rocks and stones are arguably beyond our understanding qua noumena – but that doesn't stop us from having good & workable models that we strongly suspect correspond in a meaningful way to the underlying reality.

    The idea that we can distinguish between God's essence and energy is a very Aristotelian concept – in current thinking – an object is its properties and there is no separate essence distinct from those properties. I fail to see any particular utility in departing from this linguistic convention and am of the view that talking about the essence of things apart from their properties represents a view of reality that can only end up misleading us.

    Revelation

    I'm going to forgo a review of the scholarly evidence supporting or critiquing the idea of an historical Jesus – and simply refer you to Richard Carrier's book on the subject. My position is that there is a consensus of scholars that support the idea of an historical Jesus (but not a miracle worker) but that there is reasonable concern for bias by reason of a) the disproportionate number of scholars who are predisposed to choose a religious resolution and b) texts, artifacts, funding and education are largely sourced from religious institutions. It doesn't mean that there isn't good work being done – but that the conclusions have to be reviewed with additional care. Richard Carrier has spoken frequently on youtube and appears to be expert and intelligent and claims to be shocked at the lack of scholarship and merit in the arguments supporting a historical Jesus. There's also the work of some decades ago of John M. Allegro which i think probably has been legitimately discredited but I still have a worry that the church's immediate response was to destroy his career rather than to provide a scholarly refutation of his arguments. In either event, establishing that revelation is a vehicle to knowledge will be a difficult road to hoe. Humans have revelations all the time – I may wake up tomorrow having the revelation that i am the 2nd coming of Jesus – and if I do so, I suppose we're going to have to spend time and energy trying to ascertain the truth of my revelation.

  79. “A universe containing a clock that pops into existence, the clock ticks for 10 minutes and then the universe ceases to exist is perfectly intelligible and logically consistent. There are no axioms of formal logic that conflict with this universe. We don’t have to assume that there is a cause or explanation for this unprincipled behaviour by reason that neither our universe or universes in general do not owe us an explanation. If God exists (in the absence of some proof that he/she/it/? is a necessary being, than either God is the result of some previous cause or God’s existence is simply a brute fact. Perhaps in lieu of a universe containing a clock that pops into existence, there is another clock universe creating universe that is the brute fact (or necessary entity)”

    I think we may be in a fix here. I am arguing that something like, say, a chair or a clock could not simply pop in and out of existence in any intelligible or logical way. Such things – from every last possible phenomenological experience we have with nature – could such things simply pop into existence without any efficient cause, or at a minimum without any relation to anything else. A clock has a clock maker, the clock maker has parents, those parents have parents, all the way down until we either regress for infinity or we come to a “necessary”. Your claim of a world where a clock appears for 10 mins and disappears is not intelligible, but you’ve made a catch-all just in case someone is to question this magical appearance and disappearance: “the universe does not owe us an explanation.” So on the one hand you claim it is intelligible yet on the other admit that the universe is mysterious and does not owe it to us to make such a possible world intelligible.

    But this conundrum was introduced when the argument of necessity was packaged in the logical format of “is it true in all possible worlds” thinking, though the argument does not require this regardless of what some Stanford dictionary of philosophy claims. Just look at how the term is used by Plato, Heraclitus, Hegel, Kant, Schopenhauer, etc. you get the point. There is no one way the term is used. Yet the way I have used it fits perfectly in any possible world using the same definitions and situations. Socrates is indeed mortal since he is a man regardless of what possible world we are considering. This is because a change in worlds does not change the language and the logic. Same with the argument of necessity as I’m using it. In any possible world where everything in existence is contingent there must be something necessary by which all things are contingent (otherwise we fall into infinite regress or come to a logical absurdity like “absolute contingency”). If the clock magically appears it surely has something as its cause. I know you have made a case against this but I think it continues to fail because you have not provided evidence that any event can be its own cause. The idea is simply illogical, no matter what world we are imagining. You’ve used a lot of quantum theory to make the case but it seems the case is anything but demonstrated. The quantum world does not follow Newtonian physics for sure, but that doesn’t mean that what we find in the quantum field is wholly lawless or without cause and effect. Even those quantum phenomenon that seem to be causeless did not happen without relation to something else. I’m no scholar in the field so I’ll ask: are there quantum scientists that claim quantum events cause themselves? That they are their own ground and being, that they are self-sufficient, self-creating, self-sustaining, uncaused effect? If so I haven’t heard. And if so then truly the universe is not a closed-causal system which is something theists are all for, no doubt.

    The argument I am speaking of with all the “necessity” talk is Aquinas influenced for sure, but perhaps originally spelled out by Avicenna. He began with the assumption that whatever begins to exist must have a cause. These are “possible” beings, possible because they must be caused by a prior being, which must also have a cause, but there cannot be an infinite series of such causes. There must be a first cause by which all possible beings are possible – this being is necessary, having its existence in itself and not from a cause. Simple, I know. The logic is over 1000 years old and is still as strong then as it is now in my estimation. As an Orthodox Christian I would not define God as “First Cause” but the logic works to get the listener on the same page as far as why one would call God “necessary” and not contingent. God is not a being among all other beings in existence. In fact, I like how Kierkegaard and others have put it: God never came into existence (i.e., time) and therefore does not “exist” (as material exists) because He always was, eternally. Is this an argument from ignorance? Aboslutely. Isn’t any argument about origins from ignorance? Of course, so we both argue from the same limited brain handicap, but that doesn’t mean we forgo logical deductions based on the evidence. The evidence says the universe began to exist. What caused it? “The universe owes us no explanation.” I don’t care, what caused it? Do we have a right to know. Nope. Our task in life is not to know nature to its last material detail, but, as Socrates says, to know ourselves (which would be a lovely segway back to the OP but what we’re onto now is fun so screw it).

    That’s all I have time for. Forgive me for not tying it up neatly and please interpret anything that may come off short and snarky as my mind in race mode to get to the next thing on the daily agenda. I will do my best to reply to the rest soon, though I’m sure you will respond to this and potentially doom the rest to silence.

  80. Hi Guys,
    I would like to get back to the original argument. Thank you Catholicscientist for the link, I watched the video and a few others of Fr Spitzer.

    Whilst researching primate behaviour, I came across the” ALTRUISTIC” BEHAVIOR IN RHESUS MONKEYS experiment by Masserman (1964).
    In this experiment monkeys were taught to push a lever to receive food. Once they learned to do this another monkey was put in a cage next door and received an electric shock every time the lever was pushed. Monkeys would rather starve than shock their fellow primates. Monkeys that had been victims of these shocks were more likely to starve rather than shock their mates. One monkey starved for nearly two weeks rather than shock his fellow primate.
    What we have here is the “golden rule” at work. Do unto others as you would have them do unto you, or love your neighbour as you love yourself!

    Click to access masserman.pdf

    Compare this experiment with the Milgram experiment. Milgram (1963) wanted to investigate whether Germans were particularly obedient to authority figures as this was a common explanation for the Nazi killings in World War II. Here human apes were instructed by an authority figure to administer an electric shock every time a “learner” makes a mistake, increasing the level of shock each time. Unbeknown to participants real electric shocks were not administered.
    65% (two-thirds) of participants continued to the highest level of 450 volts. All the participants continued to 300 volts. Not one participant ever asked to see if the shock victim was ok or reported this torture to any authority.

    http://www.simplypsychology.org/milgram.html

    According to theists these monkeys are just doing what monkeys do and these human apes have a supernatural dimension to them. There seems to me, to be something instinctively wrong here.

    Theists need to tell us how the origins of a belief whether supernatural or just causal,determine whether those beliefs are rational, warranted, good or bad, and theists also need to tell us where materialism ends and supernatural reasoning begins, how does this supernatural mind interact with the free will theists are so adamant exists?. Without an explanation, this seems to be just a religious claim.

    It would seem that what theists are saying is; If you go through life doing what you can to leave your little world better than you found it, simply because it feels right or brings you some satisfaction, you are living some sort of causal purposeless, undignified life – But if you do things because some God dimension is the originator for your reasons for doing so…… now you have, purpose and dignity. Again, there seems to be something not right here.

    Furthermore theists claim that if evolutionary naturalism is true, then our minds are equipped for survival, but cannot be relied on for truth, especially on theoretical matters. As Reppert implies – If evolutionary naturalism is true, no-one can have a good reason to accept evolutionary naturalism itself.

    The problem with this whole argument, and the explanations for and about God given by Fr Spitzer in Catholicscientists link, is that they are based on the false notion that Logic and reason alone can find “truth”.
    Our theories, including that of evolution, have never been about truth. Our theories have always been about reasonable and rational explanations,(IE without contradictions)to best explain that which we observe.

    The problem of even discussing a claimed non‑material entity like a “mind” is that it could possess no possible attributes enabling it to be compared to anything in the material world. It has no shape or form etc. etc., or any other attribute that can be compared with any physical entity. In other words it can only be made of indescribable attributes. It could be claimed that it must therefore be somehow composed of purely abstract attributes. But what could these even be? Such claims of non – material entities are beyond rational enquiry and should perhaps be left where they belong – In the big bin of religious claims.

    Logic and reason alone cannot determine the truth of an argument. Logic cannot offer any alleged certainty. A logical argument can only infer – and that implies derivation. Logic can only derive an ASSUMED conclusion from its premises. It takes a human observer to empirically prove a proposition by using his sensory system to compare objects referenced in the proposition (i.e. referents).

    Truth doesn’t embody any systems of Logic. Truth is necessarily EMPIRICAL, whereas logic stems from Formal Systems of Logic which are necessarily CONCEPTUAL.

    It is entirely possible for an argument to be logically valid, and yet not true. Should we even be using logic and reason to examine the existence of things that are important?

  81. Sean,
    I am afraid you remain trapped in your physicalist epistemology. Maybe you should ask yourself what you are actually looking for here.
    In case you are actually seeking, let me suggest something radically different. Go to a church near you and observe an Eucharistic adoration. I bet you can see and feel the peace and joy of the people there who are freely doing what they were created to do, i.e. worship. Also, check out youtube videos of people who talk about their recent conversion experience. Don’t bother with what the say; just watch their eyes. There it is again, genuine, hard to contain joy. You may want to check out Leah Libresco who converted from atheism, for example. Listen to and encounter people who are on fire with love of God. Compare that to what you see in the eyes of many a youtube atheist. That will perhaps open your eyes to the truth much faster than any logic, reason, or empiricism.
    In any case, I wish you the best. I will retire from this strange impersonal medium of blog post comments, I guess. It lacks the personal encounter. We are much more than what can be captured by a series of keystrokes.

  82. Hi CatholicScientist,
    What I am looking for here is to resolve the argument from reason in my mind.

    I understand that you find blog post comments impersonal, but we are here to challenge and exchange reasons,beliefs and ideas which is important because it is with reasons that we weave the essential fabric of civilized existence.

    If you have a better reason than I -: I will yield.

    Are we not supposed to accept what can be shown to be reasonable and give up what is shown to be unreasonable? Or doesn’t your faith look kindly on that?

    • Sean,
      the whole point of the argument from reason is that if naturalism is true you have no good justification to use reason to assess the argument from reason, or any other philosophical argument for that matter. I may have misunderstood you when you wrote: ‘the false notion that Logic and reason alone can find “truth”’. I thought you had conceded that point and tried to get around Reppert’s argument by arguing against rationalism and for empiricism (or physicalism). You claimed that ‘truth is necessarily empiricial’. I think this is patently false. If you think about it, you cannot assess the truth of Reppert’s claim by empirical means. So if you believe what you wrote, you are in a dead end, in my opinion. In your last post you argue for reason again: ‘Are we not supposed to accept what can be shown to be reasonable?’ You have me confused by now. You are not trusting that Spitzer’s reason leads to truth but when faced with the supernatural you resort to reason again?

      If you want to advance beyond this conundrum, I recommend you jettison naturalism so you can fully trust reason again. Also, I recommend broadening your epistemology beyond rationalism and empiricism. Once naturalism is in the trash bin and one accepts that humans have a supernatural dimension, would it not be reasonable (!) to expect that we can use our supernatural faculties to acquire knowledge as well? Dualism neither negates empirical ways of learning about reality (otherwise I’d have to give up my job), nor diminishes reason in any way (quite the contrary), but postulates that we have more tools at our disposal to find the truth.

      If we have wings, what are they good for if we don’t fly? Don’t be a dodo 🙂

  83. I can well understand the idea that this debate has gone on too long or that there are better things to do with your time or that one has become worn out – so from that perspective I can understand why you might wish to withdraw, CatholicScientist. I don’t understand the reasons that you have actually given.

    Socrates commented a long time ago that the unexamined life isn’t worth living. One would think that i your stated profession, the critical examination of ideas & the investigation as to whether or not such ideas are likely to be true would be your bread and butter. Another scientist who came to you and suggested that he was convinced of a given hypothesis by reason that it made him feel good would or at least should be given short shrift. There’s no general law of nature that says that if some idea makes you happy, it’s therefore more likely to be true.

    I hadn’t heard of an Eucharistic adoration before, so I looked it up – and seemingly it would involve a long period of mediation – a method pretty much guaranteed of giving striking altered states of consciousness whether or not it is associated with religious trappings. The joy you describe in your argument would seem to me more consistent with a manic experience and more disconcerting than something I would ever wish for myself – but perhaps that’s unfair. When I look at youtube videos of Christians going amok in some form of professed (or real) ecstasy, I find i more disheartening and scary than something positive.

    There are certain people who are very susceptible to peer pressure – in an interesting experiment, people were shown 3 lines of differing lengths and asked to pick out the longest one. In point of fact 1 of the 3 lines was reasonably obviously the longest but in point of fact there was only 1 subject of the experiment – all the others were stooges of the experimenter – and they all chose a line that was not the longest line. In case after case, the subjects would agree with the other people and about 30% would continue to argue that the line that they had chosen was the longest line even after it was explained to them how they had been tricked.

    I think personalities are very much on display here – you seemingly are more likely to engage in personal attacks than others in this group and more likely to attempt to change the topic – or at least that’s my impression. If it wasn’t for the fact that I too am kind of worn out and would like a break, I would have expressed the preference that you continue to stay and argue your position.

  84. do I take it you’re taking the mature position of ignoring me? well, so be it, if that’s the case or I suppose there’s a possibility you’re being considerate in that I am actually trying to withdraw gracefully from the discussion. I think a response to your last post is nevertheless warranted:

    1) yes – the point of the argument from reason is exactly as you describe – and as discussed, it fails. And it fails for a few reasons. We are evolved to be intelligent, somewhat rational beings. Unintelligent, irrational beings would tend to die and so there is an evolutionary pressure in support of intelligence. The test for rationality is fairly straightforwards – deductive arguments based upon fairly simple rules and inductive arguments based upon the observation of patterns. Plus abduction wherein we attempt to reconcile our ideas into a simple logically consistent manner – that’s one of the methods that allows us to judge that we are in fact capable of recognizing intelligence. And the result being that these techniques work and allow us to make reasonably accurate predictions about the future – and that’s our proof. We are not tremendously rational creatures and I refer you to my earlier quote from Neil deGrasse Tyso: ” I am convinced that the act of thinking logically cannot possibly be natural to the human mind. If it were, then mathematics would be everybody’s easiest course at school and our species would not have taken several millennia to figure out the scientific method.” so based upon this quote – our supernatural aspect hasn’t been all that successful over historical times. And lastly a supernatural element is no guarantee that rationality will work – and so an extreme skeptic would in a logically consistent manner could reject the idea that anyone is rational whether we have a supernatural aspect to our existence or not.

    2) and Sean’s point that logic and reason alone can’t find the truth would simply imply that one needs observations and experience in addition to logic and reason – nothing untoward about that

    you’ve done nothing to establish that humans have a supernatural dimension or even to explain what you mean by such a claim or how that would be a fruitful hypothesis. You’ve haven’t put naturalism into the dustbin – that just isn’t true. And even if one had a supernatural dimension, that would not justify a claim that it would provide facilities to acquire knowledge – that require additional proof.

    No one is claiming that dualism negates empirical ways of learning about reality – but rather that empirical ways of learning about reality would appear to largely discount dualism. That may change in the future but I’m dubious – but there’s considerable difficulty in finding an appropriate link between spirits and physical objects such as human bodies that makes any sense in light of observations.

    “Don’t be a dodo” – can you stop engaging in personal attacks – you’re the only one doing so. It’s not helpful to your argument.

    • David,
      Remember you wrote: ‘I applaud your decision to bow out. It will be interesting to see if you meant it.’ ?
      I meant it. It clearly was the right decision.
      Good day!

  85. you’ve posted 8x since then – but who’s counting. And the only reason that I objected to your posts was your penchant for personal attacks and changing the topic – and I suppose various strawman arguments. In any event, good luck to you – I wish our discussions had gone more smoothly.

    • None of these posts were addressed to you and I don’t recall any personal attacks.
      You do know what a dodo is right? At some point there was a bird in the dove family that flew to an island where it had no predatory pressure. No need to fly anymore. So it didn’t. The wings became weak and finally useless. All it needed for survival was its legs (reason) and it forgot it had wings (supernatural faculties) and could fly. So don’t be a dodo. Use it or loose it is my point. And, no, this is not a personal attack. It’s a feeble attempt at a parable. BTW the dodo is extinct. Things changed and legs all of a sudden weren’t enough.

  86. Hi CatholicScientist,

    The whole point I am trying to make is that the so called argument from reason appears to be misrepresented to me.

    Reasoning is just the action of thinking about something in a sensible way,(whatever that may be!) Our close cousins the apes have demonstrated that they too are capable of solving problems, and forming rational concepts,(IE relations between objects), to be able to solve those problems.
    Humans are far better at solving problems than the apes as we have better developed brains. I referred previously to the physical explanation for that evidence here.

    http://news.berkeley.edu/2014/12/03/reasoning-skills/

    We humans believe things because we have “reasons” to do so. Reason = a cause, explanation, or justification for an action or event, just as the apes do.
    What would make this a “good” reason? Because after all the argument from reason does not suggest that naturalism entails not thinking at all or not having any justification, or cause to believe something. The argument states that we would not have any “good reasons” for believing the things we do.
    If a reason = a cause, justification or explanation for something; then a good reason is just a desired cause, justification or explanation that is seen as beneficial in some way. Good = .to be desired or approved of; to be seen as beneficial.

    The Rhesus monkey experiment showed that monkeys that had been victims of being shocked were less likely to shock other monkeys, even if it meant starving.Presumably there is a reason for this change of behavior in the monkeys that had been shocked.And yet theists would have us believe that there cannot be a good reason for this change in the behavior from the shocked monkeys point of view because they are just monkeys. You would need to explain why they could not have reasons for this change in behavior and explain what it is then that caused the change in the previously shocked vs non shocked monkeys.

    Hence the questions I asked in the post which I will repeat here to be answered.””Theists need to tell us how the origins of a belief whether supernatural or just causal,determine whether those beliefs are rational, warranted, good or bad, and theists also need to tell us where materialism ends and supernatural reasoning begins, how does this supernatural mind interact with the free will theists are so adamant exists?. Without an explanation, this seems to be just a religious claim””.

    Let us test this notion that logic can find truth. Here are examples of logically consistent, sound and logically valid deductive arguments given to me some time ago by somebody that gave me a good schoolin on truth vs logic…..

    Premise 1: The human body is made up of cells.

    Premise 2: Cells are invisible to the naked eye.

    Conclusion: Therefore, the human body is invisible to the naked eye.

    This is a logically valid conclusion stemming from the axioms and the assumed premises!
    If you are one of those believers who think that logic has anything to do with “truth” or with reality, then you have no choice but to accept this conclusion as proven truth.
    And yet – my sensory system tells me this is BS – I see people……
    AAHHHH composition fallacy they all scream. The question becomes; how do you know that there is a composition fallacy here? Not by any laws of logic, but rather as I stated, “It takes a human observer to empirically prove a proposition by using his sensory system to compare objects referenced in the proposition (i.e. referents).”

    Lets test this and deliberately commit another “composition fallacy”.

    P1: Air is comprised of atoms.

    P2: Atoms are invisible to the human eye.

    C: Therefore, air is invisible to the human eye!

    And here my sensory system tells me that this would appear to indeed be the case.
    We cannot find truth by asserting some or other fallacy. In fact we can take any axiomatic rules or “context” and get anything to say anything according to the rules of logic.

    1=number

    2=number

    Thus, 1=2

    Huh say what??????

    Similarly, A=5, B=5, Thus, A=B

    You can’t say that the letter A is not the same as the letter B. It’s all about the context (i.e. axiomatic rules) of the tautological system in question.I have defined and used 1 and 2 as numbers and not as the values they might represent to you.

    So when somebody like Fr Spitzer states that we come to the conclusions we do about God by extending the laws of logic – and we have no observations to take into account with the argument – I am skeptical of the conclusions we arrive at. Just as I am about this argument from reason.

    We humans do not just reason logically. We identify and recognize concepts; IE relations between objects; based on what we observe and experience.We “test” our beliefs everyday in the real world.You too resort to observation and human experience when you suggest that I should observe people worshiping and experiencing God for “proof”.

    Definitions.

    Believe = an acceptance that a statement is true or that something exists.
    Good = to be desired or approved of
    A Reason = a cause, explanation, or justification for an action or event.
    To reason = the capacity for consciously making sense of things, applying logic, establishing and verifying facts, and changing or justifying practices and beliefs based on new or existing information.

    Why can a naturalist not do any of the above. Where is the super natural required for any of the above?

    The rhesus monkeys believe(accept that things exist)
    The rhesus monkeys can reason (by making sense of the shock/food system,and changing their practices based on the new information)
    The rhesus monkeys have a reason(by identifying the cause of the electrical shocks + starving)
    The rhesus monkeys are good to/for their fellows (despite causing themselves discomfort by starving.)

    The argument from reason appears to be no argument at all.

    • Sean, I think I partially understand what you are saying here.
      I’m afraid your logical examples are flawed; they are mostly non-sequiturs, but that is beside the point. No need to argue about these. I agree with you that philosophers sometimes have the tendency to overstate what they can learn by deductive reasoning. William Lane Craig does this with the Kalaam argument and Fr. Spitzer with the Fine Tuning argument.
      I look at these arguments not as proofs. That is in fact an area where I have some problems with a teaching of the Catholic Church (i.e. God’s existence can be known by reason alone – the Catholic Church has always had a quite strong tendency in favor of philosophic reasoning. Maybe that’s why I like to hang out on Orthodox blogs and web sites :-)). When I consider these arguments, I look for the most likely explanation. I ask myself, is it more likely that there is an intelligent mind behind the extreme complexity of the universe – and it’s very existence – or is it more likely that it is mindless. Is it more likely that something begins to exist without cause, and has to be accepted as a brute fact, or is more likely that there was a first cause. Is there likely fine tuning for intelligent life in the universe and, if so, is it more likely a consequence of a multiverse or more likely by design. What people find more likely depends on their worldview.

      From a dualist point of view all these questions have a rather convincing and reasonable explanation in the existence of God. Materialism (naturalism) on the other hand makes little senses to me.

      I believe that the ‘mind’ is primary. It is what got everything started and defined it. You often hear the phrase that God ‘thought’ or ‘spoke’ the world into existence. You find that in Genesis even.
      Most Christians believe that humans are created ‘in the image of God’, which means that our minds are in some way similar to God’s mind albeit in an imperfect way. Nevertheless, this is the supernatural part of us that gives us agency, true free will and the capacity to discern the truth (including ethical truths) in harmony with our natural faculties (the material brain). In this worldview there is an ontological difference between animals and humans, as animals lack the supernatural capacities (as far as we know) that allow us to become co-creators (have dominion) and give us moral responsibility and a capacity to either freely accept or freely reject God. The trustworthiness of our reasoning is safeguarded by our likeness, however flawed it may be, to the perfect mind of God which is the source of all existence (or existence itself as some say).

      This (!) concept of mind does not exist in materialism at all. I cannot see how there can be agency or true free will in a world that is solely determined by natural law or chance. In a naturalistic worldview, I cannot see how we have any control or responsibility over what we do or think, that includes all reasoning. It is automatic or random. Consequently, there is really no essential difference between animals and humans, only in developmental state and capacity of the brain and associated programming. In my opinion the logical consequence of naturalism is that there cannot be a mind in the sense that I perceive it. There is only the illusion of a mind, including an illusion of reasoning.

      This is what I think C.S Lewis was and now Victor Reppert is saying (but you can ask the latter yourself – he has a blog where he answers questions). In materialism reasoning is an automatic brain function that was selected for by evolution and while it must be good enough to keep us from stepping of a cliff, it does not necessarily follow that it suffices to solve difficult ontological questions. Back to my original take on these arguments: I find it very unlikely that it would suffice. (That is where people disagree. In my experience that disagreement is often motivated by denial or a futile attempt to preserve a view of oneself that is incompatible with ones worldview.)

      So from the viewpoint of a theistic dualist or idealist, who assumes a divine spark in us that elevates us far beyond the machine-like functioning of animals, the diminished concept of reason under materialism is a drastic step down. It’s like having no wings anymore.

  87. Sean,
    regarding your other questions which I am tempted to summarize with ‘How does this all work?’ I would like to answer: “I observe that it does.” A supernatural event can also be called a miracle. These tend to evade, by definition, explanations that would satisfy a materialist.
    To go beyond that horizon you’d need to get comfortable with contemplation (read a book by Thomas Merton, or Eric might be able to recommend something else). If you want to learn how your mind works you need to approach the mind that started it all, the mind of God. No electron microscope is going to help you there, in my opinion. Don’t worry, the folks I know who tried this did not loose their mind, they found it (and some joy on the way).
    Peace

  88. “You do know what a dodo is right?” yes, I do – Webster’s dictionary for example defines it as follows: “a type of bird that lived in the past and that was large, heavy, and unable to fly or alternatively “a stupid or silly person” – there are many flightless birsds but the dodo was notorious for lacking intelligence and so it’s a widely recognized insult to describe someone in that fashion. And your description of the eyes of atheists that you found in youtube was insulting. And there have been other insulting comments that you’ve made throughout this thread that I have previously drawn your attention to.

    There was an interesting experiment on the subject of free will in which the experimenters demonstrated that they were significantly more capable of predicting a subject’s future behaviour than the subject was – a point that I find interesting given your widely expected return following another proclamation that you were packing up your bags and leaving.

    On your most recent substantive points – a) in the thread in this blog regarding the compatiblity of religion and science – I’ve responded to your post regarding the fine tuning argument. The extreme complexity of the universe is misleading – the actual origin of the universe involved very simple processes which scientists can duplicate in particle colliders back to about 10 to the – 12th seconds. Scientists know how to build a universe – we’re just lacking the technology – we’d need a small chunk of false vacuum. Subsequent developments creating the apparent complexity in the universe are pretty much broadly explicable in terms of material science and natural laws and various feedback mechanisms.

    I’d begun an addendum to my last post to Eric regarding Nagel’s book and it appears relevant here:

    Nagel seems to be of the view that it is inherently implausible that physical processes would lead to life – and yet from the scientific community, pretty much the exact opposite is happening. see https://www.newscientist.com/article/mg23130870-200-life-evolves-so-easily-that-it-started-not-once-but-many-times/

    Up to about 5 or 10 years ago, there was only about 10 or 20 scientists in the whole world exploring the origin of life – now it’s very much a trendy area of science and I see significant new discoveries almost every week – or at least on a very regular basis. Nagel is almost hinting at a life force as a possible explanation for living organisms, a theory that disappeared on its merits several centuries ago.

    I think you’re running a god of the gaps argument & an argument from personal incredulity – as opposed to putting forwards some sort of viable, testable meaningful explanation of, well anything. If miracles are happening o/s of natural law – then where are they? Why don’t they show up when scientists scrutinize the purported evidence?

    oh – and this is a public forum with no expectation of privacy on a topic of public interest, not a private messaging between 2 debaters – there is absolutely no reason that I should not be free to respond to what I perceive as errors or mischaracterizations on behalf of what I shall call, team atheist. I’m of the view that my interjections should not take us off topic and interrupt the flow of the argument – but addressing substantive points at issue does not do that.

  89. Hi CatholicScientist,
    If you think that my examples of “logical” arguments are flawed as they are mostly non-sequiturs, then please google examples of non-sequiturs and give me an example. I can see no examples given that resemble the arguments I presented.

    I find it strange that you refer me to Fr Spitzers video for an explanation on the existence of God and now state, “I look at these arguments not as proofs. That is in fact an area where I have some problems with a teaching of the Catholic Church (i.e. God’s existence can be known by reason alone –….”.

    CatholicScientist, I want both you and me to discuss the points we make as true Christians. The “golden rule”, (Christs teaching on treating others as you wish to be treated), has implications for how we ought to argue and reason.
    If I don’t like it when others are obstinately unpersuadable, I think that I should avoid being obstinately unpersuadable too. I shouldn’t deploy reasons in ways that I wouldn’t want others deploying them. If it’s inadmissible for others to invoke their articles of faith in support of conclusions I disagree with, it’s equally inadmissible for me to invoke my articles of faith for conclusions they may disagree with. To “cut to the chase here”, I think that there is something profoundly “unChristian”—and morally wrong—when people feel that they can make claims based on articles of faith, beliefs, and any “feelings” they might have, and place the burden of rational accountability on those who do not agree with them and insist that they make their case with hard facts and rational explanations. Please – lets keep our expectations of how we think others should react in check.

    I wish to question some of the claims you make, NOT those stated as beliefs, or articles of faith.
    You write, “Is it more likely that something begins to exist without cause……” Eric also refers to the universe beginning to exist in his discussions with David. I have reasoned out why existence does not begin in a previous post. Let us test this, and hopefully put this crazy idea to rest.

    Let’s create a cheese omelette, sprinkled with chives and served at exactly 45 deg C.
    What we have is a bench, bowl, an egg, pinch of salt, chopped chives, grated cheese, and knob of butter, a pan, fork, spatula, thermometer, gas burner and serving plate. This is what exists.

    Step1. You break the egg into the bowl, beat it with the fork and add the salt. Place the butter in the pan; light the burner and start to melt the butter. -What exists now is an emulsion in the bowl with added salt and a pan on a burner containing melting butter.

    Step 2. Once the butter is melted, pour the beaten egg into the pan, and using the spatula gently move the egg mixture to the centre of the pan. -What exists now is an egg mixture in the pan.

    Step 3. Once “set”; turn over the whole mixture to cook the other side, and place the grated cheese in one half of the ‘round’. -What exists now is a half cooked mixture with grated cheese on one half of the” round”

    Step 4. As soon as the underside is set, fold one half of the ‘round’ over the other and place on the serving plate. Place the end of the thermometer in the centre of the cheese filling. As soon as the drops down to 46 deg C sprinkle the chives over the top. -What exists now is a cooling meal waiting to reach temp before garnish!

    Step 5. As soon as the thermometer reads 45 deg C present the 45 deg C Cheese omelette with chives to the panel at show and tell on Eric Hyde’s blog 2016. -What exists now is what we set out to create. A cheese omelette sprinkled with chives and served at exactly 45 deg C.

    So CatholicScientist tell us when the cheese omelette, sprinkled with chives and served at exactly 45 deg C begins to exist, at what step? I mean it must have begun to exist at some point because it certainly did not exist eternally in the past – besides, you just made it.

    You cannot. You cannot because existence does not begin. To exist is to be; and to be, means to be something in particular – not a “was” or a “going to be”. At each of the stages I have stated what exists – something in particular, not a ‘beginning’ or a ‘used to be’.

    Everything is formed from pre-existing matter. Existence does not begin – it is what something is. You may well state that the Big bang theory claims that an expansion begins, I am sure all would agree with you. But you cannot claim that the universe began to exist, because it did not; something had to have expanded, and that is what exists.

    As far as your claim that the mind is a supernatural part of us goes. This is full of problems.
    For example:
    It is a law of nature that matter and energy can neither be created nor destroyed. But that would mean that some new energy must come into existence when the mind causes the brain to be in some or other state. In other words a suspension of natural law, (a miracle) must take place on an almost continuous basis.

    I understand that you are tempted to summarize with ‘How does this all work? But it will not do as there are many aspects of this argument that should bother the theist. Try and answer the questions I asked.
    I find it a bit off sides that theists can come with a so called argument from reason with their claims full of holes and then state that materialists should be worried, or uncomfortable with their thinking.

    As previously stated, It would seem that what theists are saying is; If you go through life doing what you can to leave your little world better than you found it, simply because it feels right or brings you some satisfaction, you are living some sort of causal purposeless, undignified life – But if you do things because you believe some God dimension is the originator for your reasons for doing so…… now you have, purpose and dignity.

    Theists admit they have problems with their dualism claims but are quick to state that they still think they are better off, or as you state, “it makes more sense…” It just does not cut ice. A bit like the theist laughing at the materialist for trying to hit a home run without a bat or a ball, and then claiming a “win” because although they too cannot hit a home run, at least they have a bat!

    • Sean, I’ll give you a few thoughts:
      I pointed you to Fr. Spitzer, because I think he is a smart and well educated guy worth listening to. In the same category are Bishop Robert Barron and David Bentley Hart. They can explain things better than I can.

      I’m not placing a burden on anyone. I’m trying to explain to you what I think and believe.

      I don’t think it is very likely that the universe did not begin. According to our current understanding of physics, space-time was created with the big bang. Time itself began with the big bang. There was no ‘before’. Now, this may turn out to be wrong but I have to run with mainstream physics here.

      The part that interests me is where you see the problems with a dualist view of the mind. Conservation of mass-energy is assumed for closed systems. I am not convinced that the mind is a closed system. The universe – if it had a beginning – is not a closed system either.

      ‘In other words a suspension of natural law, (a miracle) must take place on an almost continuous basis.’ Yes, that is what I think happens. Once you try to explain reality with both natural and supernatural elements, interaction between the two is expected. Thus, miracles are expected. That may sound incredible from the perspective of a naturalist but it is not from the perspective of a dualist. Miracles are not prohibited in this worldview. They are normal elements of it. However, I would define ‘miracle’ a littler broader as ‘an effect with a non-physcial cause’ – it may not be necessary to suspend natural laws for that to happen. An area were there seems to be a possible interface between the two realms is indeterminacy and the role of an observer (in this case a supernatural one) to collapse the wave function of a system (the brain). This would lead us into the Copenhagen interpretation of quantum mechanics. If that is your cup of tea I’d love to discuss it further. But, assuming the primacy of mind over matter, even miracles of the more obvious kind should be possible.

      You say my claims are full of holes. What are these holes?

      The part about human dignity: From a Christian point of view the human being derives its dignity from the fact that we are created in the image of God. That is, our dignity is essential. It is part of our nature as spirit being. It is independent of age, health, race, mental state, our actions, etc. It is inalienable in the true sense of the word. Hence an atheist has the same human dignity as a saint. All human life without exception and without condition is sacred.
      I don’t understand the concept of human dignity in naturalism and what this dignity rests on. I’ve heard many conflicting stories.

      ‘Theists admit they have problems with their dualism’
      Not sure I understand what you mean. Do you expect a perfect theory of everything? I don’t think that we will ever get to the point of complete understanding during our earthly lives, not in the material and not in the immaterial world.

  90. To be honest, ChristianScientist, I can’t help feeling that your entire post simply ignores posts written previously by me that addressed pretty much all of your “arguments”. I put arguments into quotations in that your post really purports to set out your beliefs w/o regard to much in the way of critical examination.

    It might be better to suggest that OUR universe probably had a beginning – both with respect to space and with respect to time. Having said that, there are innumerable speculations as to why our universe may have begun – and I’ve alluded to a number of them in this thread – none of which seemed to have percolated into your understanding. Take for example eternal inflation that pops off universes, each starting with their own time T= 0 but for eternity. Perhaps it would be helpful if you actually review various actual cosmological models proffered by cosmologists and indicate where miracles are required & how you would substantiate that claim. And I’ll point out that I previously drew your attention to the idea that space has the capability of creating more space w/o regard to the 2nd law of thermodynamics – that doesn’t mean it’s a miracle – it just means that we’ve made a series of observation & it would appear that the expansion of space is simply something space does (perfectly consistent I might add with predictions of the general theory of relativity).

    Your logical deduction – if our universe had a beginning, it must therefore be a closed system seems opaque – please justify this argument. Our sun had a beginning – is it a closed system? Now it’s perfectly likely that the universe as a whole may function differently than its parts – but how do you address, for example an earlier comment of mine (relying upon a link to a notable cosmologist) that the expansion of the universe implies that space is in effect creating space – each new unit of space created has exactly the same energy as the previous unit of space – is that a miracle under your new definition of miracle or is it simply the operation of natural law? How do you address the argument that the cosmic microwave background might well disclose the impact of another universe striking our own – it’s an open speculation but there’s a couple of places on the CMB that would seemingly support that claim.

    On the one hand you suggest that miracles take place almost continuously and involve the suspension of natural law – but fail to address what natural laws are involved, how we know that they are suspended, what you definition of miracle is (and yes, you do define miracle later but in a way which is inconsistent with this part of your argument), what is a natural law, how do we identify natural laws, etc. Now I provided a definition of miracle earlier but you never commented or responded at the time. And, of course, you’ve failed to address why scientists don’t actually observe miracles and alleged miracles, once thoroughly scrutinized would seemingly be susceptible to natural secular explanations again and again.

    On the other hand you suggest that we should broaden the meaning of miracle to include events that are the result of natural law – and so the supernatural and miracles presumably are in fact natural, albeit not physical. Here’s the punch line – you are implicitly assuming that scientists think of the universe as something composed only of physical objects – but that isn’t true. Quantum fields, for example are not physical objects, a light photon is not a physical object and quantum particles aren’t really like little balls bouncing off each other. According to your definition, every time an electron emits a photon, we have a putative miracle as opposed to a theory that the electron emitted a photon because that’s what photons do from time to time. Surely that’s stretching the meaning of miracle beyond any reasonable bound. In your view, is God actively involved in every quantum interaction just to keep the universe on track and rolling? How does that help us in understanding reality? How plausible is it? Not, I would suggest.

    I think you’re failing to understand what science does – it doesn’t provide an ultimate explanation – at the end of the day we have regularities that we can describe using mathematical formulae and we have pictures in our mind that we’ve built up from common experience that form something of a foundation for our general comprehension – we have a sense of what a force is because we get to push things around with our muscles – but our common every day experience is very limited compared to all of what the universe has to offer. And our everyday notions don’t hold up very well. You seem to be taking the view that insofar as our everyday notions don’t hold up, we should simply call that a miracle – as opposed to nature doing whatever it is that nature does. The term ‘miracle” has a lot of baggage tied in with various religious evocations – broadening the definition as you’ve suggested would simply lead to loads of misunderstandings.

    And, by the way – the Copenhagen Interpretation does not require a mind in order to collapse the wave function – a camera or other detector will collapse the wave function with or without a conscious observer. In fact it would appear that given sufficient quantum interactions, the wave function will collapse.

  91. Sean, CatholicScientist is quite correct with respect to your argument about the limits of logic. By “non sequitur” all he means is that your argument does not follow logically and is not formally valid.

    Take your argument “1 = number” and “2 = number”. Therefore 1 = 2. The problem with this argument is that we have a well understood definition of “1” and “2” which implies that they do not mean n are the same thing as “number”. 1, 2, 3 , 4, etc are all numbers (ie. they are elements in the set of all numbers) but that doesn’t mean that we define them as “number” . Russell helpfully defined “1” as the set of all sets having 1 element and defined “2” as the set of all elements having 2 elements – and using the language of set theory thee definitions can be set out w/o being recursive or circular. And so when you run your argument, you are in effect redefining “1” and “2” to mean the same thing as number and not what we commonly understand to be the meaning of “1” and “2”.

    Now we can define letters or symbols for the purpose of an argument or demonstration. Letters such as A, B, C or x, y, z can be placeholders for constants or variables and so if we define A=5 and B=5, then A = B. That’s not problematic. In other contexts, we might define A = 7 – but that would be for a separate argument. Or sometimes we might have a variable run over a certain range of numbers leading for example to formula that can be mapped onto a curve on a chart. At no point will x = 5 and 6 at the same time – that would be a contradiction unless we were running an argument specifically to prove that one of our premises was necessarily false.

    Your argument that Humans are composed of cells is true. Your 2nd premise that cells are invisible to the human eye is false – we can see them in the aggregate, at least some of the time. Clearly individual cells are invisible to the human eye but that says nothing whatsoever about aggregations of cells. There is no such thing as a logically valid argument that commits a fallacy. Given true premises and a logically valid argument, the conclusion will be true. Contrarily, one can have a true conclusion regardless of whether or not the argument is valid or whether or not the premises are true. Alternatively one can criticize the actual logic of your argument – the phrase “is composed of” does not mean “has the same properties as”. And yet you are assuming the latter interpretation in order to justify your argument.

    Examples of logical & valid arguments are: all men are mortal, Socrates is a man, therefore Socrates is a mortal. Or, if some p & q are true (where p and q represent 2 propositions) then p is true – a claim that is hardly earth shattering but is a necessary truth that is the direct result of the meaning of our language. Similarly if p is true and p implies q, then q is true. These are not things you are going to disprove or find a counterexample in any logically possible world

  92. Sean,
    I just stumbled across another Fr. Spitzer video that was posted a few days ago that I believe you may find interesting. It is very pertinent to our discussion. watch?v=x0DMTZud0FI
    He discusses differences between primates and humans and evidence for a supernatural soul.
    The first part deals with Bible exegesis and the historicity of the Genesis account, so if that is not so interesting to you, just skip the first 20 min or so.

  93. Hi David,

    David RE the numbers argument.
    We are talking about tautologies here. All tautologies are pre-defined irrespective of whether you like it or not. If you wish to use that system of tautology to solve a problem, then you have no choice but to follow its axiomatic rules. For example, anyone can invent a system of logic where 783 = 5, 87+6=K whatever, whatever.
    You can’t come along and claim it’s wrong. Tautologies are NEVER wrong or right, false or true, incorrect or correct. There are what they are DEFINED to be…whether you like it or not. I defined the symbol 1 to be a number. I also defined the symbol 2 to be a number. As you rightfully state, “Now we can define letters or symbols for the purpose of an argument or demonstration”, which is exactly what I did.
    Where in any system of logic do we have to use what Bertrand Russel defines or what the “common understanding is? Do we hold a referendum to discern common understandings to see which logic fails and which corresponds with reality?

    You state,”Your argument that Humans are composed of cells is true. Your 2nd premise that cells are invisible to the human eye is false – we can see them in the aggregate, at least some of the time. Clearly individual cells are invisible to the human eye but that says nothing whatsoever about aggregations of cells.”
    Bingo – this was exactly my argument. Logic has nothing to do with reality. We are talking about a composition here. If an object is comprised solely by invisible (to the human eye) objects, then it goes without saying that the object itself is invisible to the human eye. Air atoms are invisible, and so is the air in front of you. Reasoning 101.
    But according to you “Atoms are invisible to the naked eye” must also be false then because individual atoms are invisible to the human eye but that says nothing whatsoever about aggregations of atoms.(as per your cell argument) And folks that is why we can see air!

    My point is that It takes a human observer to empirically prove (i.e. validate) a proposition by using his sensory system to compare objects referenced in the proposition (i.e. referents). Logic alone does not = reality. So how reliable is a so called logical argument when we cannot validate propositions where the referents are non material entities that are unknowable?

    Thanks for the chimp link David.

  94. Hi CatholicScientist,(CS)
    I note that you do not attempt to identify a beginning of existence in the omelette example I presented. You now make an appeal to authority on the “universe” having a beginning.

    People like William Lane Craig make the following claim which seems pretty popular amongst theists and genuinely is the current scientific thinking on this subject. “The 2003 Borde-Guth-Vilenkin paper shows that almost all inflationary models of the universe will reach a boundary in the past – meaning our universe probably doesn’t exist infinitely into the past.” From this guys like Craig arrive at the beginning of the universes existence.
    This is incorrect.
    Victor Stenger asked Mr. Vilenkin the following question.
    Does your theorem prove that the universe must have had a beginning? Vilenkin replied, “No. But it proves that the expansion of the universe must have had a beginning. …….” CatholicScientist, this is what I stated.

    You further claim,” According to our current understanding of physics, space-time was created with the big bang.” Say what??? Space time was created? Really? Space- time is a concept, a notion in a human head. CatholicScientist, we are back to definitions of object; concept and exist.

    From Wikipedia –“ In physics, spacetime is ANY MATHEMATICAL MODEL that combines space and time into a single interwoven continuum….. In cosmology, THE CONCEPT OF SPACETIME combines space and time to a single ABSTRACT UNIVERSE…MATHEMATICALLY it is a manifold whose points CORRESPOND to physical events.” (Capitalization’s are mine for emphasis!) Did you get that? Mathematical model; concept of space-time; mathematically corresponds to physical events.

    Here is a direct quote from Einstein, “Space-time does not claim existence in its own right, but only as a structural quality of the gravitational field”. Here is another quote by Einstein, “Space and time are modes in which we think, not conditions in which we exist” which is a view also expressed in 900 AD by the Arabic physicist Ikhwan al-Sufa, “Space is a form abstracted from matter and exists only in consciousness”.
    I trust that your ‘beginning of existence’ problem and the creation and existence of concepts like space-time will now be laid to rest.

    You state –“ it may not be necessary to suspend natural laws for that to happen. An area were there seems to be a possible interface between the two realms is indeterminacy and the role of an observer (in this case a supernatural one) to collapse the wave function of a system (the brain).”
    CS the wave function collapses upon interaction with the experimental apparatus. No observer is required. Quantum weirdness is confined to the atomic and subatomic domain and does not bubble up into the world of ordinary reality. When large numbers of quanta interrelate, their individual “quantum weirdness’s” cancel each other out. Nothing to do with “consciousness.” Nothing mystical.
    As Bohr himself insisted: “There is no quantum world.” So no I don’t want to go “quantum”.

    I would like to get to the big questions on this argument.

    In the examples of the Rhesus monkeys, starving rather than shocking their mates for food, vs the human apes shocking their mates, (with lethal doses), because a guy in a lab coat tells them to do so.
    How do you explain this?
    Here we have primates able to stop themselves falling off a cliff, and that’s about it according to you; practicing the golden rule and human apes shocking the crap out of their fellow man despite humanity, (and here I will quote you), deriving,” its dignity from the fact that we are created in the image of God. That is, our dignity is essential. It is part of our nature as spirit being……….. It is inalienable in the true sense of the word.” In your link of Fr Spitzer; he speaks of a human soul and why apes and monkeys are incapable of this kind of behavior and yet here we have monkeys living by the golden rule and humans behaving like , well- like monkeys!

    I am sorry CS but this cries out for an explanation.

    You state,” I don’t understand the concept of human dignity in naturalism and what this dignity rests on. I’ve heard many conflicting stories.”

    Here is another’ story’. “Proscriptions against lying, cheating, stealing, and killing find their way into the behavioral codes of widely different cultures, not because they are handed down from God, but because they “pay their way”—they give the members of those cultures far more than they take away. We voluntarily give up certain freedoms (most notably, the freedom to transgress against others) in exchange for the benefits that come from others doing likewise. If we agree to forego mugging each other—and thereafter observe the agreement—we give up the spoils of mugging, but gain something of greater value: freedom from being mugged.”

    At its core, human dignity is about becoming civilized and is a matter of submitting to the force of the better reason. I worry that religious beliefs sometimes prevent the force of better reason from prevailing.

  95. Sean,
    we only have access to reality by forming concepts about it in our minds. By your logic nothing exists. But this is not important to the discussion anyways. If you prefer to think that the universe had no beginning, so be it. I don’t see why that is important for dualism.
    So you believe the quantum world does not provide an interface with the supernatural. Maybe you are right, maybe not, but again it is not important and I don’t feel like arguing about it. It was only one option out of many.
    How do I explain the primate behavior versus human behavior? I am not sure I understand your point. The primates behave instinctively. They have strong group coherence. That does not keep them from what we would call infanticide as a reproductive strategy. Primates do not have a way to assess the morality of an act. They simply act. They are not conscious of their acts. They can’t differentiate between subjects and objects. Hence, they cannot have a conscience. Humans have free will and that allows them to do evil, as you pointed out. They can override their instinctive behavior for good or bad.

    In your last paragraph you make a case for a utilitarian view of morality. It reminded me of what Sam Harris says about morality. The problem I have with that is that once I understand that this is all that morality is, it becomes subjective and its normative power is gone for me personally. There are no absolute prohibitions that I have a motivation to follow in all circumstances. The ‘force of the better reason’ will compel me to eliminate competitors for important resources by whatever means necessary, if my survival or that of the group I identify with demands it. History is full of that sort of behavior. That is a massive can of worms, in my opinion, but has nothing to do anymore with the argument from reason.

    Your sentence on human dignity is a tricky one. What then about the folks who do not behave in a civilized matter? What about those who have little capacity to reason. Has a patient with dementia who cannot act according to the rules of our civilization anymore and cannot reason anymore lost his or her dignity? Has a supercomputer that runs an AI program and fulfills an important role for society some sort of dignity that we should respect? The answer is probably already visible in our society where many dedicate an enormous amount of time to interaction with machines and seem to be oblivious to the mentally ill outcasts from civilization. You know, the raving guys on the street corners that everyone tries their best to avoid, because they are seemingly worth nothing.
    I’ll leave it at that. It would be an interesting topic for a different thread.

  96. Sean – my last post addressed to you simply addressed the issue as to whether or not what you claimed to be examples of valid reasoning in your previous post were in fact valid – and clearly they were not. I did not address in that post the issue of the relationship between validity and knowledge of the external world – although I think my argument regarding logically possible worlds earlier in this thread addressed that issue. And a logical argument only assuredly gives rise to a true conclusion in the event that the premises are true – which we then refer to as a sound argument.

    Logic does have something to do with reality – given true premises, we may reason our way to a true conclusion that may not be intuitively obvious – working out the ramifications of quantum theory or of the general theory of relativity is hard work. And that’s not to dispute the issue that the premises have to be true – and that’s generally going to be decided from our perspective by empirical evidence.

    And no, you can’t arbitrarily decide upon a system of logic, or at least one that is useful – although there are different versions of formal logic. If you redefine 783 = 5, then you’re simply redefining these symbols in a manner that is apparently useless – now what? You have to distinguish between straight logic & mathematics which in the case of arithmetic introduces the certain axioms such as the Peano axioms in which to generate its truths and these axioms conflict with the idea of 783 – 5. There are different mathematical systems depending upon the choice of axioms. I gave you some examples of logical operators – try demonstrating a valid argument using them that fails even though the premises were true. I don’t know why you think that by arbitrarily redefining our terms that you are demonstrating failures of logic. If you redefine the word “not” to mean the opposite or that it meant no applied only 50% of the time – then so what? That doesn’t vitiate all of logic. We’d still find it useful to take advantage of another symbols that had the effect of absolutely negating some assertion.

    & CatholicScientist – “we only have access to reality by forming concepts about it in our minds” – well that’s obviously untrue – we can simply experience reality. Are you suggesting that animals w/o sophisticated linguistic skills have no access to reality?

    “The primates behave instinctively” and so do humans. However both animals (at least the more sophisticated one) and humans have the ability to reason and to deliberate and there’s at least half a a dozen or more that are capable of recognizing themselves (you should be familiar with the mirror test) although one of the criticisms of the mirror test is that there may be animals with a sense of self that are not captured by the mirror test. Infanticide as a reproductive strategy was used by a multitude of our human ancestors – and animals of course do not have access to abortion clinics and generally live in a highly competitive environment with nasty consequences for failure. Humans have incredible riches and so generally aren’t put to the test – but for our ancestors, infanticide may well have been the difference between life and death. If you look to failed states or environments run by criminal gangs, the veneer of civilization rapidly falls off.

    “Primates do not have a way to assess the morality of an act.” – sure they do. Wrong them and see what happens.

    Now humans have vastly superior reasoning and linguistic skills than other species – but that doesn’t make such skills in any way more magical than the skills that they bring to the table – consciousness itself in all likelihood being currently the most mysterious. Studies show that our vaunted moral reasoning is more likely to result from our gut visceral instinctive reactions (like other species) and then we use so called moral reasoning to rationalize a conclusion that we’ve already reached (that’s demonstrated by brain studies using fMRI & other technologies). There really isn’t any great difficulty to explain both why we have morality and how it functions from a secular viewpoint. Check out or google “evolutionary games theory”, for example.

    “In your last paragraph you make a case for a utilitarian view of morality” – no, he’s painting a picture of a much broader understanding of morality of which utilitarianism happens to be one possible element or theory – one which I agree with you is at least somewhat problematic. Actually, what Sean is proposing is more of a implied contract theory similar to Hume’s proposal. And if you think you’re going to turn into a sociopathic monster simply because there’s no God to ship you into eternal horror, then by common normative standards, you are a bad person. And if you made that choice, there would likely be consequences including the disappointment of your spouse and loved ones not to mention the possibility of police enforcement given sufficiently dire breaches of protocol. And one of the functions of morality is the enforcement by society of societies’ standards – something we see happening with chimpanzees as well – & that includes the use of punishment and positive reinforcement. Humans are well evolved to live within society )(ie. we’ve self domesticated) and that includes features such as empathy, loyalty, a sense of fairness, etc., (not unlike animals) plus we engage in a lengthy educational/enculturation process to engrain a moral code to fit us for living within society. Animal societies are simpler but demonstrate the same features but at a more primitive level – even mice demonstrate altruism if you reviewed the links I supplied earlier.

  97. Hi David,

    I NEVER claimed that the examples I gave were examples of valid reasoning. NEVER EVER. The examples I gave were to show how something can be “formally valid”, or should we say logically valid?, and yet clearly untrue. That was the point of the post. You have to go outside of the realms of logic and use rational reasoning to know that “human cells are invisible to the naked eye” is not always true.

    As I said we are talking tautologies here.
    Tautology Logic ;a statement that is true by necessity or by virtue of its logical form.

    In the numbers example I stated 2 = a number.

    You can fight this all you like, stomp your feet, cry that you have consulted with all of the math Profs in the western hemisphere, you name it, it matters not.

    Here is your example of a logical argument. all men are mortal, Socrates is a man, therefore Socrates is a mortal.Rubbish,David, Socrates is my wife’s cat. He is sitting on the window sill as I type this. How is this argument valid? All men are mortal. Socrates is a cat. therefore Socrates is mortal.

    You are going to tell me, don’t be a jackass Sean this is a tautology.I don’t care who your wife’s cat is. I do not care if Socrates ever lived. I do not care if all men are mortal.The argument is logically valid. It has nothing to do with reality.

    I am going to tell you, don’t be a jackass David this is a tautology. I don’t care what value the symbol 2 represents. I do not care what you think a number should be. The argument is logically valid. It has nothing to do with reality..

    Just as I cannot argue that Socrates is not a man. You cannot argue that 2 is not a number. For to do so is to go outside of logic into the world of rational inquiry, which has nothing to do with a logically valid argument.
    I think I did not make it clear that what I was trying to show is that logical arguments do not = truth. .

    You state,”Logic does have something to do with reality – given true premises, we may reason our way to a true conclusion” This is just it David; who is going to validate the premise? This is why I wrote that a human observer will have to validate the premises. Where in Fr Spitzers extensions of the laws of logic,(the means he uses to get to the God he postulates) is a place for validation? There isn’t. He bases his arguments on logic alone just as you have in your example, using tautologies.

    Perhaps I should gather my thoughts more carefully to make my point instead of doing other stuff whist typing posts.

    You write,”And no, you can’t arbitrarily decide upon a system of logic, or at least one that is useful.”
    Is it still useful if all it does is win an argument David? 🙂

  98. “A deductive argument is said to be valid if and only if it takes a form that makes it impossible for the premises to be true and the conclusion nevertheless to be false. Otherwise, a deductive argument is said to be invalid.

    A deductive argument is sound if and only if it is both valid, and all of its premises are actually true. Otherwise, a deductive argument is unsound.”

    http://www.iep.utm.edu/val-snd/

    “In logic, an argument is valid if and only if it takes a form that makes it impossible for the premises to be true and the conclusion nevertheless to be false. It is not required that a valid argument have premises that are actually true, but to have premises that, if they were true, would guarantee the truth of the argument’s conclusion. A formula is valid if and only if it is true under every interpretation, and an argument form (or schema) is valid if and only if every argument of that logical form is valid.”

    https://en.wikipedia.org/wiki/Validity

    “Validity is a property of arguments.

    An argument is valid just if it would be impossible for its premises all to be true and its conclusion false simultaneously.

    Or, which comes to the same thing:

    An argument is valid just if the set consisting of its premises and the negation of its conclusion is inconsistent.”

    http://logic.philosophy.ox.ac.uk/tutorial1/Tut1-07.htm
    This is philosophy of logic 101. It’s not something that is even remotely debatable. This is what it means to say that an argument is logically or formally valid.

    shall I pull up more authorities? An argument can be valid and yet give rise to false conclusions if the premises are false. An argument that has true premises and is logically valid will always give rise to a true conclusion. In your example where the reference to Socrates is to your cat – then the premise is false notwithstanding that the argument is valid.

    >>who is going to validate the premise?<< doesn't matter – the premises are either true or false whether or not anyone actually knows it – truth is not subjective – our evaluation of whether or not something is true may well be subjective but that's a very different question

    "Let us test this notion that logic can find truth. Here are examples of logically consistent, sound and logically valid deductive arguments given to me some time ago by somebody that gave me a good schoolin on truth vs logic"

    The example you then give is neither valid nor sound, it's as simple as that. You are not using the terminology in accordance with standard English usage. The person who schooled you on the difference between truth vs logic did not do a good job.

  99. Hi CatholicScientist (CS),
    Thank you for your reply.
    You write, “we only have access to reality by forming concepts about it in our minds. By your logic nothing exists.”
    I agree that we experience nothing directly CS. I defined “exist” for you. You are the guy that seems to be having problems defining “exist” so that it can be used coherently and consistently in our discussions.
    Object = that with shape/form
    Concept = relations between objects
    Exist = Object + location

    Please understand that what you or I think about the universe having a beginning or not is irrelevant, the best we humans can do with our very limited senses is to have a rational explanation IE one without contradictions. If we cannot identify a beginning of existence for anything, (even an omelette!), then how do we rationally speak of a beginning of existence for everything? I mean we do not even have a definition of what “exist” is from you.

    You write, “So you believe the quantum world does not provide an interface with the supernatural”
    No CS, what you or I believe is irrelevant. The wave function has been found to collapse upon interaction with the experimental apparatus whether a conscious being was there or not. Therefore we have no rational reason for thinking that consciousness interacts at a quantum level. Our beliefs have nothing to do with it.

    Regarding primates you state, “That does not keep them from what we would call infanticide as a reproductive strategy.” Indeed it did not keep Eskimo’s from infanticide either as a strategy.

    https://www.jstor.org/stable/2799344?seq=1#page_scan_tab_contents

    You state that “Primates do not have a way to assess the morality of an act. They simply act. They are not conscious of their acts.” This is not true, please watch this presentation.

    You state, “They (referring to primates) can’t differentiate between subjects and objects and hence cannot have a conscience” They certainly seem to differentiate CS, or how could they associate the lever for a reward for food with the shock it delivers to their mate. Here is interesting reading, and gives the yes or no answer based on your definition of conscience.

    https://www.quora.com/Why-do-humans-have-a-conscience-Is-there-any-evidence-to-support-the-idea-that-other-animals-also-possess-a-conscience

    I would like to sum this up according to your reasoning; The Rhesus monkeys that were more likely to starve than inflict a shock on their fellow primates were those that had experienced being a shock victim, and did this because …. (I quote you here)The primates behave instinctively. They have strong group coherence
    The human apes(over 600 of them) that inflicted shocks to their fellow man because they were instructed to do so by an authority figure and did this because…..(I quote you here)” Humans have free will and that allows them to do evil,….. They can override their instinctive behavior for good or bad.”

    The research says that our close cousins are more like us than we would like to think. Science has an explanation for the differences between apes and humans, which are the differences found in our brains.(check previous ref) Furthermore these differences can be traced back to the better developed and additional capacities we have over our ape cousins, because brain injury in the additional human brain structure affects these better developed capacities that we have. EG language.

    CS, I think theists, Fr Spitzer included, are trying too hard to distance yourselves from your inbred close cousins that live in the misty mountains and sometimes behave badly. Don’t be embarrassed CS, after all, even the best of families have a bit of Jerry Springer in them.

  100. Hi David,

    Let us start at the beginning, go through this s-l-o-w-l-y with me.
    CatholicScientist referred a video to me where Fr Spitzer uses the extensions of logic to prove that there is a God. William Lane Craig does the same thing with his Kalam argument to prove a creator. WLC states if you accept his premises you have absolutely no choice but to accept the conclusion because he is giving us a “logically tight” IE ‘valid’ argument. I gave the following example of what appears to be a so called logical argument which had premises that appeared to be true and the conclusion false to show that so called logical arguments do not = truth.

    P 1: The human body is made up of cells.
    P 2: Cells are invisible to the naked eye.
    C: Therefore, the human body is invisible to the naked eye.

    To show that one cannot use “logic” alone to debunk this argument I gave a similar argument with a “true” conclusion.How do you know that the human body is invisible to the naked eye? Through logic? Of course not, we know this through observation.

    P1: Air is comprised of atoms.
    P2: Atoms are invisible to the human eye.
    C: Therefore, air is invisible to the human eye!

    CatholicScientist dismissed these arguments as non-sequitur. I stated that looking at examples of non-sequitur arguments I did not see any that resembled my arguments.
    In formal logic,an argument with a conclusion that does not follow from its premise is a non sequitur, the conclusion could be either true or false (because there is a disconnection between the premise and the conclusion), but the argument nonetheless asserts the conclusion to be true, and is thus fallacious.

    I suppose that all invalid arguments are special cases of non sequitur.We know exactly what invalidity in an argument is:IE: to have true premises and a false conclusion.

    Is the example of the air/atom argument invalid? In other words does it have true premises and a false conclusion? Apples are also comprised of atoms. Are apples invisible to the naked eye? No, of course not. Therefore the air/atom argument although valid, with a true conclusion, it has a premise that is clearly not true. The human cell argument too is valid although it has a false premise and a false conclusion. An invalid argument is one with true premises and a false conclusion.Neither of these arguments given by me are so.

    Example; valid argument with false premises and false conclusion;Dogs are cats. Cats are birds.Therefore, dogs are birds.

    Example; Valid argument with false premises and a true conclusion.Cats are birds.Birds are mammals.
    Therefore, cats are mammals.

    From Peter Suber, “truth and validity” Quote.
    “Logicians tell us whether reasoning is valid, not if something is true,or not……But to test for invalidity, we must know when we are dealing with true premises and a false conclusion…… logicians do not know whether statements are true or false. (They are not empirical scientists or private detectives.)

    “We should never be misled by true premises or true conclusions to suppose (automatically) that an argument is valid. Nor should we be misled by false premises or false conclusions to suppose that it is invalid. Nor should we be misled by valid reasoning to suppose that statements are true, or by invalid reasoning to suppose that statements are false. If we recognize this, then we have already far surpassed “common sense” in protecting ourselves from deception.

    Truth and validity are combined in the concept of soundness. An argument is sound if (and only if) all its premises are true and its reasoning is valid; all others are unsound. It follows that all sound arguments have true conclusions.” End quote.

    You write, “>>who is going to validate the premise?<< doesn't matter – the premises are either true or false whether or not anyone actually knows it – truth is not subjective – our evaluation of whether or not something is true may well be subjective but that's a very different question"

    I am confused, the word “truth” is a concept which has been conceived by humans for use as a conceptual label of validation on statement types known as propositions.

    Propositions are statements which propose an alleged case or scenario.

    But since truth ultimately stems from the validation of propositions, it necessitates an observer who must VALIDATE the proposition before they can label it as ‘true’ or ‘false’.

    It is obvious that the word “truth” is ultimately dependent on a dynamic process that an observer must perform before labeling a proposition as true/false.

    Since the concept of truth is ultimately dependent on a human’s subjective use of their limited sensory system, it is easy to understand why all truths are subjective.

  101. Hi Sean
    I haven’t looked at the video but there’s a decent albeit somewhat summary analysis & critique of Craig’s Kalam cosmological argument at:

    https://en.wikipedia.org/wiki/Kalam_cosmological_argument

    I think that I’d already suggested some of the standard counterarguments earlier in this thread. I don’t think this link provides a comprehensive list of counter arguments but it does a reasonably decent job. At the end of the day, Craig’s argument seems to be one based upon an argument from ignorance plus a bald or unwarranted assertion.

    ” We know exactly what invalidity in an argument is:IE: to have true premises and a false conclusion” That’s an example of an invalid argument – but you could also have false premises and a true conclusion and the argument would still be invalid – or false premises and a false conclusion & it would still be invalid. A valid argument GUARANTEES that one’s conclusion will be true if it’s premises are true based upon the formal structure of the argument. You examples P1, P2 & c are not examples of a genuine valid argument and your claim: ” An invalid argument is one with true premises and a false conclusion” is true only of some invalid arguments but there are other arguments that are equally invalid which your claim doesn’t include because even if the premises had been true, the conclusion would not be guaranteed based upon the form of the argument.

    so: Grass is blue Horses are red. Therefore humans are invisible would have false premises and false conclusion and would nevertheless be an invalid argument. Grass is blue. Horses are red. Therefore humans are visible has a true conclusion but iit nevertheless has the same form as my immediately preceding argument and is therefore invalid.

    Your argument “cats are birds. Birds are mammals. Therefore cats are mammals is ok it would be better to have said “all cats are birds” and “all birds are mammals” and then drawn your conclusion, otherwise there’s a potential ambiguity.

    “I am confused, the word “truth” is a concept which has been conceived by humans for use as a conceptual label of validation on statement types known as propositions.”

    The term “proposition” is intended to refer to the meaning of a sentence or a belief. If you do a bit of research, you will find that a number of logicians will reject this concept (a proposition) by reason that (they claim) there is no referent (ie. propositions are fictions and don’t actually exist)- and will apply the term “truth” to either a sentence or a belief or other truth holder. I agree with this usage although I don’t think it matters for the purpose of your issue.

    “But since truth ultimately stems from the validation of propositions”. That isn’t true. Assuming you intend the word “validation’ and not ‘validity”, then what I think you are saying is that for a sentence or belief to be true, we have to be in a position to justify (validate) the sentence or belief. But the whole concept of validating or justifying a sentence or belief is the idea that we have to investigate and determine whether or not the sentence or belief is true – the actual truth or falsity of the sentence is a feature of the world – our beliefs or knowledge of a given feature of the world is subjective – I may have a belief and you don’t.

    I may ascribe truth to a given sentence or belief and be wrong – the question of whether or not I’m wrong is not something that is subjective but is a separate and objective property of the sentence or belief whether or not I’m in a position to validate the claim. I personally subscribe to the correspondence theory of truth which I understand was and perhaps still is the running favourite – you’ll find a summary of its strengths and weaknesses at http://plato.stanford.edu/entries/truth-correspondence/

    “It is obvious that the word “truth” is ultimately dependent on a dynamic process that an observer must perform before labeling a proposition as true/false.”

    why couldn’t you have a computer randomly spitting out sentences and labeling each one (also randomly) true or false?

Leave a reply to Zia Cancel reply